You are on page 1of 171

CONTENTS

How to Use This Book IV

Acknowledgments IV

CHAPTER 1 The Geometry of Euclidean Space 1


CHAPTER 2 Differentiation 21
CHAPTER 3 Higher-Order Derivatives; Maxima and Minima 43
CHAPTER 4 Vector-Valued Functions 63
CHAPTER 5 Double and Triple Integrals 77
CHAPTER 6 The Change of Variables Formula and
Applications of Integration 97
CHAPTER 7 Integrals over Paths and Surfaces 17
CHAPTER 8 The Integral Theorems of Vector Analysis 143
CHAPTER 9 Sample Exams 161
APPENDIX Answers to Chapter Te ts and Sample Exams 167
1

THE GEOMETRY OF EUCLIDEAN SPACE

1.1: VECTORS IN TWO- A ND T H REE-D IM ENSIO N A L SP ACE

GOALS

1. Be able to perform the following operations on vectors: addition, subt raction , scalar multipli­
cation.

2. Given a vector and a point, be able to write the equation of the line passing through the point
in the direction of the vector.

3. Given two points, be able to write the equation of the line passing through them.

STUDY HINT S

1. Space notation. T he symbol ]R or ]R I refers to all points on the real number line or a one­
dimensional space. ]R 2 refers to all ordered pairs (X , y) which lie in the plane, a two-dimensional
space. ]R 3 refers to all ordered triples (x, y, z) which lie in three-dimensional space. In general,
the "exponent" in ]R71 tells you how many components there are in each vector.

2. Vectors and scalars. A vector has both length (magnitude) and direction. Scalars are just

~~
~u mbers . Sc~lars do not have direction. Two vectors are equal
If and only If they both have the same length and the same
direction . Pictorially, they do not need to originate from the
same starting point. The vectors shown here are equal.

3. Vector notation. Vectors are often denoted by boldface let ters , underlined letters, arrows over
letters, or by an n-tuple (Xl, X2 , ... , x 71 ) . Each Xi of the n-tuple is called the lth component.
BEWARE that the n-tuple may represent either a point or a vector. T he vector (0,0, ... , 0) is
denoted O. Your instructor or other textbooks may use other notations such as a squiggly line
underneath a letter. A circumflex over a letter is sometimes used to represent a unit vector.

4. Vector addition. Vectors may be added componentwise , e.g., in ]R2

+ (X2, Y2) = (Xl +YI,X2+Y2).

;;;?J
(XI , yr)

Pictorially, two vectors may be thought of as the sides of a


parallelogram. Star ting from the vertex formed by the two
v vectors, we fo rm a new vector which ends at the opposite corner
of the parallelogram. This new vector is the sum of the other

£?:j
U
U + V
I

_ _ _ _ _ _ __ 1

I
two. Alternatively, one could simply translate v so that the
tail of v meets the head of u . The vector joining the tail of u
to the head of v is u + v .
2 CHAPTER 1

5 . Vector subtmction . Just as with addition, vectors may be su btracted componentwise. Think
of this as adding a negative vector . Pictorially, the vectors a , b
a b and a - b form a triangle. To determine the correct direction ,
hb ~
a- you should Ibe able to add a - b and b to get a. Thus a ­ b
~ go," f<om the tip of b to the tip of 8 .

6 . Scalar multiplication. Here, each component of a vector is multiplied by the same scalar, e.g.,
in JR. 2,
r(x , y) = (rx, ry) for any real number r .

The effect of multiplication by a positive scalar is t.o change the length by a factor. If the
scalar is negative, the lengthening occurs in the opposite direction. Multiplication of vectors
will be discussed in the next two sections .

7. Standard basis vectors. These are vectors whose components are all 0 except for a single 1. In
JR.3, i, j and k denote the vectors which lie on the x, y and z axes. T hey are (1, 0, 0), (0 , 1,0)
and (0,0 , 1), respectively. The standard basis vectors in JR. 2 are i and j , which are vectors lying
on the x and y axes, and their respective components are (1 , 0) and (0,1). Sometimes, these
vectors are denoted by

+ tv. This is called the


8. Lines. (a) The line passing through a in the direction of v is l (t) = a
point-direction form of t he line because the only necessary information is the point a and the
direction of v .

(b) The line passing through a and b is l(t) = a+t(b-a) . This is called the point-point fo rm
of the line. To see if the direction is correct , plug in t = 0 and you should get the first point.
Plug in t = 1 and you should get the second point.

9. Spanning a space. If all points in a space can be written in the form A I V I + A2 V 2 + . .. + An V n ,


where Ai are scalars, then the vectors VI, . . . , Vn span that given space. For example, the
vectors i and j span the xy plane.

10. Geometric proofs. The use of vectors can often simplify a proof. Try to compare vector meth­
ods and non-vector methods by doing example 10 without vectors.

SOL UTIONS TO SELECTED E X ERCISES

1. We must solve the following equations:

-21- x - 25
23 - 6 y.

We get x = 4 and y = 17, so (-21,23) - (4,6) = (-25,17).

4. Convert -4i + 3j to (- 4, 3,0), so

(2,3,5) - 4i + 3j = (2,3,5) + (-4 , 3,0) = (-2 , 6, 5).


THE GEOM ETRY OF EUCLI DEAN SPACE 3

7. To sketch v, tart at the origin and move 2 units along the x


axis , then move 3 units parallel to the y axis, and then move
-6 units parallel to the z axis. T he vector w is sketched anal­
ogously. The vector - v has the same length as v, but it points
in the opposite direction . To sketch v + w , t ranslate the tail
of w to the head of v and draw the vector from the origin to
the head of the translated w . The vector v - w goes from the
y head of w to the head of v.

9. On t he y axis, points have the coordin at s (0 , y, 0) , so we


z (~~~'51 _ . (O,y,z) must restrict x and z to be 0. On the z axis, points have
the coordinates (0,0, z) , so we must restrict x and y to be
(x,O,z)/ / y
0. In t he xz plane, points have the coordinates (:I: ,O , z). so
(
(o, y, o) we must restrict y to be 0. In the yz plane, points have the
x coordinates (0 , y , z ), so we must restrict x to be O.
12. Every point on the plane spanned by the given ve tors can be writ ten as aV I + bV2, where a
and b are real numbers; therefore, the plane is described by

a(3 , -1,1) + b(O , 3,4 ).


15. Given two points a and b , a line through them is l(t) =a+ t(b - a). In this case, a =
(-1 , - 1, -1) and b = (1, - 1, 2), so we get

l(t ) = (-1, -1, -1) + t(2, 0, 3) = (2t - 1, -1, 3t - 1).

19. Substitute v = (x, y , z) = (2 + i, - 2 + t, -1 + i) into the equation for x, yand z and get
2:1: - 3y + z - 2 2(2 + t) - 3(-2 + t) + (- 1 + t) - 2
4 + 2t + 6 - 3t -- 1 + t - 2 = 7.

Since 7 f. 0, there are no points (:I: , y , z) satisfying the equation and lying on v .
23. J ust as the parallelogram of example 17 was described by v = sa + tb for sand t in [0, 1], the
paralielpiped can be described by w = sa + t b + rc, fo r s, t and r in [0 , 1] .
28 . Let a , band c be the sides of the triangle as shown , and let
Vij denote the vector from point i to poin j . We assume that
each median is divided into a ratio of 2 : 1 by the point of
intersection. Then we have

V I2 -a/ 2 = - (c - b) / 2;
1 V23 = (1 / 3)(a / 2 + b );
V34 (- 2/ 3)(a+b/ 2);
V45 (a + b )/ 2.

The vector VIS should be the sum V1 2 + V23 + V34 + V45, or

1 (a ) 2 ( b) a
V1 5 = -c-
- b
2- + 3 2 + b - 3 a+ 2' +
+ b
- 2-
c
= b - 2'
which is the median of the vector that ends on c. The other two median ' are analyzed the
same way.
4 CHAPTER 1

30. (a) Using x, the number of C atoms; y, the number of H atoms; and z, the number of 0 atoms,
as coordinates, we get

p(3, 4, 3) + q(O, 0, 2) = r(l, 0, 2) + s(O, 2,1).


{b) To find the smallest integer solution for p, q, rand s, we balance the equation componen­
twise:

3p= r (equating x)

=
2s 4p i. e., s = 2p (equating y)

2q + 3p = 2r + s ~ 6p+ 2p i.e., q = (5/2)p. (equating z)

Let p = 2, then the smallest integer solu t ion is p = 2, q = 5, r = 6, s = 4.


(c) In the diagram, P is (6,8 ,6), Q is (0,0,10) , R is (6,0 ,1 2) and

S is (0,8,4). Both sides of the equation add up to the vector

(6,8,16).

p y

1.2: THE INNER PRODUCT, LENGTH AN D DISTANCE


GO A LS
l. Be able to compute a dot product.

2. Be able to explain the geometric significance of the dot product.


3. Be able to normalize a vector.
4. Be abl e to compute the projection of one vector onto another.

STUDY HINTS
l. Inne1' product. This is also commonly called the dot product, and it is denoted by a . b or
(a, b). The dot product is the sum 2.:::7=1 ajbj, where aj and bj are the ~th components of a and
=
b , respectively. For example, in lR 2, a . b al b1 + a2b2. Note that the dot product is a scalar.

2. Length of a vector. The length or the norm of a vector x = (x, y, z) is J


x 2 + y2 + z2. It is
denoted by Ilxll and is equal to VX-:-X. This is derivable from the fact that x . y = X1YI +
X2Y2 + X3Y3 with x = y.

3. Unit vector. These vectors have length l. You can make any non-zero vector a unit vector by
normalizing it. To normalize a vector, divide the vector by its length, i.e., compute a/liali .
4. Cauchy-Schwarz inequality. Knowing that la· hi ~ Ilallllbll is most im portant for doing proofs
in the optional sections of this text and in more advanced cou rses. .
5. Important geom etric properties. Know that a· b = Ilallllbll cos e, where B is the angle between
the two vectors. As a consequence, a . b = 0 implies that a and b are orthogonal. The zero
vector is orthogonal to all vectors.
6. Projections. The orthogonal projection of b onto a is the "shadow" of b falling onto a . The
projection of b onto a is a vector of length (a· b)/llall , in the direction of a/liali. Thus, the
projection of b onto a is
(a ' b)) a (a·b)
( -W W=~ a.

.. .

THE GEOMETRY OF EUCLIDEAN SPACE 5

7. Problem solving. Since vectors have magnitude and direction, they can be represented picto­
ri ally. It is often useful to sketch a diagram to help you visualize a vector word problem .

SOLUTIONS TO SELECTED EXERCISES

3. From t he defin it ion of t he dot product, we get

cos (j =
u·v = (0 7 19) · (- 2 - 1 0)
' , " =
-7

~ -0 1546
Ilullllvll V 72 + 192 V22 + 12 V410V5 . .

From a hand calculator, we find that (j ~ 99° .

7. If w = ai + bj + ck, then Ilwll = va2 + b2 + c2 , and so

Ilull = vT+4 = ..)5; Ilvll = v1+l = h .

Using t he formula for the dot product, we get

u· v = (-1 )(1) + (2)( - 1) = - 3.

10. Using t he same fo rmulas as in exercise 7, we get

Ilull = VI + 0 + 9 = JiO; Il vl l = v O+ 16 + 0 = 4.

Since u does not have a j component and v does not have any i or k co mponent, the vectors
are perpendicular; therefore n . v = O.

12. A vector w is normalized by constructing the vector w/ll wll. For the vectors in exercise 7:

u v
W =
1 ( . 2 ')
V5 -) + J; IRI 1 (' ')
= V2 ]- J .

15. T he projection of v onto u is

n·v _(-1)(2)+( 1)( 1) + (1)( - 3)(_, , k)-_i(_, , k)


Ilul1 2u - (V 1 + 1 + 1)2 l+J+ - 3 I+ J + .

16. For orthogonality, we want the dot product to be O.


(a) The dot product is (2i + bj ) . (- 3i + 2j + k ) = -6 + 2b , so b must be 3.
(b) The dot product is (2i + bj ) . k = 0, so b can be any real number.

21. (a) Looking at the x componen s, the pilot needs to get from 3 to 23 . His velocity in the x
direction is the i component , 400 km/ hr. T hus,

Llt = ~d = 23 - 3 = ~.
v 40 0 20
The pilot flies over the airport (1/ 20) hour or 3 minutes later. The sam e answer could have
been obtained by an alyzing the y components.
(b) We loo k at the z components and use the formula

~d = vLlt , i.e., h- 5 = (- 1)( 1/ 20) ,

so h = 99/20 . Thus, the pilot is 4.95 km above the airport when he passes over.

24. (a) It is convenient to draw the diagram with A on the .r axis.


6 CHAPTER 1

A x

(b) From the diagram in part (a), we get

A = 150i and B = (llO cos 60 0


)i + (llO sin 60 0 )j.

A + B = (150 + llO cos 60 0 )i + (llOsin 60 0 )j = 205i + 55V3j.


The angle that A +B makes with A is

() = tan- 1 ( ; ) = tan- 1 (5~:) ~ t an - 1 (0.4647) ~ 250.


Alt.ernat.ively, the definition of the dot product gives us

() = cos- 1 ( A · (A + B ) ) = cos- 1 (1 50)(205) + (0)(55V3)


IIAII IIA + BII (150)(J51l00)
~ cos-l(O.gO~g) ~ 25° .

27. (a) Geometrically, we see that the i component of F is

IIFII cos (). Similarly, the j component of F is IIFII sin B. T here­ y

F
fore,

=F IIFII cos ()i + IIFII sin ()j,

where () is the angle from the x axis. Since the angle from t he

y axis is 7r/4, () is also 7r/4, so F =


3J2(i + j ).

(b) We compute D = 4i+ 2j, so F·D = (3)2)(4) +(3)2)(2) =

x
18"\/2. Also , IIFII = 6 and IIDII = J20. From the definition of

the dot product,

F ·D 18)2 3
cos() = IIFII IIDII = 6J20 = vT5 ~ 0.9487,

i. e., () ~ 18°, or equi valently, () ~ 0.322 radians.


(c) From part (b), we had computed F· D =
18)2. Knowing t hat cos () = 3/.JfO, we calculate
= =
IIFII liDII cos() (6J20)(3/JIO) 18)2, also .

1.3: M ATRICES , DETERMINANTS AND THE CRO SS PROD UC T


GOALS
1. Be able to compute a cross product.

2. Be able to explain the geometric significance of the cross product.


3. Be able to write the equation of a plane from given information regarding points on t he plane
or normals to the plane.
TH E GEOMETRY OF EUCLIDEAN SPACE 7

STUDY HINT S
1. Ma tri ces and determi1wnts. A matrix is just a rectangular array of numbers. The array is
written between a set of brackets. The determinant of a matrix is a. number; a matrix has
no numerical value. The determinant is defined only for square matrices and it is denoted by
vertical bars.

2. Computing determin ants. K now that I~ ! I= ad - be. Also know that

abc
d e f
9 h
Note the minus sign in front of the second term on the right-hand side. The general method
for computing determinants is described next .
3. Computing n x n determinants. Use the checkerboard pattern shown here which begins with
a plus sign in the upper left corner. Choose any column or
row - usually picking the one wit,h the most zeroes saves work.
+ + Draw vertical and horizontal lines through the first number of
+ t he row or colum n. The numbers remaining form an (n -1) x
+ + (n -1) determ inant, which should be multiplied by the number
(with the sign det erm ined by the checkerboard) through which
both lines are drawn . Repeat for the remaining numbers of the
row or column . Finally, sum the results . This process , called
expansion by minors , works for any row or column. The best
way to remem ber the process is by practicing. Be sure to use
the correct signs.
4. Simplifying determinants . Deter minants are easiest to compute when zeroes are present.
Adding a non-zero multiple of one row or one column to another row or column does not
change the value of the determin ant, and this can often simplify the computations. See exa.m­
pIe 3.
5. Computing a cross product. If a = (aI, a 2 , a3) and b = (b 1 , b2 , b3 ), then
J k
a xb = a1 a2 a3
b1 b2 b3
The order matters: a x b = -(b x a). The cross product is not commutative. Also, note that
a x b is a vector , not a scalar .
6. Properties of the cross produ ct. T he vectors a , b and a X b form a right-handed system (see
figure 1.3.2 of the text) . The cross product a x b is orthogonal to both a and b . The length
of a x b is IIallllbill sinOI, where 0 is the angle between a and b. Note that the formula for
the cross prod uct involves sin 0, whereas t he dot product involves cos O.
7. More properties. If the cross product is zero , then either: (i) t he length of one of the vectors
must be zero , or (ii) sinO = 0, i. e., 0 = 0, so the vectors must be parallel.

8. Geometry. The absolute value of the determinant I~ !I is the area of the parallelogram
spanned by the vectors (a, b) and (c, d) originat ing from the same point. The absolute value of
a b c
the determinant d e f is the volume of the parallelpiped spanned by the vectors (a, b, c),
g h i
(d, e, J) and (g, h, i) originating from the same point. The length of the cross product 11a x hll
is the area of the parallelogram spanned by the vectors a a nd b. The vector a x b gives a
vector normal to the plane spanned by a and b .
8 CHAPTER 1

9. Equation of a plane. Recall that the equation of the plane is ax + by + cz + d = O. The vector
(a , b, c) is orthogona] to the plane . Knowing two vectors in the plane , we can deter mine an
orthogonal vector by using the cross product. Compare methods 1 and 2 of example 11.
10. Distance from point to plane. You should understand the derivation of the equation in the
box preceding example 12. If necessary, review the geometric properties of the dot product in
section 1.2.

SOLUTIONS TO SELECTED EXERCISES


2. (b) We subtract 12 times the third row from the first row and subtract 15 times the third row
from the second row. 'fhen we expand along the first col umn:
36 18 17 o -42 41
o -51 -42 41
45 24 20 50 =3 -51 50
1
= 3(-2100 + 2091) = -27.
.- . ­

3 5 -2 3 5 -2 1

5. The area of the parallelogram is /l a x hll . We compute

j k
a x b = 1 -2 1 = -3i + j + 5k
2 1 1

and so the area of the parallelogram is

lin x hll = v'9 + 1 + 25 = 55.

8. The volume of the parallelpiped is the absolute value of the 3 x 3 determinant made up of t he
vectors ' components. Expand along the first row:

1 0 0
0 3 -1 - _1 3
2
-1
-1 'I =- l.
4 2 -1

Thus, the volume is 1.

11. We want to find the cross product and then normalize it . We compute

J k
v = -5 9 -4 I = 113i + 17j ­ 103k.
789
So

There are two orthogonal vectors in opposing directions; they are given by

±v/llvll =: ±(113i + 17j - 103k)/ v'23667.


Since all vectors are orthogonal to 0, the inclusion of that vector into the problem does not
affect the answer.
15. (a) The equation of a plane with normal vector (A ,B , C) and passing through the point
(xo, Yo, zo) is A(x - xo) + B(y - Yo) + C (z - zo) = O. In this case, the equation is
l(x - 1) + l(y - 0) + l(z - 0) =0 or x + y + z = 1.
(d) Here, the normal vector is parallel to the line, so it is (-1 , - 2,3). Hence, t he equation of
the desired plane is

-1(x - 2) - 2(y - 4) + 3(z + 1) = 0 or - x - 2y + 3z + 13 = O.


TH E GEOMETRY OF EU CLI DE AN SPACE 9

16. (b) Two vectors in the desired plane are v = (0 - 1, 1 - 2, - 2 - 0) = (-1,-1 , - 2) and
w = (4 - 0, 0 - 1, 1 + 2) = (4, - 1,3). The cross pr duct v x w is orthogonal to both vectors,
and hence normal to the desired plane. We compute
vxw = - 5i - 5j + 5k,

so the desired equation is

-5(x - 1) - 5(y - 2) + 5(z - 0) =0 or - x - y + z + 3 = O.

22. (a) Let D be the matrix with rows u, v, w . T hen


Ul U2 U3
det D = u· (vxw)= VI V2 V3
WI Wz W3

Use the fo llowing property of determinants: v· (w x u ) corresponds to two row exchanges of


the matrix D , 80 we hav
v . (w x u ) = (-1)( -1) det D = det D and w · (u x v ) = (-1)( -1) det D = det D.
To prove the other three, recall that u x v = -(v xu).
26. The line perpendicular to the plane is parallel to the normal of the plane, so t he equation of
the line is
l (t) = (1, - 2, - 3) + t (3, -1, -2).
29. Let u be the vector normal to the plane. Then u is perpendicular to 3i + 2j + 4k since v
is on the plane. Alsu u is perpendicular to 2i + j - 3k, because the vector Ai + Bj + Ck is
erpendicular to all vectors in t he plane Ax + By + Cz = D . To find u, we take t he cross
produ t of 3i + 2j + 4k and 2i + j - 3k:
J k
u = 32 4 =- 10i + 17j - k.
2 1 - 3
When t = 0, we find that a point on the plane is (- 1,1 , 2), so the equation of the plane is
- 10(x+ l) +17(y -l)- (z-2}=0 or - 10x + 17y-z - 25 =0 .

30 . First, find the normal ofthe plane. The norm al ofthe plane is perpendicular 0 the line passing
through (3, 2, - 1) and (1, - 1, 2). T he equation of the line is l(t) = (3,2, - 1) + t(2, 3, - 3). The
=
normal of t he plane is also perpendicular to v (1 , - 1, 0) + t(3, 2, -2). T herefore, two vectors
on t he desired plane are 2i + 3j - 3k and 3i + 2j - 2k, an the normal is (2i + 3j - 3k) x (3i +
2j - 2k) = - 5j + 5k. Now, we need a point on the plane, say, (3,2, -1). Thus, the equation
of t he plane is
O(z - 3) - 5(y - 2) + 5(z + 1) = 0 or - y + z = l.
34. T he plane passi ng through the origin and perpendicular to i - 2j + k is x - 2y +z = O. By
the dist ance formu la with (A, B , C , D ) = (1, - 2, 1, 0) and (Xl, Y1, zt) = (6 ,1, 0),

d= IAx l + BYI + C Zl + DI = 6 - 2 _ --±- _ 2V6


VA2 + B2 + C2 VI + 4 + 1 V6 3 '

37. Since all vectors in this exercise are unit vectors, liN x all = sin 01 and II N x hll = sin O2 . From
Snell's law, n l sin 01 = n2 sin O2 . Hence,
nlllN x all = n211N x hll.
To establish that N x a and N x h have the same direction, we assume t hat N, a and hall
lie in the same plane, and a and h are on the same side of N. Hence N x a and N x h both
are perpendicular to this plane and parallel to ach other . Thus nl llN x all and n 211N X hll
are equal.
10 CHAPTER 1

38. First, 4 times the first row is subtracted from the second row. Next, 7 times the first row is
subtracted from the third row . The next step is expansion by minors along the first column.
Finally, the 2 x 2 determinant is computed.

1.4: CYLINDRICAL AND SPHERICAL COORDINATES


GOALS
l. Be able to convert back and forth between the cylindrical, spherical and cartesian coordinate
systems.
2. Be able to describe geometric objects with cylindrical and spherical coordinates.
3. Be able to describe the geometric effects of changing a coordinate.

STUDY HINTS
l. Review. You should review polar coordinates in your one-variable calculus text .
. ~ .
2. Cylindrical coordinates. Denoted (r, {}, z), this is just like polar coordin ates except that a z
coordinate has been added. Know the formulas x = rcos{}, y = rsin{}, r =
Jx 2 + y2 and
{} ::::; tan-1(y/x).
3. Spherical cooridinates. Denoted by (p, {}, ¢», p is the distance from the origin, ¢> is the angle
from the positive z axis and {} is the same as in cylindrical coordinates. Know the formulas
x=pcos{}sin¢>, y=psin{}sin¢> and z=pcos¢>.
Also know t.hat
p = Jx2 + y2 + Z2, {} = tan-1(y/x) and ¢> = cos-1(z/ Jx 2 + y2 + z2) = cos- 1 (z/p).

4. Graphs of r, p = constant. Note that r = constant in cylindrical coordinates describes a


cylinder and that p = constant in spherical coordinates describes a sphere. You may have
suspected this from the name of the coordinate system.
5. Computing {} and ¢>. Remember that, in this text, ¢> takes values from 0 to rr and {} ranges
from 0 to 2iT. In some instances, it is more convenient to define {} in the range -rr to iT . You
should be very careful about computing {}. If x = y = -1, t.hen tan-1(y/x) =
iT/4, but plotting
the point (-1,-1) in the xy plane shows that, in reality, {} = 5rr/4. This is why the authors
fuss with tan- 1(y/x) in the definition. Plotting the x and y coordinates is very helpful for
determining {}.
6. Negative r, p. Note that we have defined r and p to be non-negative. If the distance is given
as a negative number, we need to reflect the given point across the origin .
7. Unit vector's in spherical and cylindrical coordinates. The unit vectors in cylindrical coordi­
nates are e r , eo and e z . The vector e r points along the direction of r, while eo goes in the
direction in which {} is measured , and e z = k. As one might expect., those three unit vectors
form an orthogonal basis, and e r x eo = e z . Those vectors, however, lire not fixed as is the
case with i, j and k , that is, if you change the point (7', {}, z), the set of unit vectors rotates.
For spherical coordinates, there is also a set of unit vectorse p , eo and e",. Those vectors , in
terms of i, j, k and the cartesian coordinates of the point are worked out in exercise 7 (see
below) of this section.
T HE GEOM ETRY OF EUCLIDEAN SPAC E 11

SOLUT IONS TO SELECTED EXERCISES


1. (a) To conver t to rectangular coordinates, use x = r cos & and y = 1'Sin 11:
x = 1cos 45° = v2/2 and y = 1 sin 45° = V2/2.
Next , use p =J x2 + y2 + z 2 and rP = cos - 1(z / p) to get the spherical coordin ates:

P :::;; V2"I+ 2"1+ 1= V2


Ir.
and ¢ = cos - 1 ( 1) = "4 '
)2 1l'

Hence, for the cylindrical coordinates (1,45°,1), the rect angular coordinates are given by
(/2/ 2,)2/ 2,1 ) and hesph r ical coordinates are (V2,rr/ 4, 1l'/ 4).
(b) To convert to cylindrical coordinates, we use l' = ..jX2 + y2 and (J = tan -1(y/x ):

r = .) 4+1 = v'5 and B = tan - 1 (1/ 2).

Next , use the same formulas as in part (a) to get t h spherical coordina tes:

p=V4+ 4+ 1 = 3 and ¢ = cos-1(- 2/ 3).


Hence, the rectangular coordin ates (2, 1, - 2) convert to the corresponding cylindrical coord i­
nat s (/5, t an- 1 (1/2), - 2) and t o the spherical coordinates (3, tan - 1 (1/2),cos-1(- 2/ 3)).

2. (b) T his mappin g takes a point a nd rotates it by 1l' radians about the z axis . This is followed
by a reflection across the xy plane. The net effect is that the point is reflected through the
origin.
3. (b) Recall that the angle ¢ is measured from the "Nor th pole." If ¢ is 7i radians , t hen t he
location is at the "Soutl pole." The effect. of changing ¢ to 1[' - ¢ is taking a point and reflecting
it across the x y plane.

5. Let p ~ 0, then (p, 0, 0) is the positi ve z axis. Now, let ¢ vary from 0 to 1l'. Then (p, 0, ¢) is
the half plane in the xz plane wi t h z ~ 0. By allowing & to vary fro m 0 to 21l', we rotate t he
°
half plane described above. Therefo re, p ~ 0, ~ (J < 21l' and 0 ~ ¢ ~ 1l' describes all points
in JR3.
If p < 0 also , the coordinates are not unique . For example, (x, y, z) = (1, 1,0) has spherical
coordinates (V2, 1l'/4,1l'/2) and (- V2, 51l'/ 4,1l'/2).

7. (a) First , e p is the unit vector along the vector (x, y, z) ; t herefore, the formula is

xi + yj + zk

ep = .
J x 2 + y2 + z2

z
y
-----k------~

y r

x x
(x, y,O)

Next, e o is parallel to the xy plane and denotes t he direction in which t he angle B is measured .
It is perpendicular to r = (x , y, 0), so (ai + bj) . (xi + yj ) = O. Since we want B measured
counterclockwise, a = -y (instead of y) and b = x. Therefore,
- yi + zj
eo = J x 2 + y2'
12 CHAPTER 1

To fi nd e ,p , we note that e p , eo and e,p is a set of orthogonal vectors; they form a right-handed
coordinate system with e p x eo = - e,p. So .

(x, y, z ) x (- y, x, 0) x zi + yzj - (x 2 + y2 )k
~ =-
rp
= J x 2 + y2Jx2 + y2 + Z2
.

9. (a) The length of xi + yj + zk is J x 2 + y2 + z2 , which is the definition of p.


(b) Note that Il v ll = Jx 2 + y2 + z2 = P and v· k = z; therefore, <p = cos~l(z/p) = cos-1(v.
k/llvll)·
(c) Note that Ilull = J x 2 + y2, which is the cylindrical coordinate rand u · i = x; therefore,
() = cos- 1(x/r) = cos- 1(u . i /llull).
13. Note that <p will be between 7r/ 2 and 7r because the region lies in the lower hemisphere. From

the triangle, we see th at cos a = (d/6) -;- (d/ 2) = 1/ 3; therefore, we have 7r - a ::; <p ::; 7r or
7r - cos-1(1/3) ::; ¢ ::; 7r. Now, p can be as large as d/ 2; however, as p gets smaller, its lower
=
limit depends on <p . P ick any ¢ , then ¢+ (3 7r and according to the diagram cos (3 (dj6) -;- p. =
Rearrangement gives

d/(6cos(3) = p = d/(6cos(7r - ¢)) = -d/(6 cos¢).


Therefore, -d/ (6 cos ¢) ::; p ::; d/2. So far , we have described the cross-section in one quadrant.
The entire volume requires a revolution around the z axis, so its description is

__d_ < p < ~ 0 < () - < 27r and 7r - cos- 1 (-31 ) <_ A, <
_ 7r.
6 cos ¢ - - 2' - 'i'

1.5: n-DIMENSIONA L EUC LIDEAN SPACE


GOALS

1. Be able to extend the ideas of the previous sections to jR n.

2. Be able to multiply matrices.

STUDY HINTS

1. The space ]Rn . Most of this textbook deals with the Euclidean spaces that we can visualize,
jR2 and jR3. Many of the same properties hold in jRn. Vector addition, scalar multiplication,
vector lengths, the dot product and the triangle inequality are defined similarly.

2. No cross product analog. The cross product in ]R3 does not have an easy analog in jRn, n ~ 4.

3. Standard basis vectors. The analogs of i, j, k are defined e j . The vector ej is (0,0, ... ,1, ...0)
with 1 in the zth position. The vectors ej and ej are orthogonal if i =f j.
THE GEOMETRY OF EUCLIDEAN SPACE 13

4. Matrices. A matrix is a rect angular array of num bers . Unlike a determinant, a matrix has no
numerical value. You should remember th at an n x m matrix has n rows and m columns. The
(i, j) entry is the number located in row i, column j.

5. Mat rix m ultiplication. You should practice until matrix multiplication becomes second nature
to you. Let the components of A be a i j and let those of B be bkl, where A is an m x p matrix
and B is a p x n matrix. Then the components of A B are
p
(A B )mn =L amjbjn .
j =l

We can only multiply an m x p mat rix with a p x n matrix, i.e., [m x p][P x n]. Note that
the number of columns of A and the number of rows of B must be equal (p in this case). The
result is an m x n mat rix ("cancelling" the p).

6. Non-commutativity of m atrix multiplication. In general, AB # B A . In fact, AB may be defined


when BA is undefined . However , matrix multiplication is associat ive; i.e., (AB)C = A(BC) if
the product AB C is defined .

7. Matrices and mappings. An m x n matrix can represent a mapping from ]Rn to ]Rm. To see
this, let A be the matrix and let x be a vector in ]R n, represented as an n x 1 matrix, and y
be a vector in ]Rm, an m x 1 matrix. T hen the matrix A takes a point in ]R n to a point in ]Rm
by the equation Ax = y.

SOLUTIONS TO SELE CTED EXERCI SES

2. (a) Use the properties of leng hs and dot products:

(x + y) . (x + y) + (x - y) . (x - y)
x . x + 2x . y + y . y + x . x - 2x . y + y . y
2x· x + 2y· y = 211xl12 + 211Y11 2.
The figure at the left depicts the equation geometrically. By

12<tl
the law of cosines, we have

y and

We also note that a + j3 = 1r, so j3 = 1r - a . T herefore , cos j3 = cos( 1r - a) = - cos a. Adding


the two equations from the law of cosines yields 211xl1 2 + 211Yl12 ;: Ilx - Yl12 + I'lx + Y112.

4. To verify the Cauchy-Schwarz inequality, we compute

Ix· yl 1(1)(3) + (0)(8) + (2)(4) + (6)(1)1 = 17.


Ilxll '1'1 + 0 + 4 + 36 = J41.
Il,yll J9 + 64 + 16 + 1 = V90.
Thus, we indeed have Ix .yl = v'I7v'I7 < v4fJ90 = II'xlillyll. For the triangle inequality, we
compute

x + y = (4,8,6,7) and IIx + yll = J16 + 64 + 36 + 49 = v'l65 < 13.

Indeed, we have Ilx + yll < 13 < 15 = 6 + 9 < AT + v'9O = Ilxllllyll.

14 CHAPTER 1

8. We compute

AB~ r:1
1
-1 ~31
1 -1
Expanding by minors across the first row gives
and A+B~[:
0
2
1 n
det A 3:1 02 -1
1 I + 1111 ~ I = 6- 2= 4,
detE
11 1
0
~ 1- 11 ~ ~ 1 = -3,
5 -1
det(AE) 31-1 1 1_ 1151
1 -1 ~1 1- 31 1 1 1=-12 and

det(A + B) - 41 ~ ~ I = 8.
11. (a) For n = 2,
'xall
det('xA) = 'xa21
I
For n = 3,

'xall Aal2 'xa13

det( AA)
Aa21 'xa 22 'xa23
Aa31 'x a32 'xa33
'xall det('xAd - 'xa12 det('xA 2 ) + 'xa13 det(AA3)
,X . ,X2(a11 det Al - al2 det A2 + a1 3 det A3)
,X3 det A,

where AI , A2 and A3 are 2 x 2 matrices obtained by expanding across the first row .
Assume that for n = k, det('xA) = =
Ak det A. The for n k + 1, det(AA) can be found by
a process analogous to the 3 x 3 case:

det(AA) 'xall det('xAd - 'xa12 det('xA 2 ) + ... + (_l)k 'xal,k+l det('xAk+d


,Xk+l det A,

where AI, A 2 , ... , Ak+l are k x k matrices obtained by expanding across the first row .
By induction, det('xA) =
An det A for an n x n matrix A.
14. Assume, as in the book, that det(AE) = (det A)(det E). Then det(AEC) = det[(AB)C] =
det(AE) det C = (det A)(det E)(det C).

17. Multiply the two matrices to get the identity matrix:

a
[ e
b]
d
1
ad - be
[d-e -b]
a

Similarly, we can show that


TH E GEOMETRY OF EUCLIDEAN SPACE 15

SOLU TIONS TO SELECT ED R EVIEW EXERCISES FOR CHAPT ER 1

1. v + w = = =
(3,4, 5) + (I , -I , 1) (4, 3, 6) 4i + 3j + 6k; v + w is the di agonal of a parallelogram
whose sides are v and w.
= = =
3v 3(3,4 ,5) (9 , 12, 15) 9i + 12j + 15k; 3v has the same direction as v with length 3
times the length of v.
= = =
6v + 8w 6{3, 4, 5) + 8(1, -I , 1) (26, 16,38) 26i + 16j + 38k; 6v + 8w is the diagonal
of a parallelogram whose sides are 6v and 8w. 6v has the same direction as v with length 6
times th at of v and simi larly for 8w.
- 2v =
- 2(3 , 4, 5) =
(-6, - 8, -10) =
- 6i - 8j - 10k; - 2v is a vector in the opposite
direction of v with length twice that of v.

y
y

v ·w =
(3)(1) + (4)(-1) + (5)(1) = 4; v · w is the number II v llll w ll cos B, where B is the
angle between v and w .

v x w is perpendicular to both v and w . Its length is the area of the parallelogram spanned
by v and w .

4. (a) Using the point-direction form of the line, we get l(t ) = (0 , 1,0) + t (3, 0,1).
(b) Using t he point-point for m of the line, we get

l(t ) = a + t(b - a) = (0 , 1, 1) + t [(O, 1, 0 - 0, 1, 1)] = (0 , 1, 1) + t (O, 0, -1) .

(c) T he normal to the plane is (a, b, c) = (- 1,1, -I) , so th equation of the plane is

a(x - xo) + b( y - Yo) + c(z - zo ) = 0,

i.e.,

- 1( x - 1) + l(y - 1) - 1(z - 1) = ° or x - y+z = 1.

5. (b) v · w = (1)(3) + (2)(1) + (- l ){O) = 5.


i j k
~ ~1 = 1 ~ ~ ~ ~ ~ ~
1 1
6. (b) v x w = ; 1j -I 1j + 1 1k =i - 3j - 5k.

7. (b) We compute IIvll = VI + 4 + 1 = V6 and Il w ll = V9 + 1 + 0= v'lO. T hen the definition


of the dot product and the result of xercise 5(b) gives

cosO = Jlvvllll· ww il = Y6v'lO


5
=-5
- -5-
V60 2.Jl5 ·
16 (HAPTER 1

8. (b) The area of the parallelogram is the length of v x w. Using the result of exercise 6(b), we
=
get Ilv x wll VI + 9 + 25 v'35., =
12. We compute the following dot products using the fact that u, v and w are orthonormal:

a .u (au + f3v + , 'w) . u = a.


a· v (au + (3v + ,w) . v = ,B.
a·w (au + (3v + ,w) . w = ,.
/
I
v
Geometrically, a . u is the projection of a on u; similarly for
the others.

15. From the definition of the dot product and the fact that U · u = lIull 2 , we compute

v . a ~ lIall,b . a + IIblillall 2 _ b· a + IIblili a ll


cos ()
II v II II all 11vlliiall IIvll
v .b lIallllbW + IIblla . b lIalillbl,1+ b . a
cos () =
IIvllllbll IIvllllbll Ilvll
Since t he angle between a and v is the same as the angle between b and v, the vector v must
bisect the angle between a and b.

°
18. (a) Given that a · b = a' . b, this implies that a· b - a' . b = or (a - a') . b = 0 for all b.
Choose b = a - a' to get lIa - a'1I2 = 0, so a - a' = 0, which means that a = a'.
(b) The answer is yes . Ifax b = a' x b for all b, we can conclude that (a - a') x b = 0 for
all b. Choose b to be a unit vector orthogonal to a - a'. By definition of the cross product,
this implies lI a - a'il = 0, so a = a'.
22. (e) Note that the x and y coordinates lie in the third quadrant

of the xy plane. The definitions from section 1.4 are used: For

cylindrical coordinates, we compute

r JX2 + y2 =
v'12 + 4 = 4.

() 7r + tan- (y/x) = 7r + tan- 1(1/V3)

7r + 7r/6 = 77r/6.

Thus, the cylindrical coordinates are (4,77r/6,3). y


x
For spherical coordinates, we compute

p = J x 2 + y2 + z2 = V12 + 4 + 9 = 5.
4> = cos-1(z/p) = cos-1(4/5).
Thus, the spherical coordinates are (5, 77r /6, cos- 1(4/5)),

23 , (b) Using the formulas from section 1.4, we calculate


z
x = r cos () = (3)( V3/2)
y
and

y = rsin() = (3)(1/2).

Thus, the corresponding cartesian coordinates are

(3V3/2, 3/2, ~4).

For the spherical coordinates, we compute

THE GEOMETRY OF EUCLIDEAN SPACE 17

p= Jx2 + y2 + Z2 = / 27 +~+ 16 = 5.
V4 4

1
cjJ = cos- (z /p) = cos- 1(-4/ 5).

Hence, the corresponding spherical coordinates are (5,1r/ 6,cos- 1 (- 4/5)).

24. (b) Using th formu las from section 1.4, we compute:


z
x pcosBsincjJ = 2(0)( 1/ 2) = 0,
y psinB sin <fJ = 2(-1)(1/2) = - 1,
y
z Pcos <fJ = 2(..;3/ 2) = v'3.

T hus, the corre ponding oordinates are (0 , - 1, ..;3).

x
For the cylindrical coordinates, we calculate:

r = J x 2 + y2 = JO+l = 1.
We note that x = 0 and y = -1 , so B = 311'/2. Therefo re , the corresponding cylindrical
coordinates are (1, 3rr/2, V3).
28. Using the methods of sec ion 1.5, we get

AB = [ ~
0
0
0 n[~ n[! n 0

0
1
=
0
0
0

nr: n
and

[~ ;][l
0 0 0
BA = 1 0 0
0 0 0

Clearly, AB :j: B A.

33. (a) r is t he vector 7i + 2j, so W = F · r = 70 co B + 20 sinB .


(b) If F has a m agnitude of 6, then F = 6 cos Bi + 6sinBj. Since B = 11'/6, we have F =
6( J3/2)i + 6(1/2)j , and so W = F . r = (21 V3 + 6) foot-lb.
36. Su btract the fir t row from the second and third rows , t hen expand by minors along the first
column to get
1 X x2 1 x x2 2
1 Y y2 0 y- x y2 _ x2 y2 _ x
=I y - X z2 I
z - x _ x2
1 Z Z2 0 z- x z2 _ x 2
(y - x)(z2 - x2) - (z - X)(y 2 - x 2) = (y - x)(z - x )( z - y) :j: 0
if X , y, z are all differen t. The last step used the fact that (z2 - x 2) = (z - x)(z + x ).
=
39. (a) We recogniz a· (b x c) as a t riple product . Let a (a1 ' a2 , a3 ) and use a similar notation
for band c. T herefore,
1 a t a2 a3
V = 6" b1 b2 b3
Cl C2 Cs

(b) Use t he formula from part (a) and subtract the first row from th second row;

1
1 1 1
- 1 -1 1 1 1 1 1 110 - 2 1 1
6 1 1 0
6 ~ ~2 ~ = 6" 1 1 = 3'

T hus , the volume is 1/3.

18 CHAPTER 1

44 . A vector which is normal to t.he first plane is v = 8i + j + k. A vector which is normai to the
second plane is w = i - j - k. T he cross product v x w is orthogonal to both normal vectors,
so it should be parallel to both planes. We compute v x w = 2i - 7j - 9k , so the unit vector
in question is (2i - 7j - 9k)/v'f34.

47. We want v = ai + .Bj + ,k to be such that !lvll = 1. From the definition of the dot product,
we know that
v'3
= 2v'3,1 + /3.'J +,k .
o v ·j
cos 30 = 2 = Ilvllllill' so v

Since v makes equal angles wi h j and k , we must have .B = , . Since Ilvll == 1, we determine
that .B = 1 = 1/20. T herefore,

TEST F OR CHAPTER 1 (The answers are at the back of this book .)

1. True or false . If false, explain why.

(a) Given vectors u= i + j in IR 2, v = i + j +k in IR3 and w = 2i -k in IR 3, we have u·v = 2


and v· w = 1.
(b) Gi ven two m atrices A and B, det(AB) is defined if and only if det(BA ) is defined.
(c) For non-zero vectors a and b, the set a , b , and a x b, in that order, always forms a
right-handed system.
(d) Suppose a , b, and c are non-zero vectors in IR3 and a· b = a · c = 0. Then either b is a
multiple of c or b and c are parallel vectors.
(e) A cross product is 0 only if one of the vectors is O.

2. Find an equation for the plane passing through the point (0,0,1) and parallel to the line
contain ing t he points (1, 0, 2) and (-1,2, 1).

3. Let A = (2 , 1,3) , B = (- 3,2, 4) and C = (1 , 2, 2) be the vertices of a triangle. Let a be the


vector from A to B and let b be t he vector from B to C.

(a) Calculate a and b .


(b) Describe the triangle ~ ABC using vectors of the form sa + tb . (Find restrictions on the
parameters sand t.)
(c) Calculate the area of ~ABC .

4. Let u = 3i + OJ + k and v = i - 2j + k have a common tail . Let () be the acute angle between
u and v. Calculate sin ().

5. A parallelpiped spanned by the vectors (3, - 2,1), (0 ,2 ,-1) and (- 1, 0, 1) is filled with sand.
The sand will fill another parallel piped spanned by the vectors - i, 2j and zk. Compute all
possible values of z.

6. (a) In ]R4, fi nd an equation for the line passing through A = (1,3, - 2, 0) and B == (0,1, -1,1).
(b) Show that the line in part (a) is not orthogonal to the line l(r) = (1 , 3, -2 , 0) +r(2, 0, 3, 1).

7. Let

(a) Compute AB.

,. "
THE GEOMETRY OF EUCLID EAN SPACE 19

(b) Compute det(AB) and interpret geometrically.


(c) If det(BA ) exists, compute it and interpret it geometrically.
8. Convert the spherical coordinates (- 2, -11"/4, 27r/3) to

(a) Cylindrical coordinates.


(b) Rectangular coordinates.

9. Let u = (2, -1, 0,1) and v = (-1, - 2, 1,2).


(a) Verify the Cauchy-Schwarz inequality for the given vectors.
(b) Calculate the projection of v onto u.

10. A unique house has an inclined floor where the owner


keeps his dog . The dog 's play area is a fenced en­ z
closure with vertices at A = (2,4,1), B = (1, 2, 0) ,
C = (0,0, -1), D = (1, -1, -1) and E = (3,0,0), as A
shown in the diagram.
y
(a) All of the vertices lie on a plane. Find the equation

of the plane.

x D
(b) Find the area of the dog's play area.

(c) An object is located at (1, 1, 3). W hat is the min­

imum distance the dog must leap off the floor to

reach the object?

21

- - - - - - - - - - - - - - -- - - -- - - -- - -- --

2 DIFFERENTIATIO N

2.1: THE GEOMETRY OF REAL-VALUED F UNCTIONS


GOALS

1. Be able to define a graph, a level cur ve, a level set, and a section.

2. Be able to graph a function f : ]R 2 --+ ]R .

ST UDY HINTS

1. Notation. f : A C ]Rn --+ ]Rm describes a function f The domain is A, which is a subset of ]Rn.
Points in A a re mapped to points in the range, which is a subset of ]R m . The only restriction
on nand m is that they have to be positive integers.

2. Real-valued functio n . In the not ation, f : A C ]R n --+ ]R m, we restrict m to be 1 in this section.


A real-valued function assigns a unique real number to each point in A.

3. Graphs. T he graph of f : A C ]Rn --+ ]R is dr awn in the sp ace ]Rn+l. If x is a point of A, the
the graph consists of all points in ]Rn+l wi th the form (x,f(x)), where f (x) is a real number.

4. Level set. This is the set of x such th at f(x) is a constant. In ]R2, such a set is called a level
curve and in ]R3, it is called a level surface. Level sets are important for graphing.

5. Sections. These are intersections of graphs with a vertical plane. Usually, the most helpful
= =
sections for graphs in ]R3 are he intersections with the planes x constant and y consta.nt.

6. Graphing in JR3. Often, the best way to d raw a graph in JR3 is to draw level curves for
=
z constant. Then lift or drop the curves to the appropri ate "height" for z =
constant.
Analyzing the sections helps complete the graph. It is a good idea to review how to sketch
the graph of an ellipse, a hyperbola, a circle and a parab ola from your calculus or precalculus
text.

7. Sketching planes. Many of us are poor artists, and as a result, three-dimensional geometry
may be frustrating due to this problem rather than a lack of mathematical understanding.
Planes are sometimes easily sketched by plotting three non-collinear points (usually on the
coordinate axes) and then passing a plane through them.

8. Spheres. The equation (x-a )2+(y-b)2+(z-c)2 = r2 represents a sphere ofradius reentered


at (a, b, c) .

9. Cylin ders . A surface in ]R3 is called a cylinder if x, yor z is missing from the equation . A
cylinder can be sketched by drawing the level curve in the plane where the missing variable is
zero. Then move the curve along the axis of the missing varia ble.

10. Graphs in ]R4. Example 5 gives a fu nction whose graph cannot be drawn on paper. To see the
"graph," one can make a movie which shows the concentric spheres expanding.
22 CHAPTE R 2

SOLUTIONS T O SELECTED EXERC ISES

1. (a) To determine the level curves, we look at t he equation c = x - y + 2, where c is a constant .


T he equation is the same as y = x + (2 - c), which is the equation of a line with slope 1 and
y intercept 2 - c.
The graph of f is a plane with intercepts at (- 2, 0, 0), (0 ,2, 0) and (0 , 0,2).

2. (b) We look at c = =
1 - x 2 - y 2 for c constant . This rearran ges to x 2 + y 2 = 1 - c, which
is the equation for a circle of radius v'1='C, centered at t he origin if c < 1. If c = 1, then the
level set is a point at the origin. Th re is no level curve if c > 1.

3. (3) Substitute x = rcos (} and y = rsinB to get

Since z = 1'2 does not depend on () , the sh ape of the graph does not depend on () .

5. For constant c, the equation c = y'10 0 - x 2 - y2 is equivalent to c2 = 100 - x 2 - y2 or


x 2 + y2 = 100 - c2 . T h I vel curves are circles with radii VI00 - c2 , centered at the origin.
So, for c = 0, 2,4,6 8, 10 the radii are 10, v'96, j84, 8, 6 and 0, respectively. Dr awing the
level curves and raising them to t he appropriate z values, we obtain the following graph. T he
graph of f (x , y) is a hemisphere. T he level curves and the graph a re shown here.

y ,

10. The level curves have the equation c = x / y or y = x / c for onstant c. Th se level curves are
lines thro ugh the origin with slope l /e. One restriction is that y f:. 0. Next we discuss the
graph z = =
x/ yo Noti e t hat when x is held constant, the sect ion are the hyperbolas c yz ,
and when y i held constant , we get the lines c = x / z . Putting all this information together,
we get t. he "twisted plane" as t he graph.
DIFFERENTIATIOI\l 23

12. The level surfaces have the equation c == 4:z;2 + y2 + 9z 2. T here is no level surface if c < O. If
c = 0, then t he level surface is the origin. If c > 0, then we should look at the level curves
for constant k, i.e., analyze c - 9k 2 = 4x 2 + y2 . We recognize that if 9k 2 < c, then the
level curves are ellipses which get smaller as Ikl approaches v'C/ 3. Similarly, we see that level
sections parallel to the yz plane have the equation c - 4k 2 = y2 + 9z 2 , which are ellipses with
decreasing "radii" as Ikl approaches .;c/2. Also , the level sect ions parallel to the xz plane have
the equation c - k 2 = 4x 2 + 9z 2 , which are again ellipses which get smaller as Ikl approaches
.;c. The level surfaces are ellipsoids if c is positive.

17. If c < 0, the level curve is empty. If c = 0, t he level curve is the x-axis. If c > 0, it is the pair
of parallel lines Iyl = c; that is, the lines y = c and y = -c. In the yz plane, we sketch the
graph of z = Iyl. Since x does not appear in the equation , we get a "cy,)inder" and this sketch
is shifted along the x axis to obtain the graph in 1R 3 .

22. The equation can be written as (x 2 - 2x + 1) + y2 = 1 or (x - 1)2 + y'2 = 1. In the xy plane,


this is a circle with r adius 1, centered at (1, 0). Since z is not in the equation, z may take on
any value, so the circle is sh ifted up and down the z axis.
24 CH APTER 2

- .---:;,.-<-+-- (1 ,0,0)

x
, /

25 . First, we sketch the graph of z ::: x 2 in the xz plane. T hen shift the graph along the y axis to
get a parabolic ylinder.

I I
/ /
/ /

y
x

29 . An equi alent equation is 4x 2 + 2z2 ::: 3y2. When y ::: 0, t h level curve is the origin. When
y :j:. 0, we have level sections which are ellipses centered around the y axis. T he major axes
are parallel to the z axis and the minor axes are parallel to the x axis. T he ellipses get larger
as IYI increases. To complete the graph, we note that when x ::: 0, the section is the straight
lines z ::: ±y'372y. T hus our graph is two cones.

32. Subs itute x ::: rcosB and y ::: rsinB. Therefore, if x 2 + y2 ::: r2 :j:. 0, then z ::: f (x, y) :::
2x y/ (x 2 + y2 )::: 2(r cos 0)( l ' sin B) / r 2 ::: r2 (2 sin 0 cos 8)j r2 ::: sin 20. Thus, t he function reduces
to z ::: sin 2B if r' :j:. O. If - 1 ~ z ~ 1, the level curve is one or two straight lines through the
or igin satisfying z ::: sinO (see sketch below at left). The level curves are raised to a height
z ::: sin 2B to obtain the graph of a "wrinkled plane." (See graph below at right. The dot ted
line is a portion of the xy plane.) If z > 1 or z < - 1, t here is no level curve. Notice that the
plane becomes flatter as I' gets larger.
DIF FERENTIATION 25

2.2: LIMITS AND CONTINU ITY

GOALS

1. Be able to defi ne the following: open disk, open set , neighborhood, boundary point , limit ,
continuous.

2. Be able to determine where a function is continuous.

3. Given a function, be able to compute a limi t or show that a li mit does not exist.

STUDY HINTS

1. Theoretical section . This section is not essential for computation al purposes. Your instructor
may choose not to emphasize the material in this section . Use your lectures to determine how
important the material is for your course.

2. Definitions , (a) An open disk around Xo is the se of points x such that Il x - xoll < 1' . It is
denoted Dr (xo) . Note the strict inequality.
(b) An open set U is a set such th at every xo has an open disk entirely within U. You will need
to find an r when proving that a set is open.
(c) A neighborhood of Xo is an open set containing Xo.
(d) A boundary point of a set A has no neighborhood en irely inside or entirely outside of A.

3. Review. You should review the concepts of limit and continuity from your one-variable calculus
textbook before continuing.

4. Limits. In the defin ition, be aware that Xo doesn't have to be in A ; Xo may be on the boundary.
Also, f (xo) does not have to be defined. We are only interested in the points x n ar Xo . For
proofs, we need to choose U, which is dependent upon N.

5. Properties of limits. Most of th se are what you would intuitively xpect. Note hat for
multiplication and division, the m apping is into }R I.

6. Conti nuity. Analogous to the one-variable definition, a functi on is continuous at Xo if

lim f(x ) = f( xo),


X~ X o

I.e. , the limit equ als the function value. T he limi t on the left-hand side is concerned about
points near Xo. The right-hand side, f(xo), is concerned about the point Xo itself.
26 CHAPTER 2

7. Non-existent limits. Showing that the limit of f(x, y) does not exist is sometimes simple. To
show a limit does not exist , one can, for example, look at the limit of f by first holding x
constant, then holding y constant. If the two values differ, the limit does not exist.

SOLUTIONS TO SELECTED EXERCISES

2. Take °< r ~ y, th n for all points (x , y), the open disk Dr (x,y) C B. Thus, B is open.
5. (c) Recall the defin ition of the derivative:

f ' (X o ) -- I'1m f(x o + h)h - f (xo) .


h -tO

By letting f(x ) = e"" and xo = 0, we get

lim - h-
e" - 1 = lim eO+ h - eO
= _ eX
d I = 1.
h -t O "-to h dx x =o

6. (c) From one-variable calcul us recall that lirn.,-t 0 (sin :z: / x ) = 1. This fact an al 0 be verified
by using I'Hopital's rule . Using this fact and the properties of limits, we compute

.2
lim sm 2 x -_ I'1m
(.)2 ( sm.)2

Sill X x _ 12 -
_
-
}'
1m - -
1
.
x-tO x x -t O X x-t O X

g. (c) It is obvious that the limit of the numerator is 0, and the limit of the denominator is 2 # 0,
so t he limit of he quotient is 0/ 2 = O.

10. (b) First, hold y = 0 constant and let x approach O. Then use I'Hopital's ru le:

. cosx - 1 - (:z:2 / 2) . -sinx - x . - cosx - 1


hm
x-tO X4
= 11m
""-to 4x 3
= x-t
hm
O 12:z:2
.

This last limit tends to -00 since the numerator tends to - 2 and the denominator tends to 0,
Thus, the limit does not exist.

11, (a) Let t = xy and use continuity of compositions (theorem 5) to get


· sin xy
- - = 1m - -
}' sin t = 1,
I1m
(x ,y ) -t( O, O) xy t -tO t

13. (c) Use the fact that the limit of a vector is the limit of each component (theorem 3(v)). So
we get limx -t l (x 2 ,e X ) = (l,e).

16. (a) We will use theorem 5 (continuity of compositions). Note that f (:z: ) = (1 - x)8 + cos (1 + x3 )
is the sum of two functions . The first is u 8 with 'U = 1 - x. Since tJ is continuous, theorem 5
says that (1 - x)8 is continuous. The second function is cos v with v = 1 + x 3, Again, since v
is continuous, theorem 5 says that cos (l + x 3 ) is continuous. T he sum of continuous functions
is continuous, so f(x ) is continuous.

17. (a) We can make a function continuous by equating f (xo ) and limx-t xo f(x) . As in exercise
11(a) , we can Ie t x + y , so
=
· sin(:z: + y) _ l' sin t - 1
11m - Im-- - .
( x ,y )-t(o,O) x +Y t-t O t

Thus, we let

sin (x + y)

-~----''-'-
x +y
1 for x = = y = 0,

"
DIFFERENTIATION 27

and we have a continuous function.


(b) First , we note that if x = y , then
xy . x'.l 1
lim ')
( x . y ) ~ (o. O ) x- + y-" = lim 2x-., = -2 '
x~o

On the other hand , if x = -y, then


xy . _x 2 1
lim
(x.y) ~ (o.O ) x 2 + y2
= xhm - - = -­
~o 2x 2 2
Since the Hmiting value depends on the direction of approach, the limit does not exist at the
origin , so it is not possible to make the function continuous at the origin.
22. (b) We want to find a 6 for every N > 0 such that 0 < x < 6 implies that 1/1xl > N.
Let 0 < 6 < l i N , then Ixl < liN implies that 1/1xl > N. T his is not true if t he absolute
values are omitted , i.e., limx~o(llx) may be +ex> or -ex> depending on which side of 0 we are
approaching from (see the figure below.)

x
x

y= l/lxl y = l/x

27. (a) By the triangle inequality, 1a. 3 + 3a 2 + al < la 2 1+ 31a 2 1+ lal, since lal < 1 (we assume it is
small, since 6 ,is small), this is less than (or equal to) 514 Choose 6 < 1/500 , then for lal < 6,
la 3 + 3a 2 + al ::; 1/100 (note that this is a very rough estimate; a bigger 6 would probably work
if we work harder to improve the inequality.)

2.3: DIFFE RENTIATION


GOALS
1. Be able to state the definition of partial derivat ives.
2. Be able to compute a partial derivative or a matrix of partial derivatives.
3. Be able to compute a gradient .
4. Be able to compute a tangent plane.

STUDY HIN TS
1. Nota-tion. Class en means that the nth derivative is continuous.
2. Partial derivatives . Know the definition

- ' f( Xl, "" Xi + h, ... , x n) -


of = l In1 f(Xl' ... , Xi, ... , Xn)

=---..:........:....:.---''---'---'----'-~~-'--'----=-'--'--'-'--'----'-'-'-

OXi h~ O h
To compute ofI OXi in examples, consider all variables except Xi to be constant and differentiate
by one-variable methods . Differen iation is performed with respect to the variable Xi.
28 CHAPTER 2

3. No tation lor partial deriv atives. In many texts, Ix is used for 8f/8x. If we wish to evaluate
at a given point, we write

81
ax 1(xo,Yo)
'

fx l(xo ,yo) ' or ~: I(:Co ,Yo) if z = f (x , y).

4. Tangent plane. The tangent plane to the graph of a function f (x , y) at (x o, Yo, f(xo, Yo) ) is
given by
z= f(x o,yo)+ 8f
ax l 81 1
(x -xo) + {) (Y -Yo).
(Xo,Yo ) y (Xo ,Yo)
T his equation is also used to compute linear approxi m ation . Compare this equation to the
equation of a tangent line and the linear approximation in the one-variable case. See section
2.6 for a gener lization.

5. Differ ntiability. Equation (2) in the text tells you that f : R 2 -+ is differentiable if t he
tangent plane approaches f (xo,yo ) as (x,y) approaches (x o,yo ). Now, if f : U C ]Rn -+ ]Rffl,
then
· Il f(x ) - I(xo ) - T(x - x o) 11 0
I1m - (2)
X--+Xo Il x - xoll - ,
where T : ]Rn -+ ]Rm is the derivative. You should be able to get equat ion (2) from this
definition . T his definition of differe ntiab.ility is most important for theortical work .

6. Gradient. T he gradient is a vector whose components are the partial derivatives of f , with
8f/8x; in the ,t h position. Here, fis a real-valued function. Thi operation is denoted by the
sy mbol 'V . Someti, es , it is denoted "grad." For a function I : ]R3 -+ JR. , you should remember
the form ula

7. Derivative of vector-valued fu.nctions. Con ider a fu nction I : R n -+ ]R m. T he derivative is an


m x n matrix of partial derivativ s. The range consists of vectors with m components. Think
of t he components as real-valued vector fu nctions; th n each row of the derivative matrix is a
gradient. The derivative m atrix of f, evaluated at Xo, is denoted Df(xo) .

8. Important facts . Diffe rentiability implies continuity of the fun ct.ion, but continuity does not
imply differen tiability. The existence of continuous partial derivatives im plies differentiability
but the converse is not true. If a funct ion is differentiable, then its partial derivatives exist ,
but the converse is, again , not true.

SOLUTIONS TO SELECTED EXERCISES


1. (b) Holding y constant and differentiating with respect to x, we get 81/8x = ye xy . By
symmetry, 8 f / 8y = xe XY • In this problem, all we did to compute fJ!I fJy was to switch x and
y. T his is what we mean by "symmetry" ; it only works for functions that are unchanged when
x and yare swapped.

2. (b) Hold y con taut and use the chain rule to differentiate wi th respect to x. We get
f}z 1 1 1 y . '1 1 8z x
-- .- 'y - . ar y, 8y
ox - J1+ xy 2y'1 + xy - 2{1 + xy) ' SImI
2(1 + xy) .
At (1,2), 8z/8x = 1/3 and 8z/ oy = 1/6. At (0,0), az/ax = az/ 8y = o.
3. (b) Holding y constant and using the quotient rule, we calculate:

8w _ 2x (x 2 - y2 ) - 2x (x + 2 + y2)
8x (x 2 _ y2 )2
DIFFERENTIATION 29

Holding x constant and differentiating with the quotient rule , we get:

ow _ 2y(x 2 - y2) - (-2y)(x + 2 + y2) _ 4x 2y


ay (x 2 - y2)2 - (x - y2)2 .
2

4. (b) We must show that the partials are continuous in the domain : of/ax = l/y - y/x 2 =
(x 2 - y2)/ x 2y, which is continuous for x f. 0 and y f. OJ a f/ ay = _X/y2 + l/x :;;:: (y2 - x 2 )/xy2,
which is continuous for x f. O. T hus, f(x) is C l since its partial derivatives are continuous.

6. (b) The equation of the tangent plane is given by

Z = Zo + [fx(xo, yo)](x - xo) + [fy(x o, yo)](y - Yo).

Using .the result of exercise l(b) , we compute:

afl - l' afl =0', f( 0,1 ) = 1.


ax (0 ,1) - , oy (0,1)
Therefore, the tangent plane is z = 1 + l(x - 0) - O(y - 1) or z - 1 + x.

7. (b) The first row contains the partial derivatives of xe Y + cosy. The second roW contains
those of x, and the third row con tains those of x + eY • The fi rst column contains the partial
derivatives with resp ect to x, and t he second colu mn contains those wi th respect to y. Thus,
the matrix of partial derivatives is

eY xeY - sin y
1 0
1.
[ 1 eY

8. (b) The function f is a m apping from JR3 to JR 2, so the matrix of the partials is 2 x 3. Let
=
It (x, y, z) x - y, the first component of f. Similarly, let Iz(x, y, z) y + z. Then =
af l
Df(x, y , z) =[ a;: ay %1 ] _ [1 -1 ()]
aiz alz olz - 0 1 1 .
ax ay az

11. Using t.he result of exercise 1(b), x(of/ax ) = xye xy = y(af/ay) .


12. (b) Use the linear approximation form ula, which is the same as the equation of the tangent
plane: z = Zo + [fx(xo, yo)](x - xo) + [f y(xo , yo)](y - Yo). Let z = f(x, y) = x 3 + y3 - 6xy,
IO = 1, I = 0.99, Yo = 2 and y = 2. 01. We com pute:

8flax = 3x 2 - 6y, so fx(1 ,2) = -9.

a f /ay=3 y2_6x, so / y(1, 2) =6.


Therefore, our linear approxim ation is z ~ - 3 + (-9)( -0.01) + 6(0.01) = -2.85. The actual
value is -2.8485 .

13. (c) The gradient is defined as the vector (of/ox , of/oy , a f/az) . Thus,

14. (c) The tangent plane is defined by \If(xo) . (x - xo) = O. Exercise 13 (c) gives us

\l f (1, 0,1) = ei + 2ek,

so the tangent plane is e(x - 1) + 2e (z - 1) = 0 or x + 2z = 3.

30 CHAPTER 2

17. We compute 'V/ (0,0 , 1) = (2:1:, 2y , - 2z )1(0 ,0,1) = (0 , 0, - 2) = - 2k.


20. We want to find T in equation (4). By linearity, / (x) - f( x o) = f{ x - xo ). Denoting x - Xo
by h , we want to find T so that

I ·
hl:To
II/ (h) - Thll -
Ilhll -.
°
If we choose T = f, the numerator vanishes for all h, so this T sati ties the condition; that
is, t he deriv tive of a linear map is the map itself. For exam ple, in one variable, consider
f (x) = ax . From one-variable calculus, T = f ' (x o) = a fo r all Xo.

2.4: INTROD U CTIO N T O P ATHS AND CURVES


G OALS

1. Given a path, be abl to compute t he velocity vector.

2. Be able to find a tangent line for a given path.

ST U DY HIN TS

1. Paths . A path is a "formula" that describes a curve in space . The picture of the pat h, which
we can draw on paper, is called the image of the path or the curve of the pa th .

2. Path images. Often, it is convenient to express a path in terms of x and y when you want
to know the image of a path. This is done by elim inating the parameter. For example
(x, y) = (t2,t4) means t =..;x,
so y = t 4 = (-J x)4 = x 2. Caution: In this example, x = t 2, so
x is always non-negative.

3. Circular functions. If a path is parametr ized by an expression involving cos t nd sin t , the
parameter can usually be eli minated by squaring and adding. Use the identity cos 2 t +sin 2 t = 1
and 0 her trigonometric identities.

4. Velocity. T he velocity vector 's components are first deri vat ives of the components of the path.
The velocity vector is tangent to the path.

5. Tangent lines. It is easy to find a tangent line if you rem ember that a line can be described
as x + tv. The vector x is chosen to be a point on the path at to and v is the velocity vector
c' (t o). Thus, the tangent line to a path i

l(t ) = c(to ) + (t - to )c'(to).

SOLUTIONS TO SELECT ED EXERC ISES

1. From y = 4 cost , we get y/4 = cost. Use the fact that

and substitution to obtain

1 x

Since 0 ~ t ::; 211", th curve is an ellipse with y intercepts at -4


± 4 and x intercepts at ± l .
DIF FE RENTIATION 31

3. As in example 1, c(t) has the for m (xo, Yo , zo) + tv,

where (xo, Yo , zo) = (- 1,2,0) and v = (2, 1,1). T hus,


z
c(t) is a line in R3. Specifically, it is the line t hrough
(- 1, 2,0) with direction (2, 1,1 ). 2
(-1,2,0)

7. A path's velocity vector is found by differentiating the individual components. In this case,

d 2 d 3 d )
1" ( t) ( dt (cos t ), dt (3t - t ), dt (t)
(- 2costsint,3 - 3t 2 , 1).

12. T he tangent vector to a curve c(t) = (x(t), y(t» is c' (t ) = (x'(t),lI(t». In this case, the
tangent vector is (2t, 0).

15. T he equation for the tangent line is l(t) = c(to) + (t - to) c'(t o). Here , to = 1 and c'(t) =
(3 cos 3t, - 3 sin 3t, 5t 3 / 2 ). T herefore , the tangent line is

l(t) = c(l) + (t - l) c'(I) (sin 3, co 3, 2) + (t - 1)(3 cos 3, -3 sin 3, 5)


(sin 3 - 3 cos 3, cos 3 + 3 sin 3, -3) + t(3 cos 3, - 3 sin 3, 5).

17. F irst , we need to find the tangent lin at to. We compute c (2) = (4,0 ,0) and c'(t)
=
(2t , 3t2 - 4,0) , so c' (2) (4,8, 0). T hus, the tangent line is l(t) =
(4,0,0) + (t - 2)(4 ,8, 0).
T he position of the particle a t t l = 3 is 1(3) = (4,0,0) + (3 - 2)(4 ,8, 0) = (8 , 8, 0).

2.5: PRO PERTIES OF THE DERIVATIVE

GOALS

1. Be able to state the ch ain rule.

2. Be able to compute a partial derivative by using the chain rule .

STUDY HINT S

1. Chain rule. Suppose f is a funct ion of Yl, Y2, ... , Yn and each Yi is a function of x. Then

df
- =of
- .dYl
- +of dY2 '
- . -+ ... +of- . ­dYn
dx aYI dx aY2 dx aYn dx '

Notice how each term appears to be dJl dx wit h ay,


a.nd dYi "cane ling." However, bewar
that the "su m" on t he right-hand side is df / dx, not n times df / dx. Also note the different d's:
"a" is for a fun ction of many variables, while "d' is for a function of one variable.
2. Multivariable chain rule. T he multivar iable chain rule st ates th at

D(f 0 g)(xo ) =::; Df(yo) Dg (x o) ,

where Yo = g(xo) . T his is he product of two deriv tive m atrices , so any desired partial m y
be obtained by multi plica ion .
32 CHAPTER 2

3. Chain rule, gradient relationship. Know that if fis r ai-valued and h (t ) == f( c (t)) , then

dh

dt = \7f(c(t)) . c' (t ).

SOLUTIONS TO SELECTED EXERCISES


2. (b) The function f is differentiable by the sum rule. Its derivative is
af af]
[ az' ay = [1 , 1] .

(f) The function is differentiable by the chain rule. We know that z2 and y2 are differentiable
by he product rule and that 1- Z2 - y2 is differentiable by the sum rule. Also, the square root
function is differentiable (where its argument is positive), so th entire function is differentiable .
Its derivative is

3. (b) This is a spe ial case of the first special case of the chain rule:
dh af dx af du af dv
dx = ax· dz + au . dx + av . dx
af al du af dv
az + au . dx + av . dx·

5. (b) First, we compute I(c (t)) = exp(3t 2 • t 3 ) = exp(3t 5 ), so


f' (t) = 15t4 exp(3t 5 ).
Next, by the chai n ruie,

a 1 dx
-dl
dt
= - . - + -aayf .-dx
ax dt
dy
= ye '" Y . 6t + :te'" Y • 3t 2 .

Substitute x = 3t 2 and y = t 3 to get


dl j dt = t 3 exp (3t 5 ) • 6t + 3t 2 exp(3t 5 ) · 3t 2
= 15t4 exp(3t6 ),

which is the same as we got from a direct computation.


6. (b) Take th derivatives of each component to get c'(t) = (6t,3t 2 ) .
9. Substitute u = e"'- Y and v =x - y to get

By the chain rule,


D(f 0 9)( X, y) = D/(u, v )D9(X, y).
First, we calculat

Df(u, v) = [ ~~~~~: ~~~~~~] = [ sec\uu- 1) =;:] .


When (x, y) = (1 , I ), we have g(l , 1) = (e 1 - 1 , 1 - 1) = (1, 0) . Hence,
D/(I, O) = [12 -1° ].
DIFFERENTIATION 33

Next , we calculate
1
so Dg(I, 1) =[ 1
-1 ]
-1 .

Therefore
D(f 0 g)(I, 1) == [~ ~1] [~ =~] = [~ ~2]'
Alternatively, we may calcul ate D (f 0 g)(x, y) directly from (f 0 g)(x, V) .
13. (a) By the chain rule,
dT = V'T(c(t)) . c'(t),
dt

where c( t) = (cos t, sin t). Differentiate:

Substituting x = cos t, Y = sin t gives

V'T(c(t)) (2 cos te sin t - sin 3 t, cos 2 te sin


t - 3 cos t sin 2 t),

c' (t) ( - sin t, cos t) .

Thus,

-dT
dt
= -2sintcoste Sm. / +sin 4 t+cos 3 te smt
' 2 ')

-3cos tsin~t .

(b) Plug in x = cos t, Y = sin t into the expression for T and get

T(t) = cos 2 te sin


t - cost sin 3 t.

Using techniques from one-variable calculus, we have

-dT
~
. t cos team
= -2 S1l1 . t + cos 3 teS int + · 4
S1l1 t - 3 cos 2 ' ?
t S1l1~ t,

which is the same as the answer obtained by the chain rule in part (a).
17. (a) If y(x) and G are differentiable, then by the chain rule, and the fact that G(x, y(x)) is
constant,
dC = aC . dx + aC . dy = aC + oG . dy = 0
dx ax dx ay dx ox oy dx .
Solve for dy/dx: If oC/ay ¥ 0, then
dy oC/ox
dx - oC/oy'

(b) As in part (a), we differentiate C I and G 2 by the ch ain r ule:

dCI = aC I + aC I . ciYI + oC I . dY2 =0 and dC 2 = aC 2 + aC 2 . dYI + oC 2 . dY2 = O.

dx ox aYI dx 8Y2 dx dx ox OYI dx oY2 dx

Assuming YI (x), Y2 (x) and G are differentiable and

aCt/aYI aC I /aY2 11=0 for all x ,


I oCdOYI 8C2/ aY2
then we can solve for dyt/dx and dY2/dx. Rewrite the two equations as

oC I dYI oC I

- . - + oG
_I .dY2
- (1)
oYI dx oY2 dx ox
oG 2 dYI oG 2 dY2 OC 2
-. - +_.- (2)
OYI dx aY2 dx ax
34

Mul tiply (1) by OG 2 / OYl and (2) by - 8GI/OYl . Add t h two together to get :
8G 2 8G 2 8G l
-8G -1 . - + _ .­
dY2 8x 8Yl 8x 8Yl
l ---;<8~
dx = - 8" G=- G;:-=2~-;;
8-;:
G;-2 ----;8"G=:-l"­
8Y2 . 8Yl - 8Y2 . 8Yl
Sim ilarly,

8G 2 -8G 2 . ­ 8G l

- _8G l ._+
dYl 8x 8Y2 8x OY2
d;' - 8G l OG2 8G 2 8G l .
-._ - -.­
8Yl 8Y2 8Yl 8Y2
18. Begin with (x, y, z) = 0. L t x = I (y, z ), y = g( x , z ) and z = h(x , y); this means'
°
F(f(y, z), y, z) = 0, F (x, g(x, z ), z ) = and F (x y, h(x , y)) = 0. Differentiating F(f(y, z) ,
with respect to y and z , we get

and
8x
F~ 8z + Fz = 0.
Similarly, differentiating F(x , g(x , z ),z) and F (x,y, h(x , y)) with resp ct to x and z and y
z, respectively, we get
8y
Fx + Fy 8x = 0,
8y
Fy 8z + Fz 0,

and
8z
Fx + Fz 8x = 0,
8z
Fy + Fz 8y = 0.

Solving (2) for 8x/ 8z, (3) for By/ ox and (6) for oz/ 8y gives
ox
8z
assuming that none of the partials Fx , Fy and Fz are O. Multiply and get -l.
20. (a) Use the definition of the partial derivative:

h(0) 2 _ 0
81 (0, 0) = lim 1(0 + h, 0) - 1(0 , 0) = lim h 2 + 02 = lim Q= O.
8x h -t O h h-t O h h-tO h
T he last step holds since O/ h = ° for all h =F O. Similarly,

O(h)2 _ °
h -tO
°
81 (0,0) = lim 1(0 , + h) - 1(0 ,0) = lim 02 + h2
8y h h -t O h
= lim Q= O.
h -tO h

Therefore, 01/8x and 01/ 8y exist at (0 , 0) and equal O.


(b) If g (t) = (at, btl, then

(at )(bt)2 ab 2t 3 ab 2
(f 0 g )(t) = (at )2 + (bt )2 = (a2 + b2)t2 = a2 + b2t,
DIFFERENTIATION 35

so
, ab 2
(J 0 g) (t) = a 2 + b2 .
On the other hand, from part (a) , we have "V f(O, 0) =
(of /ax, of/o y)(O , 0) = (0 , 0). Aliso, we
compute g'(t) = (a, b), so "V f (O, 0) . g'(O) = (0,0) . (a, b) =
O. This verifies, as required, that
the chain rule does no t a pply to this function.

24. The t.ermow/ax on the left-hand side means the partial derivative of w(x , y,g(x , y)) with
respect to x, holding y constant. T he term ow/ox on the right-hand side means the partial
derivative of w(x, y , z ) with respect to x holding yand z constant . These two terms are not
equal because different independent variables are held constant. Thus, the reasoni ng is indeed
flawed .

2.6: GRADIENTS A ND D IRE C TIO N A L D ERIVAT IVES


GOALS
1. Be able to define a directional derivative.
2. Be able to compute a directional derivat ive.
3. Be able to explain the significance of the gradient.
4. Be able to understand the relationships among the direct ional derivatives, the gradient , the
tangent plane and level sets.

STUDY HINTS
1. Important example. Many ex m ples in this book use the fact that "V1' = r/1', where r =
(x , y , z )
and r = Ilrll = Jx 2 + y 2 + z2. T his is derived in example 1. Much time can be saved by
remembering this result .

2. Definition. The directional derivative is defined to be

+ hv) -
! f (x + tv) It=o or
. f(x
I 1m
11-+0
h
f(x)
.

The directional derivative gives a rate of change of f in the direction of v at x .

3. Geometric interpretation. Suppose v = ai + bj is a unit vector , (xo , YO) is a given point and
z := f(x , y) is a urface. T he d irectional derivative tells us the "slope" of a curve at (xo , Yo)
in the direction of v . T he curve is formed by the intersection of the surface with the pl ane
described by the set of points sv + tk. If v is not a unit vec tor , then the "slope" may be
determined by norm alizing v to be a unit vector .

4. Relation to partial derivatives . T he partials of/ox , of/ oy and of/ OZ are t he directional
derivatives of f in the directions i , j and k , respec ively.

5. Computing direction al derivatives. To compute t he direct ional derivative of fin the direction
of v , the easiest formula to use is "V f (x)· v . T he directional derivative is a scalar, not a vector .
The vector v is often chosen to be a unit vector .

6. Gradient pmperties. Recall that "V f = (af/ ox )i + (of/oy) j + (af/oz)k. You should know
that "V f points in the d irection in which f is increasing fastest and - \l f points in the direction
of fastest decrease. The gradient of f is always orthogonal to a level surface of f

7. Tangent plane. In terms of the grad ient , the equ ation of the tangent plane at Xo is "V f(xo) .
(x - xo) = O. This generalizes the formul a given is section 2.3.
36 CHAPTEP

SOLUTIONS TO SELECTED EXERCISES


1. The directional derivative of / (x) at Xo in the direction v is "V / (xo) . v . In thi case,

V' / (1,1,2)· (1/ V5,2/ V5, 0) = (z2,3 y 2, 2xz )I(1 ,1,2)' (l/V5,2/ v'5,0)
(4, 3, 4)· (1/ vf:5,2/vI5,0)
= 1O/v15 = 2v'5.
2. (b) Given / (x,y) = \n(lx2 + y2 ), we compute

"VI (x,y) =

= (1 , 0) and the directional derivative i V' /(1 ,0) · v = 2/ vI5.


At (1 , 0) , V'/ (1 , 0)
3. (b) Given /(x, y, z ) = eX + yz, we calculate "V /(x, y, z) = eXi + zj + yk and thus V' / (1 ,1 , 1) =
ei + j + k. The unit vector parallel to (1, -1 , 1) is (1, - 1, 1)/..)3. Thus, the directional derivati
is V' /( 1, 1, 1) . (1, - 1, 1)/..)3 = e/ ..j3.

4. (c) For a function of three variables, I (x, y , z), the tangent plane to the surface is

V' /(xo, Yo , zo) . (x - xo, y - Yo, z - zo) = 0.


To use this formula, we need to describe t he surface f(x, y, z) = constant. In this Cal:
f (x,y,z) = xyz = 1, so V'/ (x,y,z) = (yz,xz , xy) and at (1,1 1), V'f = (1, ] , 1) . Therefor
=
the desired tangent plane is (1,1 , ] ) . (x - 1, y - 1, z - 1) = 0, i.e., x + y + z 3.
5. (b) We have z = (cosx)(cosy), so Zx = - sinxeosy and, by symmetry, Zy = - sinycosx . .
(0 , rr/ ,0), we compute zX = 0 and zy = - 1. Therefore, the equation of the tangent plane i_

z = Zo + [zx (xo, yo)]( x - xo) + [Zy(xo . yo)]( y - yo ) = 0+ O(x - 0) - l(y - rr/ 2) ,

i.e., z + y = 7r/ 2.
6. (c) Given f (x, y, z) = 1/(x 2 + y2 + z2 ) = 1/1,2, we have Ix = - 2x / (x 2 + y2 + z2)2 = - 2x/r
By symmetry, I y = - 2Y/ I·4 and 10 = - 2z/ r4 . Then "VI = (- 2/ r4 )( xi + yj + zk) = - 2r/r
where r = xi + yj + zk and r = l x2 + y2 + z2.
7. ( ) The direction of fastest increase is along the gradient vector. U ing the result of exerci:
6(c), we get the direction of fastest increase as 1(1,1,1 ) = (- 2/9)(i + j + k ).
8. The gradient vector is normal to a surface. Here, we have I(x, y, z) = x 3 y'3 + y - z + 2 = _
so Ix = 3x2y'3, Iy = 3x 3 y2 + 1 and /z = -1. At the point (0 , 0,2), we compute Ix = 0, Iy =
and f. = -1. Therefore, a normal vector is V' / (O, 0,2) = j - k. Normalize this to get the un
normal: (l/.../2)(j - k) .
13. (b) By definition, "VI = (81/ 8x,8// 8y,8f/8z), so "VI = (yzexyz,xzexY"xye xyO). Giv
g(t) = (6t,3t 2 ,t 3 ) , we differentiate each componen to get g' (t) = (6 ,6t,3t 2 ) _ From Seetio
2_5, we know that

(lo g)' (I ) = V'/(g(I)) . g'( I) = exp(1 8t 6 )(3t 5 ,6t4 ,18t3 ) . (6 ,6t ,3t 2 )lt=1
= e18 (18 + 36 + 54) = 108e 1S _
17. By definition, "V I = (Ix , Iy ). Since f is independent of y, I y = 0 and given that I (x, y) = g( z
we have Ix = 81/ 8x = g' (x ). Th refore, "V I(x, y) = (g'(x ), 0).
DIFF ERENTIATION 37

20. T he direction in which the alt it ud is increasing most rapidly at the poi nt (x, y) is
\7z(x, y ) = (-2ax , - 2by).
At the point (1,1) , \7 z(l , 1) = -2(a, b) , so the desired direction is - (ai + bj )/Ja 2 + b2 • If a
marble were released at (1, 1) , it will roll in t he di rection at which the altit ude i decreasing
most rapidly, so the m arble will roll down in the direction -\7z, i . . , (ai + bj )/Ja2 + b2 .
24 . (a) We want to maximize I (c(t)) = (cost )(sin t ). Set the first derivative equal to 0: df/dt ::::
-(sint)(sin t ) + (cost )(cos t) = 0, so we get cos2 t = si n 2 t or t :::: ±(7r/ 4 + n7r ), where
n = 0, 1, 2, .. .. Since °.: ;
t .::; 211', we only want t = 7r/ 4,3 7r/4 , 511'/ 4, 711'/4. Evaluating at
these points, we get l (c{7r/4)) =
l (c (57r/4)) =
1/2 and l (c (37r/ 4)) :::: l(c(77r/ 4)) :::: - 1/ 2.
T herefore, the maximum value of lalong the curve c(t ) is 1/ 2 and the min imum value is - 1/ 2.

SOLUTIONS T O SELECTED R EVIEW EXERCISES FOR CHAPTER 2


1. (a) Since 3x 2 and y2 are non-negative, t here is no level curve if z < O. If z :::: 0, then t he I vel
cu rve is the origin. If z > 0, then t he level curve is an ellipse wit h the m aj or axi parallel to
the y axis and the minor axis parallel to the x axis. The ellipses get larger as z increase . Put
all of these level curves together to get an elliptical paraboloid .
2. (c) First, consider the surface x yz :::: O. T he surface consists of the planes x :::: 0, y :::: 0 and
z :::: O. Now consider x yz :::: 1. When z :::: k , a positi ve constant, th I vel curve is xy :::: l/ k .
T hus we get a hyperbola in the fi rst and third quadrants with asy mptotes on the x and y axes.
T he hyperbol as get closer to the origin as z gets larger. Thus, t he surface in the first octant
has t he plane x :::: 0, y :::: 0 and z :::: 0 as asy mptotes. Similarly, there is a surface in t he octant
where z > 0, x < 0 and y < O. Now , if z < 0, then there is a si milar surface in either of the
octants where x < 0 and y > 0 or x > 0 and y < 0. T he level surfaces for xyz :::: c, where c
is a positive constant , consist of four similar surfa.ces which move further from the origin as c
gets larger. If c < 0, the level surface is positioned in the other fo ur 0 tants.
3. (b) The function takes a point from ~ 1 to ~ 2 , so the si ze of the derivative matrix is 2 x 1. The
derivative is
D / (x) :::: [ ~ ].

5. We need to show the vector normal to the tangent plane of I (x , y) at the point (xo, Yo , I(x o, Yo) )
is parallel to the vector (xo, Yo , zo). T he partial derivatives of I are:
01
ox (xo, yo) 9- 1 2x (1 - x 2 2 /! -Xo
~
- Y )-1 2
(xa,ya)
::::
vI - Xo - Yo 2 2 .

8I ( ) -1 2Y(I_X2_y2)-1/2! :::: -Yo


oy xo, Yo 2 . /1 2 2'
(x a,Ya) V - Xo - Yo
So the normal to the tangent plane is
- Xo . -Yo .
. ,1 + J - k.
VI - X6 - Y6 VI - X6- Y6
Multiply t he above through by -( 1 - x 5 - y6)1 / 2. We get (xo, Yo ,/(xo ,Yo)) = (xo, Yo, zo).
Geomet ri cally, we are looking at t he sphere : x 2 + y 2 + z2 :::: 1. T he v ctors normal to the
tangent planes are precisely the vectors r = (x, y , z). Those vectors that have the direction of
e p (see exercise 7(a) in section 1.4) are perpendic ular to the sphere.
7. (c) T he equation of the tangent plane is
z :::: I (xo, Yo) + [(01/ fJ x )(xo , Yo) ]( x ~ xo) + [( 0I / oy)(xo, yo)](y - Yo).
In this case, we have 1(-1, -1) :::: 1; ol / ox :::: y, (of/8x )( - 1, -1) :::: -1; of/oy :::: x ,
(of/oy)( - I , -1 ) :::: - 1. T herefor , the equation of t he tangent plane is z :::: 1 - l (x + 1)­
l(y+ 1), i.e., x +y+ z + 1:::: O.

. .
38 CHAPTER.

8. (b) If f (x , Y, z ) = constant, then the equation of t he tangent plane is "il f(x o) . (x - x o) =


where x = (x , y,z). In this case, f(x,y,z ) = x 3 - 2y3 + z3, so "ilf(x) = (3x 2 ,-6y2,3;­
and "il f (l, 1, 1) = (3, -5 , 3). Therefore, the tangent plane is (3, -6, 3) . (x - 1, Y - 1, z - 1) =
3x - 5y + 3z = 0, i.e. , x - 2y + z = O.
11. (b) The strategy here is to find a few "paths," compute the limit along those paths. L­
x = 2y; then the limi t as y goes to 0 is

(x,II)~(o,O)
lim yF X+YI-I- y~
';-=Y f
r Y1-;1-I- v'3.
3Y

On the other hand, take the path x = 4y. Then the limit as y goes to 0 is

lim
(x, y )-+ (O ,O )
JI + x y
x- y
lim f5I = f§. =/: v'3.
1= y~O Y
Y13"Y 1 V3
Since the limiting vain s depend upon the path taken, the limit does not exist .

++
12. (b) Hold y and z constant and use the hain rule to differentiate with respect to x. Th
ofl ax = 10( x y z)9 . By symmetry, ofl ay = ofl az = 10(x y + z )9. T he gradient is t
vector (of/ox, of/oy, of/oz ) = 10(x + y + z)9(i + j + k ).
+
16. We compute
~z (1, - 2) = 2x l = 2; ~z (1, - 2) = 2y l = - 4.
x ( 1,- 2) Y ( 1,-2 )

Then , the tangent plane is z = f(l, - 2) + (8 zl ox )I(1, _2) (x - 1) + (az l ay)I(1,_2) (y + 2),
z = 5+ 2(x-1) - 4(y+ 2) or 2z -4y - z = 5. Geometrically, the gradient of f (x , y) = x 2 +
at (1, - 2) is perpendicular to the level urve z = 5. The tangent plane of the graph of f
t he plane that contains the line perpendicular to the gradient of f at (1, - 2) and lying in t
horizontal plane z = 5, and the tangent plane has slope .../22 + 42 = 2v5 relative to the
plane.
18. (b) The directional derivative in the direction ofv is "ilf(xo) , v /ll v il. We compute "ilf(x) =
(y + z , x + z , x + y) , so "il f(l, 1,2) = (3, 3,2) . Also, we have

Ilvll = ) 10 2 + (- 1)2 + 22 = .../105.


Thus, the directional derivative in the direction of v is (3, 3, 2) . (10 , - 1,2) I v105 = 31 / M
21. The bug should move in the direction of - "ilT(x,y ), since this is the direction of fast
decrease. We compute - "ilT(x, y) = - 4xi + 8yj and -"ilT(1 , 2) = 4i + 16j, so the bug shou
move in the direction 4i + 16j.
24. Let u = :z: - y, then
01 01 and 81 a! du of
ox ou ay au . dy - AU .

Holding y constant, we get

8z =
ax
(!:.) 81 = (!) 01 .

y ax y au
Holding x constant and using the quotient rule , we calculate

az
ay
=~ [al . y _ 1(1)] = ~ [_ of . y - I].
y2 oy y2 au
Therefore,
DIFFERENT IATION 39

27. (ii)
(a) The sum rule tells us that x 2 + y4 is differenti able. Also , x 2y2 is differentiable by the
product rule. Finally, x 2 y2l(x 2 + y4) is differentiable by the quotient rule since (x , y) 1- (0 , 0),
and so x 2 + y4 i- O. Holding y constant, we get

of _ 2xy2(X 2 + y4) - 2X(x 2 y2) _ 2x y6


ax (X2+y4)2 (x +y4)2 ·
2

Holding x constant , we get

of 2x 2y(x 2 + y4) _ 4y.3 (X2 y2) 2x4y - 2x 2 y 5


oy (x 2 + y4)2 (x 2 + y4)2
At the origi n, we use the definition of the partial derivative:

o
o f (0 0) ::= lim f(O + h , 0) - f(O, 0) ::= lim h 2 - 0 = lim 0 = 0,
ox' h-+ Qo h h-+ O h h-+ O

and similarly,

o
of (0 , 0) = lim f(O , O+ h) - f(O , 0) = lim }l-~ = lim 0 = o.
By "-+0 h "-+0 h "-+0

(b) The partials exist at (0,0) and

lim f( x, y) = 0,
(x ,y)-+(O,o) II(x, y) - (0,0)11
so by the definition of differentiability (equation (2), section 2.3), f is differentiable at (0,0).
The function f is differentiable at all other points because the partials are continuous. Thus f
is differentiable. However, as (x,y) approaches the origin, aflox and o f loy do not approach
o (take, for example, the path x = y); t hus the partial derivatives are not continuous .
28. (b) The gradient vector is 'V f = (oflox, ofloy). If (x, y) i- (G, 0), then

-of
ox
= YSID. ( x 2 +y2 1) -
2x y
2

(X 2 +y2)2
cos (1) x 2 +y2'

and by symmetry

-of =
oy
. (
X Sill
1)
X2 +y2
-
2x y2
(X 2 +y2)2
cos (1)
1
x 2 +y2 ·

Now, if (x, y) = (0 , 0), then the definition of the partial derivative gives us

of
0
-
_ .
(0 , 0) - hm
f(O + h, 0) -
h
f(O , 0) _.
- hm
(0 + h)(O) sin ((0 +
h
h)~ + (0)2) =
I.
1m -
0
= o.
0X "-+ 0 h-+O h-+O h

Similarly, (ofl oy)(O , O) = O. Therefore, if (x,y) i- (0,0), then

[ . ( +1)
2
'Vf(x,y) = YSID
x2 y2
- 2x y ') cos (
(x2 + y2 )~
[
x 2 + y2
)] J•

+ [ . ( +1)
XSIl1 x2 y2
2xy2
- (x 2 + y2)2 cos . x 2 + y2
(1)] . J,

and thus 'V f(O, 0) ~ Oi + OJ.


40 CHAPTER _

29. (b) T he directional derivative is 'V J (x o) . v. Here,

aJ =_ x sin(v x2 + y2 ),
ax V x2 + y2
and by symm try,
~~ vx+ZY y2 sin( v x
2
+ y2 ) .
Therefore, 'V J (1, 1) = (( - 1/ V2) sin (V2), (- 1/.;2) sin (.;2)) and the directional derivative is

(( -1/ v2) sin (v2), (- 1/ v2) sin (h )) . (1 / Y2, 1/ 0 ) = - sin (v2).

33. (b) Directly, we first compute g(u) = J (h (u) ) = sin 2 31.1 + cos 8u . Then
!~ = 6(sin 3u)(cos3u) - 8sinSu.

When u = 0, dg / du = O. By the chai n rule,


-ddgu = DJ(x,y) ·D h(u) = [2x, 1] [ -38cos
' 3U
sm Su
] .
= [2sm3u, 1]
[ 3 cos 31.1 ]
- S sm
' 8u
= 6(sin 3u)(cos 3u ) - 8 sin Su.

Again, wh n u = 0, dg / du = O.
35 . The normal to the surface J(x, y, z) = x 2 + 2y2 + 3z 2 = 6 is
aJ aJ aJ )
( ax' oy' oz = (2x, 4y, 6z ) = (x, 2y, 3z ).

At (1, 1, 1), the normal is (1,2,3), so he unit normal is (1,2, 3)/ Jf4, the direc ion offiight . T
veloci y of the particle is the speed times the direction , or 10( 1,2, 3)/J14. The position of
particle at any time can be found by finding the equation of t he line through (1 ,1, 1) with
direction 10( 1,2, 3) / V14, and it is (1, 1, 1)+10t (l , 2, 3)/Jf4. This impli s tha.t x = 1+10t/ K
y = 1 + 20t/ v1A and z = 1 + 30t/ ..;u. At som t ime T, the particle is on he sph
x 2+ y2 + z2 = 103, which means that (1+ 10T ;./f4) 2 + (1 + 20T / ..;u)2 +( 1+ 30T/J14)2 = 1
Simplifying, we get 3 + (120/ v'14)T + 100T2 - 103 = O. Solving for T using the quadra
formula., and taking th positive T only, we get T = (-3 + V359 )/5V14.
3S. (a) By substitution, Z = (x + y)(x - y) = x2 - y2, so az/ ax = 2x and az/oy = - 2y.
(b) By the chain rule, we have

az oz aU. oz av ,
ax = au' ax + ov' ox = vl1) + u(l) = v+ u = 2x.
Also, we have
oz az ou az ov
ay = au . ay + av . By = ve l ) + u(-1) = v - u = - 2y.
41. Let u(t) = J (t )g(t) and h(u) = ell. By the chain rule, we have

dh dh du [ dJ d9 ] J 9
dt = du . dt =e U
dtg (t) + J (t) dt = [ddtg(t) + J(t) ddt ] exp [J(t)g(t)]..

46 . The velocity is d fined as the derivative of displacement. Therefore, we want to compu


au/ot = - 6cos(x - 6t ) + 5cos(x + 5t ). When t = 1/3, x = 1 and the velocity is au./at =
-6 cos (-1) + 6 cos(3). Sine cos ( - x) = cos x, the velocity is 6(cos (3) - cos( l )).
DIFFERENTIATION 41

49. (a) As given, P is a function of T and V. We can also write

so T is a function of P and V. Finally, we can write

p = RTV2 - a (V - {3)
V2(V - jJ) .

Upon rearranging, we get

pv 3 - (PjJ - RT)V2 + o:V - 0:{3 = O.


Since this is a cubic equation in V, it is theoretically possible to write Yin terms of P and T.
Therefore, any two of V, P or T determines the third variable.
(b) From the equation for T , we hold V constant and get

(aT/a P ) = (V - {3)/ R .

From the equation for P, we hold T constant and get

(fJP/fJV) = -RT/(V - jJ)2 + 2Vo:/ V 4 = - RT/(V - jJ)2 + 20:/V 3.


Now, hold P constant and differentiate the equation for P by implici t differentiat.ion . We get
aV aV .
_ R(V - jJ) - 7jf( R T) 2V7jf0: _ ~ _ aV [ RT _ 20:]
o- (V - {3)2 + V4 - V - {3 aT (V - jJp V3'

or equivalently,
aV R
aT - (RT 20:) .
(V - jJ) (V _ {3)2 - V3
(c) Using the results of part (b) , we get

aT) (BP) (BV)


( aP BV fJT = ~
(V - {3) ( - R T
(V - jJ)2 + V3
20: ) [
(V _ jJ)
R
((v~~p _ ~~) 1=-1.
50. (a) The question is asking for the directional deriva ive in the direction of a unit vector. Here,
our unit vector is (1,1)/v'2. Also V h(x , y) = (-0.00130x, -0.00048y) and Vh( -2, -4) ==
(0.00260,0.00196). Therefore, the directional derivative at (-2, -4) in the direction of (1,1)/0
is 'Vh( -2, -4) . (1, 1)/v'2 = 0.00456//2. T his means th at the height increases (0 .00456//2)
miles per horizontal mile traveled.
(b) The direction of the steepest upward path is V h( - 2, -4) = (0.00260,0.00196) .

TEST FOR CHAP T ER 2


1. True or false. If false, explain why.

(a) If a 3-variable function ! (x , y, z) has partial derivatives l x, Iy and Iz at the origin, then
the function is differentiable at t he origin.
(b) A grad ient vector for a function fin IR 2 is parallel to level curves of f
(c) A function f is continuous at a point Xo if

lim I(x}
X---+Xo
= I(x o).

42 CHAPTER _

(d) In general , a ontinuous function is differentiable.


(e) Given a function !(w, x, y, z), th directional derivative in the direction of (0, 0,1,0)
the same as 8f18y.

2. Let u = x 2 y + z and v = xyz. Also, let x = 2a + h, y = ab 2 and z = a + 2a 2 b. Fu r t her mor~


let a = d3 and b = c - d. Compute 8(1.£, v)18(c , d) at (c, d) = (1,1) .
3. Use the linear approximation to estimate th distance between the origin and the poi..
(8.01,3.9 1.04).
4. Let c(t ) be a mapping from ]R to ]R3 and let ! (x, y , z ) be a mapping fro m ]R3 to R. 10::­
h(t ) = !(c(t )). When t = 1, we have (z, y, z ) = (- 1,2,4) , c'( I ) = (3,4, -2), dyldt = 1 ar
dz l dt = 2. Is the rate of decrease fastest in the positive :c di r ction , the negative x directi
the positive 11 direction r the negative y direction?

5. Let z = (z + y) 2 - 5x 3 + 2ye X be the equation of a surface in space. Find the equation f t


tangent plane at x = 0, y = 3.
6. Let
Y.., if x i= 0
g(x, y) = x .
{
o if x = 0
Compute 8g18z and 8g 18y at t he origin if they exist there.
7. Let u and v be functions of x and V, and let x and y be functions of s and t. Furthermore.
is known t hat
8(
z, y) =
8(s, t)
[2 1]
0 -1
and
8(u, v)
8(z, y)
0 - 1]
[ 3 2 .
Calcul ate 8u.18sand 8v 18t.

8. (a) Evaluate the following limit for !(w, z, y, z) =w- z2y3z :

lim !(5, 2, -1, 1) - !(5, 2, - 1 + h, 1).


h -+ O h

(b) Is it possible to define g(O, 0) so that g(x , y) = (x 2 + y2) /( x 2 + y) is continuous on all


1R2? Explain why or why not .

9. Let u (x, y ) = 2x + y2 + eX. At (1,1) show that u increases faster in the direction parallel

the x axis t han in th direction parallel to the 11 axis.

10. A recent survey showed that patient satisfaction, s, at a pharmacy depended mainly up..
three factors - t he waiting time in minutes (t) , the patient's percei v d degree of illness (i)
a scale from 0 to 10, and t he dollar cost of the prescription (c) . T h patient satisfaction ind
=
is given by S (t, i, c) (1000 - c) / it 2 .

(a) In what direction does patient satisfaction increase most rapidly at the point (1 0, 0.5, 10C
(b) At the point (10, 0.5, 100) how fast and in what direction (positive or negative) d
patient satisfaction change for each extra minute of waiting time?
(c) The administrators do not want s to decrease by more than 1 per un it change of t, i
c. Is this goal met with a price decrease of 3/ 5 dollars and an increase in waiting time
4/ 5 minutes? Explain .

..--------------------------~----------------------~------------------
43

3 HIGHER-ORDER DERIVATIVES;
MAXIMA AND MINIMA

3.1: ITERATED PARTIAL DERIVATIVES


GOA LS
1. Be able to comp ute iterated partial derivatives.

2. Be able to explain when mixed partials are equal.

STUDY H INT S
1. Iterated partial deri vatives. T hese are high r-order derivatives, such as second and th ird deriva­
tives. With several variables, higher-order deri vat ives may be taken with respect to different
variables. The notation :::y means (:x ) (~~) , which is also denoted fy x .

2. EquaLity of m ixed partials. If the second partial deri vatives are cont inuous, t hen an iterated
partial derivative may be computed in any order.

3. W arning. Note that the theorem on equality of mixed partials requires continuous partial
derivatives. If this requirement is viol a ed , different orders of differe nti ation may yield different
results.

4. EvaLu ating partials at a given point. Always remember to differentiate com pletely before sub­
sti tuting given values. With mixed partials , you may substitute for a variable only after you
have completed differentiating in that variable.

5. A pplications. T he heat equation, the wave equation and the potenti al (Laplace) equation are
famous examples of how higher-order derivatives occur in nature. T here is normally no need
to memorize these equ ations in a vector calculus course.

SOLUTIONS TO SE LECTED E XERCISES


2. Note that when we differentiate wi t h re pect to either x or y, eZ is "constant ," so the first partial
= =
derivatives are of/ox -1/x 2 + e- Y and of/oy - x e- Y F inding the partial derivatives of
the first partial derivatives gives us these second partial derivatives:

7. (b) Rewrite the func ion as


2X2 + 7x 2y 2x 7x
z=
3x y
=-+­
3y 3 '
provided x #-O and y #- O. W compu te:

oz
ox
2
= 3y + 3;
7 fJz
fJy
2x
= - 3 y2 ;
fJ 2z
ox 2
0 (Ozox ) = 0;
= ox

44 ( HAPTE
2
/)2z a ( oZ ) 2 a (az ) z a 2
a z 8 ( az ) 4x
8xay = 8x 8y = - 3y2 = ay ax = ay8x; ay'.! = ay ay = 3y3 ·
The function is not continuous if either x = 0 or y = O. Hence, the funct ion is not differentia
whenever x 0 or y O.= =
..
3
a f 8
11. By defirutlOn, oxayaz = ax ay az
(8 f ))(0 af oa f a ( Oh )
. Let h = az ' so axayoz = Ox oy and
8 (8h)
theore m 1, this is als equal to ay ax . Also, by theorem 1, ax
ah = oxa ( aozf ) = 8z 0 ( aaxf
and t herefore
o3f a
8xayaz = ay
(88z (88xf ) ) = 8yozax·
oaf

15. (a) We are given f(x , y) = x arctan (x/ y), so we compute:

fx arctan (~)
y
+ ::' 1
y +
(\ /
xy~
~) = 2 xy 2
x+ y
+ arctan ( ~)
y
.
x -x _ x2
fy = 1 + (x 2 / y2 ) . --:;J2 = x + y2 ·

y(x 2 +y2)_ 2x 2y 1 1 y3 _ x 2y Y 2~

fx x = (x 2 + y2)2 Y
+ 1 + (x 2 / y2 ) - (x 2 + y2 )2 + x 2 + y2 - (x 2 + y2) 2·
-2x(x 2 + y2) + 2x 3 _ 2xy2
/:ey fyx = (x 2 + y2 )2 = (x2 + y2)2·
x2 2
fyy = --=-~"7.
(x2 + y2 )2
2y = (x22x+ y2y )2 .
19. We have U x = 3x 2 -6xy , so U xx = 6x - 6x = O. Also, u y = 3x 2, so U yy = o. Substitution gi
us 'U xx + 'U yy = 0 + 0 = o. Thus, u (x, y) satisfies Laplace's equation and so u(x, y) is harmon
= =
20. (b) For u = ;c2 + y2 , we get U x 2x, so U xx 2. Also, u y = 2y, so U yy = 2. Substitution in
Laplace's equation gives us 'U x ., + U y y = 2 + 2 :f. 0, so x 2 + y2 is not harmonic.
(d) For U = 11 + 3x 2y, we get u., = 6xy, so Uxx = =
6y. Also, U y 3y2 + 3x 2 , so U yy =
Substitution into Laplace's equation gives us Uxx + U yy = 6y + 6y "I 0, so y3 + 3x 2 y is r
harmonic.
23. Given V (x, y, z ) = - GmM/J x2 + y2 + z2 = - GmM/ r, we compute:

GmMx

24. (a) If (x, y) "I (0,0), we can compute the first partial derivatives in the usual way:
af = (y(x 2 - y2) + 2x 2y)( x 2 + y2 ) _ 2x 2y(x 2 _ y2 ) _ x4 y _ yS + 4x 2y3
ax (x 2 + y2 )2 (x2 + y2 )2
8f 2 2
(x(x - y2 ) _ 2xy2 )(x + y2 ) _ 2xy2(x _ y2)
2 x _ 4x 3y2 _ xy4
5

ay (x 2 + y2 )2 (x2 + y2) 2
HIGHER-ORDER DERIVATIVES ; MAXIMA AND MINIMA 45

(b) To compute the part ial derivatives at (0,0), we need to use t he definition of t he partial
°
derivati es. First, hold y constant at and differe ntiate with respect to x at Xo = 0. ate that
f(O,O) is defined to be 0. Then
h(0)( h 2 - 0 2 )
2
fj l (0 0) = lim 1 (0 + h, 0) - 1 (0,0) = lim --..:hc::.. .---'+h,--0,,-2_ = lim Q= lim 0= 0.
fjx ' 11-+0 h h-+O h-+ O h 11-+0

Similarly, we hold x constant at ° and differentiate with respect to y at Yo = 0. Then


0(h)( 02 - h2 )
{)I (OO) - I' I (O , O+ h )- I (O,O) -_ l'1m
, - 1m
02 + h 2 = "lim Q = 0.
fjy 11 -+ 0 h h-+O h -+ 0 h

(c) By definition, :::y = :x( ~: ). First, we use ~~ from part (a) and then perform
di ffe rentiation as in part (b) . By definition,
{)f fj f h5
4h3(0)2 - h(0)4
-
{P I . {)y (O+h,O)- {)y (O,O) (h 2 + 02 ) 2 -0 . h5/ h4
-{){) (0,0) = hm
x Y h-+O
h = hlim
-+O
_ _'---_ ..,-!....J _ _ _ _
h
= 11hm
-+ 0
- - = l.
h
Similarly,
fj f of (0) 4h - h + 4(0 )2h 5 3

02 f - (0,
--(0,0) = lim {)%
°+ h) - -(0,0) (0 +
= lim --'----~----
fjx
° 2 h2 )2 -

oyox h-+ O h h -+ O h

(d) The mixed parti als are not equal, which is consistent wi th the fact that the second partials
are not continuous at (0 , 0) .

3.2: TAYLOR'S THEOREM


GOALS
1. Be able to write down t he first few terms of Taylor's formula for a given fun ction.

STUDY HINTS
3
1. Notation. The summation symbol L: means to sum all possible combinations of (i, j) with
i ,j = l
i and j ranging from 1 to 3, i.e., (i, j ) = (1,1), (1 ,2), (1 , 3), (2, 1), (2,2), (2, 3), (3, 1), (3, 2), and
(3,3) . In general , if m indices are summed from 1 to n, there will be nm terms; in our case
there are 32 = 9 terms.
2. Review. Before continuing , you may wish to review Taylor's for mula from your one-variable
calculus text. Recall that the Taylor series can be used to approximate the values of functi ons.
3. Taylor 's fo rmula . You should know the pat tern for the general formula. As a reminder,
Taylor's for m ula is

The second term , which involves the second partial derivatives , will become important in
the com ing sections. The term involving the third partials sums up 3 n terms, so it may be
unreasonable to ask you to com pute all of these terms unless n = 2.
46 CHAPTER

4. Computing Ta ylor 's f ormula. Remember that you will n ed to compute all of the partl
derivatives of the same order. For example, when computing second partials, on must compu
82 f 82 f 82 f .. 82 f 82 f (j2 f
Xl
aX n
-82' -82' ... , 2 as well as all of the mixed partials 8 8 '8 8 ' 8
X2
,etc. Note th a
Xl X2 X2 Xl Xl
aX3
we do not need to compute !.l 8
U XiUX j
!
2
,i f. j, twice because mixed partials are equal (assuminz
continuity) .

5. Taylor's formu la remainder. Recall that in one-variable calcul us, the remainder is determina.
at some point between X o and X o + h. Now the r mainder is det rmin ed at some point on th
line between the vectors Xo and Xo + h, where h = (hl ' h2 , ... , h n ).

SOLUTIONS TO SELEC TED EXERC ISES


1. Recall that Taylor's formula is a polynomial which approximates a functi on. If our function is
itself a polynomial, then this functio n must be its own Taylor series as well. Hence the second­
order Taylor formula for f is f (h 1 + h 2 ) = (hi + h2 )2 = h~ + 2h l h2 + h~ at X o = 0, Yo = C
Alternatively, 8f/8x = a f / ay = 2{x + y) and 8 2 f/8x2 = 8 2 f / 8x8y = f}2f/8 ya x =
z
8 // 8y2 = 2. At the point (0,0), f (x, y) and the first partials are all 0, and all of the second
partials are 2. Thus, the Taylor approximation is (1/ 2)(2)(hI + 2h 1hz + h~ ) = (hi + h2)2 .

5. Here, 1(0,0) = =
1, f x ycosxy - ysinxy, fy =
x cosxy - xsinxy, In == -y2 sinxy - y 2 cosx y
fxy =
fyx =
cos xy - x y sin x y - sin xy - xy cos x y and fy y _ X2 sin xy - x 2 cos xy. At (0 , 0) =
we have Ix fy = = =
f xx I yy 0 and fxy = =
1. Thus, the Taylor series is

1
f(h l , hz) = 1 + Oh l + Ohz + "2(O h 1 + 2hl h2
2
+ Oh 2z ) = 1 + h1h z + Rz(O, h).

7. (b) For x > 0 and x < 0, f (x) = exp(- l / x) is infinitely d ifferentiable . For x = 0, we must use
the definition of the derivative:

1'(0) = lim f( x ) - f (O) = lim exp( - l / x ) .


x--+o+ X x--+ o+ X

Let u = l/x. We must show that lim.. --> oo u exp(-u) = O. By I'H6pital's rule,

lim ue - u lim (.!!:..) = lim


= u--+oo ( .-!..) = o.
u--+ 00 e tI --+
U 00 e U

In general,
(n - 1)(X) _ f( n-l)(o)
f(n)(o) = x--+li mo+ / X
.

I"( x ) = .-!..2 exp ( -1) , f(3 )( x ) = (-32 + .-!..) exp ( - 1) ,


x X X :1;4 X

so if we can show t hat lim.. --+ oo une - u = 0 for all n, then we can conclude that fis Coo at x :::: 0
as well. Again, use I'Hopital's rule n times:
. _ . un . n!
lI m une
u--+oo
" = u--+
hm - = ... = lim - = O.
oo etl u--+ oo e"
Thus, f is C oo , with all derivatives equaling °at x = 0. Now, 1 (0 + h ) = exp( -l/h) > 0, but
I f(k)(O) k
f (O) + f (O)h + ... + - k !- h + .. . = o.
Hence, f is not analytic.
HI GHER-ORDER DERIVATIVES ; MAXIMA AND MINIM A

EXTREMA OF REAL-VALUED FUNCTIONS


GOALS

1. Be able to fin d th critical points of a real-valued two-variable fu n tion.

2. Be able to use the Hessian to classify the critical points of a functi on as maxima, minima, or
saddles .

STUDY HINTS

1. Defini tions. (a) A local or relative extremum is a point Xo where ! (x o) is largest or smallest
in a sm all neigh borhood of Xo .
(b) An absolute extremu m is a point Xo where !(xo) is largest or smalles on the entire doma in
under consideration .
(c) Critica l points occu.r where all first partial derivatives are zero.
(d) A saddle point. is a critical poin L which is not a local extremum .

2. Criti cal po in t-extrem um relatzonshlP. All extrem a occur a t critical points, but not all critical
points are extrema. C ritical points m ay also be saddle points.

3. Real-valued f unctions. Note t hat we are comparing function values for re I-valued fu nctions,
not vector-val ued functions .

If a! / ox = 0 and {}!/ {}y = 0 have more than one solut ion , each com binat ion
4. F in ding extrema.
of (x, y) which satisfies t hese conditions must be considered. You must he complete in your
analysis. See example 8.

5. Hessian . T his is denoted by H! (x o)(h ) and it is equal to the second term of Taylor 's formu la,
. . 1 [p!
wh ich IS "2 L n

; ,j= 1
hi hj ~ ( xo).
Xt xJ

6. Deter-mining definiteness . To determine definiteness in genet I, we need to know the


determinants of the diagonal submatrices of the Hessian matrix.
Starting from the upper left-hand corner , i.e., all , compute the deter­
minants . If they are all positive, t hen the Hessian is positive defini te .
If a11 < 0 and t he signs alternate, then the Hessian is negative defi­
nite. ote that this test includes t heorem 6.

7. Usefulne ss of H essian. At a critical point, the first partial derivatives are all zero , so Taylor 's
fo rmul a reduces to
=
f (xo + h ) ! (x o) + Hessian + remainder,
where the remainder is small compared to the Hessian. Thus , if H ! (xo) is positive definite ,
then
!(Xo + h ) = ! (xo ) + Hessian + remainder > ! (xo),
so ! (xo) is a rel ative minimum . Similarly, if H ! (x o) is negative definite, then

! (xo + h ) = !(xo) + Hessian + remainder < !(xo) ,


and f(xo) is a relative m aximum .

8. Classifyi ng a critical poin t Xo. (a) If H! (xo ) is positive definite, then Xo is a local m inimum.
(b) If H ! (x o) is negative definite, then Xo is a local maximu m.
(c) If Hf (x o) does not satisfy (a) or (b) and not aU of the submatrices are zero, t.hen Xo is a
saddle point.
48 CHAPTE R 3

9. Second deriv ative test. T heorem 6 is a special case of how the Hessian is used and is often
used for problem solving. To use this test , one must compute the discriminant

D = ((J2 f)
2
(()2f) _(fl) 2
()X ()y2 () X()y

° °
If ()2 f I ()X 2 > and D > at a critical point , then we have a local minimum. If ()2 /1 ()x 2 <0
at a crit.ical point, then we have a saddle point.
°
and D > 0 at a critical p oint, then we have a local m aximu m. And if ()2 /1 ()x 2 > and D <0

10. Minimizing distance. Example 8 shows that distances may be m inimized by analyzing d 2 .
This is justified by the chain rule. By differentiating d2 , the chain rule gives 2d(adl ox) and
2d(adlay), so we again are solving adlax = 0 and adlay = O. Since d ~ 0, the maximum (or
minimum) of d 2 occurs at the same points as the maximum (or minimum) of d.

11. Guaranteeing absolute extrema. In lR! ', if a domain is closed and bounded, and tis continuou5
on the domain, then there exists an a bsolute minimum and an absolute maxi mum. All thre
conditions are neces ary. Think about what happens if the domain is not closed, if t he domain
is unbounded or if / is not continuous . Compare this to the extreme value theorem of one­
variable calculus.

12. Locating boundary extrema. In this section , one parametrizes the given bound ary of a region
and then differentiates to determine the local minimum and maximum points. Another method
is introduced in the next section.

SOLUTIONS T O SE LEC TED EXER CISES

3. We compute the partials aflax = 2x + 2y and a/lay = 2y + 2x . These partials vanish at


points such t hat x = -v, so these are the critical points. Since f (x, y) = (x + y)2 ~ 0 for all
(x , y), t.he extrema must be minima.

=
5. By the chain rule a/ lox 2x exp (1 + x2 - y2 ) and a f/ ay =
-2y exp(1 + x 2 - y2). Setting
these partials equal to 0, we find that (0,0) is the on ly crit.ical poi nt . To classify this critical
point, recall that the exponential fu nction is monotonic. Looking at the function along the %
°
axis, set y = t o get f (x, 0) = exp(l + x 2 ). Thus, x
° =° is obviously a m inimum. If we look
at the y axis , we set x = to get f (O,y) = exp(l- y2). The reader should be convinced that
°
y = is a maximum. T herefore , we conclude that (0, 0) is a saddle point .

9. At (0, Q1.50s~+ y2) = l. T he cosine function cannot exceed 1, so (0,0) is a maximum.


At (v7r/2 , V7r/2) , cos(x 2 + y2 ) = cos(7l') = - 1, so (v;J'i, v;J'i) is a m inimum since the
cosine function is never less than - l. Now, consider the critical point (0, ft). We calculate
cos(x 2 +y2) = -1, so this critical point is also a m inimum. Note: This exercise cannot be done
by the max-min test for functions of two variables (Theorem 6) because the second partials all
vanish at the critical points and so t.he t est is inconclusive.

a
10. The partial derivatives are 0/1 ox = in y and f/ ay = 1 + x cos y. Setting t hese equal to O.
we see tha t the critical points are (- 1, 2n7r) and (1 , (2 n + 1)7r), where n is an integer. Since
0 2 f I ax2 = 0, we cannot use theorem 6. Consider the case when x = 1. Our fu nction becomes
f(y) = y+sin y. The graph of f( y) shows t hat when y = (2 n+ 1)7l', there is an inflection point
there. Therefore , we conclude that the point (1 , (2n + 1)7l') is a saddle point since, along t he
line x = 1, the critical point is neither a maximum nor a minimum. Si mil arly, if x =
-I, th D
f( y) = Y - siny has inflection points at y = 2n7l'. Therefore, the cri tical points (-1,2n7r) are
also saddle points.
HIGHER-ORDER DERIVATIVES; MAXIMA AND MINIMA 49

fly) fly)

-11: 11: 2n
y y

fly) = y + siny .try) = y - siny

14. First, we compute of/ax = (y cos xy)/(2 + sin xy) and of/ oy = (x cos xy)/(2 + sin xy). Since
the denominator is between 1 and 3, we only need to solve y cos xy =°
and x cos xy == 0. From
°
ycosxy = 0, we get y = or xy = (2n + 1)11"/2, where n is an integer. From x cos xy = 0,
we get the additional solution x = 0. To classify the extrema, we look at f(x, y) . Since
-1 $ sin xy :::; 1, we have In(l) :::; f(x, y) :::; In(3) . So when xy ::: -311"/2,11"/2, ... , (4n +
1)11"/2, we have f(x, y) = In(3) and these points are local maxima. Similarly, when xy =
-11" /2,311" /3,711" /2, ... , (4n + 3)11"/2, we have f(x, y) =
In(l) and we have local minima. Now,
look at (x, y) = (0, 0). If x = y, then In(2 + sin x 2 ) is a minimum when x = y = 0. On the
other hand, when x = -y, we have In(2 + sin(-x 2 )) =
In (2 - sinx 2 ), which is a maximum
when x = = y 0. Therefore, the point (0,0) is a saddle point .

(31t12) 112 x

z =In(2 + sin i )

(91tI2) 112 x

z = In(2 - sin i )
18 . We shall find and classify the critical points in terms of k. First, set the partial derivatives
°
equal to to find the critical points. We have

of = 2x+ky=O and of =2y+kx==0.


ax oy
If k f:. ±2, the only critical ponit is (0,0). We shall return to the case when k = ±2 later . The
second partial derivatives are
02f 02f o2f
ox 2 = 2, oy2 = 2, and oxoy =k .
50 CHAPT ER .

The discrimin ant is D = 4 - k 2 . If k 2 < 4, i.e., - 2 < k < 2, then D > 0 and sine
0 2 f /ox 2 > 0, the second derivative test tells us t hat (0, 0) is a local mini mum. If k 2 > 4, i .E
k > 2 or k < - 2, t hen D < 0; therefore, (0,0) is a saddle point if k 2 > 4.
For t he case k = 2, we have

The graph of f is a parabolic cylinder that opens upward and whose vertex is the line x =­
on the xy pl an'~' Similarly, fo r the case k = - 2, we ge t

In this case, the graph of f is a parabolic cylinder that opens upward and whose vertex is th
Line x = y on the xy pl a ne. W hether k = 2 or k = - 2, f attains t he minimum value of 0 a t i ;:
vertex. In summary, (0, 0) is a local min imum if Ikl < 2 and (0,0) is a saddle poin if Ikl > :.
Minima occur a t points where x = -y if k = 2 and they occur at poin ts where x = y if k = -_

19. (a) We calcula te the partials: of/ox = -6x( y - x 2 ) - 2x (y - 3x 2 ) and o f /o y = (y - x 2 ) ­


(y- 3x 2 ) . Setting o f/oy equal to 0, we ge 2y -4x 2 = 0, which im plies y = 2x 2 • Substitutior
into o f/ ox giv s - 6x( 2x 2 - x 2 ) - 2x (2x2 - 3x 2 ) = - 6x 3 + 2x 3 = 0, which implies th at x = .
and so y = O. Therefore, (0 , 0) is a crit ical point.
(b) Let g(t) = (at , bt ). T hen f (g(t) ) = (bt - 3a2 t 2 )(bt - a2 t 2 ) = b2 t 2 - 4a 2 bt 3 + 3a 4 t -l
Differen iation gives f ' (g(t )) = 2t b2 -1 2a 2 bt 2 + 12a 4 t 3 = t (2b 2 - 12a 2 bt + 12a 4t2 ) which impIieo
=
that t 0 is one of t he solutions. The second derivative is f" (t) = 2b2 - 24a 2bt + 36a4t 2 and
f"(0 ) = 2b ~ 0, independent of b (or a), except possibly at b = O. For b = 0, f (g (t) = 3 a4 f~
2

which has a minimum at t = O. Thus, t = 0 is a rela tive minim um of f (t) along any straigh
line t hrough t he origin.
(c) Look at the par abola y = 2x 2. Then f (x, y) = (_ X2)(:z;2) = _ x 4 < O. T hus, along this
curve, f(O ,O) is a maxi mum.

23 . Given a vol ume V, suppose t he dimensions are x x y x z , then V = xyz and t he surface
area is S =
2xy + 2yz + 2x z . We want to mi nimize S. Solve for z, we get z = V/ x v . T hen
S = 2x y + 2V /x + 2V/ y . Taking partials, we get

oS
2y - 2V(1/x 2) (I
ox
oS 2x - 2V (1/ y2) (21
oy

Setting (1) equ al to 0, we get y = V/ x 2 . Substitu te this into (2) and set it equal to 0:
2x - 2V/(V/x 2)2 = 0, which is equivalent to x - X4/ V = 0, or x 3 = V , or x = V I/3 . T h n
y = V/x 2 = V/ V 2 / 3 = V I/3, and z = V / x y = V / (V I / 3 V I / 3 ) = VI/ 3. All t hree dimensions are
equal; hence, the box is a cube.

28. We compute the parti al derivative: of/ ox = an x "- I and a f /oy =


cn yn-l. Setting these
equal to 0, we see that the critical point is (0, 0) . The second partial derivatives are 0 2 f / ox 2 =
= =
n(n - 1)ax n - 2 , 0 2 f/o y2 n(n - 1) cyn-2 and 0 2 f/8 x 8 y O. We cannot apply t he second
derivative test since all of the second partial derivatives vanish at th origin. When n is even.
both x n and yn are positive if (x, y) :j:. (0,0) ; both x" and yn equal 0 only at th origin . We
concl ude th at t he origin is a maximum if (J and c are both negative and n is even . The origin
is a minimum if both a and c are positive and n is even . For all other cases, we have a saddle
point at t he origin since the origin is t he only critical point .

:31. We want to find the extrem a on the disk . F irst , check to see if an xtrema occurs inside t h
disk. Take the partials of f and set t hem equal to 0:

oj of
-
ox
= 2x + y = 0 and ay = 2y + x = o.
HI GHER-ORDER DERIVATIVES; MAXIMA AND MINIMA 51

= =
Thus (x , y) (0,0 ) is &. rela tive extrema and f(O, 0) 0. On the boundary, we let x cos B, =
Y = sin B. Then f( x , y) = f ( B) = 1 + sin Bcos B. Differentiation gives f' (B) 2
=
C05 B ­
sin B. Setting f' (B) = 0, we get cos 2 B = sin B, which means that B = 11'/4, 311' / 4, 511'/ 4,
2 2

or 711'/4 on t he interval ° ~ B ~ 211'. For B =


7r/ 4, f (x, y) =
! (1/ V2,1/V2) 1/ 2 + =
1/ 2 + 1/ 2 = 3/ 2; for B = 37r/ 4, ! (x,Y) = ! (- 1/ V2, 1/V2) =
1/2 - 1/2 + 1/2 1/ 2;=
for B = 511"/4, !('I: , y) = !( -1/ V2, - 1/ V2) =
1/ 2 + 1/2 + 1/ 2 = 3/ 2; and for B 71r/ 4, =
! (x, y)= ! (1/ V2, -1 /V2) = =
1/ 2 - 1/ 2 + 1/ 2 1/2. Therefore, the absolute maxima oc­
cur at (1/V2, 1/ V2) and (- 1/ V2, -1/ V2), with the maximum value of 3/ 2, and the absolute
minimu m occurs at (0 , 0), with the mini mum value of 0.

34. First, it is always wise to check for extrema inside the region . We set the part ial derivativ s
° = =
qual to 0: 8!/8x = Y = and 8!/8y x 0. Thus (0, 0) is a loc I extremum , and at (0,0) ,
! (x, y) = 0. However, we have not checked the points on the boundary. For he line segment
= =
y 1, we have -1 ~ x ~ 1. On this line segment !(x , y) 1 · x = x , so the minimum for this
line segment occurs at (- 1, 1) and the maximum occurs at (1,1). Similarly, we can analyze
the ot her three line segments. Therefore, at two of t he corners , namely (1,1) and (-1 , - 1),
we have ! (x , y) = 1. And at the other two corn rs, !(x, y) = -1. So (0 , 0) is not an absolute
extremum ; t he maxima are at (1 , 1) and (-1, -1 ) and the minima occur at (1, -1) and (- 1, 1).

°
39. Suppose \7 2 u = and u achieves its mini mum on D\ 8D. Let un (z, y) = u(x,y)-( l / n )e"' , then
\7 2u n = -( l /n) eX < 0. Thus, Un is strictly superharmonic, and by Exercise 37, can have a
minimum only on 8D, ay, a.t Pn = (Xn , Yn ). We have un (x n , Yn ) = U(XY' Yn ) - (l/ n) exp(xn ) ~
,Yn)- e1 /n, since all X n ~ 10n 8D. If (xo, Yo) isapointinD,then un( xO,Yo) > un (xn , Yn),
U(X n
or
e e
u(xo, Yo) - - > u(x n , Yn ) - - .
n n

Estimating the left-hand side upward (since el n > 0) , we have

e
u( xo, YO ) > u (Xn , Yn ) - - .
n
Since D is closed and bounded and u(x, y) is continuous, there must be a point q = (xoo, Yoo)
on 8D such th at the above inequality holds in an arbitrarily small neighborhood of q. Hence,
u(xo , Yo) ~ u (xoo , Yoo ) and u h as a minimum on the boundary 8D.
42 . First , notice that h = (y/ 2) tan 0 and d = (y/ 2) sec O. Thus, we want to maximize

A = xy + ( / 2)yh = xy + (y2 / 4) tan B.


We are given P = 2x + y + 2d = 2x + y + ysec B. Note that Pis
=
constant. Solving for x, we ge x (1/2)( P - y - Y sec B) and
our area function becomes A(y, B) = (P y - y2 - y2 sec B) / 2 +
x
(y2 tan B)/4. Taking the partial derivatives, we get

8Al oy = P/ 2 - y - y see B + (y / 2) tan 0


and
8A/8B = (_ y2/2 ) sec 0 tan 0 + (y2 / 4) sec 2 B = (y2 / 4) sec B( - 2 tan B + sec B).
Setting 8A/ 8B = 0, we get y = O ar 2tanB =
seeB, i.e ., sin O = 1/ 2, i.e., B =
11'/6 . The
solution y =° =
is impossible for this geometric problem. When 0 1r/ 6, we have sec e 2/-va =
= =
and tan B 1/-va, so 8AI8y PI2 - y - 2y/ -va + y/ 2-va. Setting 8A/ fJ y 0, we get =

y
P ( 2
= 2 1+ -va - 2-va
1 )-1- 2-vaP -va- 5.
Therefore, the maximum area is
52 CHAPTER

P y - y2 _ y2 sec e y2 tan e
2 + 4
where
P..j3 7r
y ;= 2V3 - 5 and B= 6'

3.4: CON STRAINED EXTREMA AND LAGRANGE MULTIPLIERS

G OALS
1. Be able to find the extrema if one or more constrai nts are given.

2. Be able to find minimum and maxi mum distances in geometric problems.


3. Be able to an lyze the critical points of a function with one constraint by using t he border _
Hessian .
4. For economics problems, be able to explai n the significance of A.

STUDY HIN TS
1. Notation. T he not ation fl S means "restricted to." For example, if g(x, y) = x + y, th
gl(x = 2) means we want to consider the fu nction 9 = 2 + y.
2. Method for fi nding constrained extrem a. Note that t here is an alternative method which ~
equations (3) rather than (2). First , rewrite the constraint so that the right-hand side .•
zero; for example, x + y =
2 becomes x + y - 2 =
O. Then consider the function h(x , >.) =
f( x) + >. . (constraint). Solve 8h / 8x; = 0 and 8h/ 8A = o. In example 2, w analyze h(x,>.):::;
x 2 - y 2 + >.(x 2 + y2 _ 1) and in example 3, h(x, A) = x + Z + A(X2 + y 2 + z2 - 1).

>.; only the values of tht:"


3. Solving equati ons. In general , we are not interested in the value of
variables. Sometimes, solving for A in terms of the variables, thus eliminating >., is the righ
thing to do.
4. Cautions. Remember , all of your equations must be solved simultaneously. Solving one equa­
tion alone does not complete the job of ,finding an ext remum.

5. Generalization. If there is more than one constraint, then "Y f (xo ) = >'1 "Y g1 (xo) + >'2 "Y g2(XO) ­
... + >'n "Ygn(XO)' The right-hand sides of equations (2) will have the form ti=1
>'i g;i, in plact'
J

of >.. : :' and there will be extra const raint equat ions . If you prefer to use equations (3), Ie
J
h(x , >.) = f (x) + I: Ajgj (X) and solve 8h/ 8xj = 0 and 8h/ 8>., = 0 simultaneously.
6. B ordet'ed Hessian. This is a Hessian with a "border ," which are the additional top row and
the additional 1 ft colu mn. T he entries from left to right or top to bottom are 0, - 8g/ axl
-8g/ 8X 2, ... , - 8g /8 xn , where 9 is t he const raint . Note that all entries except for the border
are second partial derivatives.
7 . Classifying critical pointsXo . (a) If the k X k subm atrices of the bordered Hessian are aL
negative for k ~ 3, then Xo is a local minimum.
(b) If the signs alternate: positive, negative, positive, .. . , starting with the 3 x 3 matrix, then
X o is a local maximum.
(c) If t he pattern does not satisfy (a) or (b) , and the submatrices are not all zero, then Xo is
at a saddle point .
HIGHER-ORD ER DERIVATIVES; MAXIMA AND MINIMA 53

8. Extrema on a region. T he method of Lagrange multipliers is only good for locating extrema
on a boundary. Don't forget to analyze the critical points inside the region.

9. Geometry. As in exam ple 8, section 3.3, we can analyze the square of the d istance rather th an
the distance itself.

10. Economics. Isoquants are curves showing all possible combinations of capital and labor which
produce the sam output. In t hese examples, ). tells you how much more can be produced with
one extra unit of capital or labor . Note that). has significance only a t the optim al point.

SOLUTIONS TO SELECTED EXERCISES


1. Use the method of Lagrange m ultipliers. We have \If(x, y, z) = (1 , - 1, 1), and the const raint
is g(x ,y,z) = x 2 + y2 + z2 - 2, so )'\lg(x , y, z ) = ).( 2x , 2y , 2z ). Thus, Vf = >'Vg gives
us 1 = )'2x , -1 = )'2y a nd 1 = )'2z. So we have x = z = - y = 1/2),. Su bstitute this
into the constr aint: {1/ 2>.)2 + (-1/2).)2 + (1/2).)2 = 3/ 4).2 = 2, or ). = ±(1/2) J3/2. For
). = + (1/2)J3/2, we have (x,y, z) = (J2/3, - J2/3,J2/3), and for)' = -(1/2)J3/2, we
have (x, y , z) = (- J2/3, .j2f3, - J2/3). These are the two extreme points and the maximum
is v'6, while the minimum value is - v'6.

3. We want to find t he extrema of f(x , y) = x subj ect to x 2 + 2y2 = 3. Use the method of
Lagrange multipliers. From the constraint , let g(x, y) = x 2 + 2y2 - 3, so V f = (1,0) and
Vg = (2x ,4y). We want t o sim ultaneously solve Vf = ).Vg and the constrai nt equation:

>.2x (1)
My (2)
3. (3)

From (2), we get y = O. From (1), x = 1/ 2), . Substituting for x a nd y in (3) gives us
(1/ 2).)2 = 3, so 1/2>. = ± J3; therefore, x = ± v'3. At (v'3, 0), f( x, y) = v'3 and at (-v'3, 0),
f (x , y) = - -/3. We concl ude that the m aximum occurs at (v'3,0) and the minimum is at
(- y'3, 0).

8. On S, y is restricted to be cos x , so f (x, y ) = x 2 _ y2 =


x 2 - cos 2 x. Applying one-variable methods , we cal­
culate f'( x ) = 2x + 2cos xsin:!.' . The derivative van­
ishes when x = - cos x sin x = - (1/ 2) sin 2x. This is a
transcendental equation and can be solved by graphi­
=
cal m ethods. The graphs of y x and y =- sin 2x / 2
only intersect at the origin , so (0 , 0) is an extremum.
Since x 2 ~ 0 and 0 ~ cos 2 X ~ 1, we conclude th at
(0,0) is a minimum.

13. We want to m inimize the surf ce area of th cylinder subject to t he constraint of the volume.
That is, we want to minimize S(r , h) = 2rrl' h + 2rrr2 subject to rrr 2 h = 1000 cm 3 . Use
the met hod of Lagrange multipliers. From the constraint, we get g(r, h) = rrr 2 h - 1000.
We compute th following first partial derivatives: 8S/ 8r = 2rrh + 4rrr , 8g /8r = 2rrrh,
8S/ 8h = 2rrr ) 8g/ 8h = rrr2 . Now we want to solve the fo llowing system of equations:

2rrh + 4rrr (1)


2rrr (2)
rr7· 2h 1000. (3)
54 CHAPTE R 3

Factor out rrr from (2) to get 2 = Ar or A = 2/r . Substitution into (1) and factoring out 2rr gives
h + 2r = (2/ r )rh = 2h , or h = 2r. Substitution into (3) gives rrr 2 h = rr7,2 . 2r = 2rrr 3 = 1000,
so r = 10/(2rr)1/3 and h = 2r = 20/(2rr)1/3. To check that our result sat.isfies the const.raint
we calculate
2 100 20
rrr h = Tt (2rr)2/ 3 (2rr)l/3 = 1000.

Therefore, the desired cylinder has height 20/(2rr)1/3 cm and b ase radius 10/(2rr)1/3 cm.

18. Use the method of Lagrange multipliers. Let

f(x , y, A) = x + 2y sec B + A( XY + y2 tan B- A).

Then of/ax = 1 + AY; of/oy = 2 sec B+ A(X+2y tan B) ; a nd OflaA = xy+ y2 tan B- A. From
=
af/ox 0, we get A = -1/y; whereas from of/oy 0, we get =
A= - 2sec B
x + 2y tan B
Hence , 2y = (x + 2y tan B)/(sec B) , so 2y(1 - sin B) = x cos B. Thus, x = 2y(sec B - tan B).
Substitute this into a fI OA = 0 to get 2y2(sec B-tan B)+y2 tan B = A. T hen y2 (2 sec B-tan B) =
A , so
y
2
= 2 sec BA- tan B
Acos ()
2 - sin B.
25 . (a) Use the method of Lagrange multipliers on t he auxiliary fu nction h(x, y, A) = x + y2 ­
A(2x2 + y2 -1). We compute the following partial derivative :

hx 1 - 4XA == 0 (1)
hy 2y - 2YA = 0 (2)
= h)., _(2x2 + y2 - 1) = O. (3)
From (2) , we get either y = 0 or A = 1.
For the case where A = 1, we get x = 1/4 from (1)
and then y = ±V778 from (3) . W hen y = 0, we get x = ±1/V2 fro m (3) . Thus , the critical
point.s for the constrained function a re located at (1/4, ±V778) and (±1/V2, 0) .
(b) We let the const raint be g( x , y) = 2x2 + y2 - 1, so h(x, y, A) = f(x , y) - Ag(X, V). Then
the bordered Hessian (T heorem 10) is

og og
0
ax oy
02h 0 - 4x -2y
og a2 h
/H/= ax ox 2 oyox
-4x - 4A 0
- 2y 0 2 - 2A
og 02h 02h
ay oxoy oy2

At (x, y, A) = (1/4, ..ft78, I), we find


0 -1 -j7fi
/H/= -1 -4 0 = 14> 0,
-Jr72 0 0

SO(01: , y) = (1/4 , V778) is a relative m axi mum point. At t he point (x, y, A) = (1/4, -V778,1).
we have
I 0 -1 Vf72
IHI = I 0~2 ~4 ~ = 14 > 0,
HIGHER-ORDER DERIVATIVES; MAXIMA A ND MINIMA 55

so (x, y) = (1/4, -ftl8) is also a relative maximum point. When (x, y) = (1/V2, 0) , equation
(1) tells us that>. = -Ji78, so at the point (x, y, >.) =
(1/V2, 0, - 078),
we see t hat

IH,I:o: -J8
° -J8
V2 ° = (2 + \1'2)( -8) < 0.
°

2 + V2
° °
Thus, (1/V2,0) is a relative minimum. Similarly, when (x,y) = (-1/V2,0), >. = +Jl78, so
at the point (x,y,>.) = (-1/V2, 0, Ji78), we get

IHI = J8
° J8 ° = (2 - V2)(-8) < 0 .
2 - °V2
-V2
° °
Thus, (-1/V2,0) is also a relative minimum. Evaluating the function f(x,y) at the critical
points tells us that (1/4,±ftl8) are absolute maxima, (-1/V2,0) is an absolute minimum,
and (1/V2, 0) is only a relative minimum.
26. With a hyperbola, there is only a minimum distance from a point. With a parabola, there
is also only a minimum. There can be no maximum distance from these geometric figures
because both figures extend to infinity.
31. Let the price of labor be p and let the price of capital be q. We want to optimize Q given the
constraint S::;;: pL + qK == B. We compute the partials of Q and S: {)Q/{)K = AaK,,-l L1-",
{)S/ {)K= q, {)Q/ {)L =
A(l - a)L -" K", {)S/ {)L = p. Use the method of Lagrange multipliers
to get the following system of equations:
AaK,,-l L 1-" >.q (1)
A(l- a)K" L-" >.p (2)
pL+ qK B. (3)
From (1 ), we get q = (A/>')aK"-lL 1-,,, and from (2), we get p = (A/>.)(l - a)K"L - ".
Substitution into (3) gives us (A/ >.) (l-a)K" L 1-" + (A/ >.)aK" L1-" = B or (A/ >.)K" L 1- " =
B or >. = (A/ B)K" L 1-". Substitute for >. in (1) and (2):

AaK,,-l L 1-" (A/ B)K" L i -"q (4)


A(l - a)K" L-" = (A/ B)K" L 1-"p. (5)

From (4), we get K = aB/q, and from (5), we get L = (1 - a)B/p. Thus, the point

(K, L) = (a: , (1 -pa)B)

optimizes the profit .

3.5: THE IMPLICIT FUNCTION THEOREM


GOALS
1. Be able to determine if an inverse function exists near a point x.
2. If an inverse exists, be able to find a derivative by implicit methods.

STUDY HINTS
1. Advanced material. The theorems presented in this section are usually proved in more advanced
courses . You should be most concerned with understanding the statements of the theorems .
- -- - - - - - - - -

56 CHAPTER J

2. Notation. DxF is used in th is section. It is just another notation for " F with respect to x
although as in Chap ter 2, DxF m ay be a m atrix if F is vector-valued .

3. Local Theorems. T he theorems introduced in this section may not apply if th range or domain
is too large.

4. Special implicit fu nction theorem. If o f / oz =F 0, then z can be wri tten in terms ofx at a given
point (xo, zo), and the derivative of z = g( x) is

_ - DxF(x, z)
D 9 (x ) - of .
a.;- (x, z)

Note that we can differentiate z even though we don 't have a formula for z. Don't forget the
minus sign in the derivat ive.

5 . Commonly used formu la . When z is a fu nction of x and y, we get t he formul a

dy oz/ox
dx - oz/oy'

It looks almost like division of fr actions, except fo r the minus sign . Again , don 't forget the
minus sign .

6. General implicit function theorem. In general , z m ay be a vector. We form a matrix with


the top row being the parti al of Fl wi th res pect to Z j, j =
1, .. . , m (alm ost like grad ient)
Similarly, the 0 her rows consist of th part ials of Fk, k =
I, ...m . If the determ inant of thi
m x m matrix is non-zero, then z is a function of x and a derivative exists.

7. Jacobian. T his is the de erminant of the m x m m atrix descri bed in item 6 above. It is denot d

8. Inverse functio n theorem. As stated in item 6 above , if J f (xo) =F 0, then z can be written in
terms of x. It may not be easy to express z in terms of x , but it is possible in principle.

g. Example 3. T his is a t ypical problem . St udy it carefully. Note that when more than one
function is given , you will n ed to solve a system of simultaneous equations to fin d a partial
deri vati ve.

SOLUTION S TO SELECTED EXERCISES


=
2. Let F( x, y , z) xy + z + 3xz 5 - 4. Since we want to know if we can solve for z as a funct ion
of (x, y), we need to know t hat o f /oz does not vanish near the desired point , so o f/oz =
1 + 15x z4. ear (1,0, I), Fz =16 =F 0, so F =
0 is solvable for z as a function of (x, y) .
Therefore,
oz - Fx + 3z 5
y and Oz _ - Fy _ x
ax Fz 1 + 15x z 4 oy Fz 1 + 15xz 4 .

At (x, y)= (1 , 0), z = 1, so oz/ox = - 3/ 16 and oz/oy = -1 /16.


7. Let Fl = y + x + uv and F2 = uxy + v. Then we want

~ = I OFI / OU
o Fdou
oFI/ ov
o F /ov
I =F 0 at (x, y , u, v) = (0, 0,0, 0).
2

The entries of the determinant are a FI/ou = v, aFi/ ov = u, o F 2 / o u = xy and oF2 /8v = 1.
We see that at (0,0,0 , 0),
HIGHER-ORDER DERI VATI VES; MAX IM A AND MINIMA

so we may not b able to sol ve for tI, v in terms of x , y near (x, y , u , v ) = (0,0 , 0,0) . To check
directly, the firs t equation gives us ltv = - (y + x ), so v = -( y + x)/u . Combining this with
the second equation , we get ux y = (x + y)/u, or 1.1 2 = (x + y)/ xy. For (x , y) near (0,0) , either
there is no solution for u small, or t here are 2 solutions for u.

10. (a) Using the definition of a(x , y)/ o(r , 0) and computing the part ial derivatives , we get

a( x, y) _l ax/or
8(r , O) - ay/or
ox/ ao
oy/oO
I_I cosO
- sinO
-r sinS [_
rcos O - r.

At ( 1'0 , ( 0 ) , r = 1'0 .
(b) By the inverse function t heorem, we can form a smooth inverse fu nction (r( x, y) , O(x,y))
as long as r I- 0. As a direct check, solve fo r r and 0 in terms of x and y: J
x 2 + y2 = r
and 0 = arctan(y/x) . Since we have written r and () in terms of x and y, the result above
is confirmed . Note t hat if x = =
0, then 0 7r/ 2 or 37r/ 2, depending on the sign of y. If, in
= =
addition, y 0, then r 0, a.nd B can have any value, so we cannot find an inverse, as we did
above.

12. Let F l = xy2+XZU +yv 2 -3 and F2 = u 3 yz+ 2x v- u 2 V 2 - 2. T hen oFt/f)u = xz , oFt/ov = 2yv,
oF2/ol.l = 3u 2 yz - 2uv 2, oF2/ov = 2x - 2u 2 v. At (x,y, z ) = (1,1,1 ) and (1.1, v) = (1, 1),

Since L\. I- 0, it is possible to solve for tJ, tJ in terms of x, y, z near the given point. To compute
ov/ ay, we use the chain rule:

and
OV AU 2
-oF2
oy
= 3u201.1 3
- yz + u z + 2x - -
oy oy
av 2
2u - v - 2v - u
oy oy
= O.
At (x , y, z) = (1,1, 1) and (u , v) = (1, I) , those equa tions become 2+ ou/ay + 1 +2(ov/oy) = 0
and 3(8u /oy) + 1 + 2(ov/oy) - 2(01l./oy) - 2(av/oy) = 0, or au/oy + 2(ov/oy) =
- 3 and
=
ou/oy - 1, so ov/8y -1. =

SOLUT IONS TO SELECTED R EV IEW EXERCISES FOR CHAPTER 3

2. (c) Compute the partials of l and set them equal to 0:

2x + 1
i = °
2xy + 4 y 3 = 0.
(1)
(2)

From (I ), y2 = - 2x (so x ~ 0). Substitute into (2) to get y(_y2 ) + 411 = y3 = or y = 0.


°
T hus, x = also and (0,0) is the critical point. T he reader should verify that the discriminant
°
=
Dis 0, so this test is inconclusive. However, l(x, y) x 2 + 2x y2 + y4. - x y2 = (x + y2 )2 - x y2,
so 1 (1: , y) > 0 for all x < 0, and for x positive, (z + y2 )2 < xy2 implies x 2 + y4 < _ z y2 but this
is impossible since Z2 + y4. is always positive and _ x y2 is always negative. We can conclude
that the minimum value of l( x, y) is 0 at (0, 0).

4. The Taylor expansion is


58 CH APTEP

where all of the partial derivatives are evaluated at (3': 0, Yo). We compute th partial derivati
and evaluate at (xo , yo) = (0,0) as follow:

f (x, y) e"Ycosx; f (O , O) = 1,
f ,, (x, y) ye"y cos x - eXY sin x ; fx (O,O) = 0,
fy(x , y) xe xy cos z; fy (0,0) = 0,
2 xy
f" x(x, y) y e cos x - 2ye"Y sin z - eXY cos z; Ix;c(O, 0) = - 1,
f"y (x, y) e"Y cos x + zye xy cos x - xe xy sin z; f;c y (0, 0) = 1,
fy y(x , y) x 2 exy cosx; f yy (O , O) = 0.

Thus, the second-order Taylor expansion is


Z2
f (x, y) =1- + x y.
2
7. Th critical points are tho e where the first parti al derivatives vanish . T hus, we need to so
of y1T cos (11'z ) = 0,
ax
of sin (11'x) = o.
oy
The equ ation si n(11'x) = 0 implies that x is an integer. Since the sine and cosine functions a
never 0 at the same angle, y11' cos (11'x ) = 0 implies that y = O. Thus, the critical points a
(n , 0) , where n is an integ r. Next, we calculate the second parti 1 derivatives. They are
02f
ox 2
82 f
11' cos (11'z ),
ozoy
f)2 f
= O.
oy2
At the cri tical points (n ,O), we have fxx (n, 0) = 0, f;cy (n, O) = (- 1)" 11' and fyy (n ,O ) =
Using T heorem 6 from Section 3.3 , we compute D = [I,,;c(n, O)][ly y{n , 0)] - (fxy(n , O)P =
o- (( _1) n11')2 = _ 11'2 . Since D < 0, we conclude that the points (n, 0) are saddle points.
11. Use the method of Lagrange multipliers with g(x,y) = x 2 + y2 = 1. From Vf = >'Vg and t
constraint eq uation, we get
og
8x>' = 2x>.,
og
8y>' = 2y>. ,
l.

If x and yare both nonzero, t hen the first two equations tell us that - sin (x 2 - y2 ) ar
sin (x 2 - y2) must both equal >., so >. must be 0; therefore, x 2 - y2 must be 0 also. Notice th
x 2 - y2 cannot be a multiple of 11' because x 2 + y2 =
1 means both IxI and Iyl must be less th
= =
or equal to 1. From x 2 - y2 0, Z2 + y2 1, Ixl ~ 1 and Iyl ~ 1, we get four critical poin t

If x = 0 and y # 0, then x 2 + y2 = 1 tells us that y = ± 1 and consequently, >. = ± sin (- l

Thus, (0, ±l) are criti al points . Simila.rly, when y = 0, z = ± 1 and>' = ± sin (l ). Therefor

- -------------------------------------------

HIGH ER-OR DER DER IVATIV ES; MAXIMA AND MINIMA 59

(±1,0) are also critical points. At the points (±,ff, ± Vt), f has the value 1. At (±1,0)
and (0 , ± l) , f has the value cos(I). Thus, f has a maximum value of 1 at four points and a
minimum value of cost 1) at four other points.

13. By the method of Lagrange multipliers, we get he foll owing system of equations:

y >.
x >.
x+y 1.

Substituti ng into the last quation gives us 2>' = 1, or >. = 1/2. Thus, x = 1/2 and y = 1/ 2.
However, in this case , it would have been much easier to solve for x and substitute to get
z = x(1 - x ) = x - x 2 . By one-variable calculus methods, x = 1/2 is the critical point, and
the maximu m value of z subject to x + y = 1 is 1/ 4.

16. (b) By the implicit function theorem ,

dy of/ 8x
dx - of/ 8y '

In this case , F( x, y) = xS - sin y + y4 - 4 = O. Thus,

18. (c) T he rectangul ar parallelpiped (box) is symmetri ,so if x is a coordinate of a point (x , y, z)


on the corner of the box, then the di mension corresponding to that coordinate has to be 2x.
W want to maximize V (x , y, z) = (2x )( 2y)(2z ) = 8xyz subject to x 2/ a2 + y2 /b 2 + z2/ e2 = 1.
Use the method of Lagrange multipliers:

oV/ 8x = 8yz 2x>./ a2 (1)


8V/o y = 8xz 2y>"/ b2 (2)
8V/ 8z = 8xy 2z>. /e 2 (3)
2
x /a 2
+ y2/h + z2 / eZ
2
1. (4)

Solve (1) for x: x = 4a 2 yz/>.. Substitute into (2): 8(4a Zyz / >.. )z = 2y>'/b 2, or z2 = >.2/ 16a2bz .
Substitute fo r x in (3): 8( 4a 2 yz / >.) y = 2z>.. / c2 , or y2 = >..2 / 16a2 e2 . Then

P lug these results into (4) to get

>.2 >.2 >.2


16a2 b2c2 + 16a:lb2c2 + 16a 2 b2 c2 = 1.

i.e ., xyz = abe/ 3v'3. T herefore, the maximum volume is


abe 8abe
V = 8xyz = 8 . 3V3 = 3V3 '
60 CHAPTE R

22 . First check for extrem a on the interior of the circle ofradius J2. We have f( x , y) = xy-y+ x - _
so set its partial derivatives equal to O. We have

-of
ax = y + 1 and
of = x - I ,
-
By
so the point (1 , - 1) is an extremum. Since (1) 2+(_1)2 = 2, this point is within the constrain
To find ot her extrema (if any), we use the method of Lagrange multipliers:
y +1 2x"\
x-I = 2y..\
x 2
+ y2 = 2.
First consider the case when ..\ = O. Equations (1) and (2) gi ve us (x, y) = (1, -1), which
had considered earlier.
If ..\ :j; 0, then (1) and (2) added together yields x + y = 2"\(x + y) or (2,,\ - 1)(x + y) =
T hus , either ..\ = 1/2 or x = -yo For x = - y, equation (3) gives us the points (1, -1) ~_
(- 1, 1). If >. = 1/2, then equation (1) reduces to x = Y + 1. Substit ution into (3) yiel
(y + 1) 2 + y2 = 2 or 2y2 + 2y - 1 = 0 or y = (-1 ± -/3) /2 . Therefore , two other possib
extrema occur at ((1 + V3)/2, (-1 + V3)/2) and ((1 - -/3)/2 , (-1 - -/3)/2). Evaluating at

e
of these possible extrema, we get

f +2v'3, - 1; v'3 ) -
2'
'

1 ­ -/3 -1 ­ v'3) 1
f ( 2 ' 2 2'
f( l , - 1) 0;
f(-I , I) -4 .
Thus, the maximum points are ((1 + v'3) /2 , (-1 + V3)/2) and ((1- -/3)/2, (-1- -/3)/2) wi
maximum vlaue of 1/2. T he minimum point i (-1,1) with minimum value of -4.
25. We use the implicit function theorem. We need to show that the determinant
of of
au ov
oe oc
au av
is not 0 near (x, y, 1.1, v) = (2, -1 , 2, 1). T he determinant is

- 31.12 2v
-4u 121.1 3
I = I --812 2
12
I= - 144 + 16:j; O.
I
Since the determinant is not 0, 1.1 and v exist as functions of x and y. To compute ou/ox, "
implicitly differentiate the given equations with respect to x. Keep in mind that 1.1 and v ~
functions of x and y. We get
201.1 OV
2x - 31.1 -
ox + 2v -ax 0
aU 3 0V
2y - 4u ax + 12v ax = O.
To make the calculation si m pler, we can plug in (x, y, 1.1, v) = (2, -1, 2,1). Then
au av
4 - 12 ax + 2 ax 0

-2 - 8-
au + 12- ov O.
ox ax
HIGHER-ORDER DERIVATIVES; MAXIMA AND MIN IM A 61

Solve this simple system of two equations by your favorite method. You should get au/ox =
13/32.
30. (a) Using the given formula, we write

s ;: f(m, b) = (1 - 1 . m - b)2 + (3 - 2m - b)2 + (3 - 4m - b)2


= 19 - (46m + 16b) + ((m + b)2 + (2m + b)2 + (4m + b)2).

The problem is to find m and b which minimize f(m, b), so we take derivatives:

of _ -46+2(m+b)+4(2m+b)+8(4m+b)
am
= -46 + 42m + 14b,

and

of -16 + 2(m + b) + 2(2m + b) + 2(4m + b)


ob
- -16+14m+6b.
Next, we set them equal to 0:

-46+42m+14b 0
-16+14m+6b O.
Solving this system of 2 equations, we get b = 1/2 and m = 13/14. The reader is encouraged
to verify that this is indeed a minimum point. T herefore, the best-fitting straight line to the
points (I, I), (2,3), (4,3) is Y = 13x/14 + 1/2, as shown in the graph below.

33. If y = mx + b is the best-fitting straight line, we must have, in particular,

Performing this differentiation, we get

"
-22)Yi - mXi - b) = 0,
.=1
which implies that the summation has to be 0, or the positive and negative deviations cancel.

TEST FOR CHAPTER 3


1. True of false. If false, explain why.

(a) If y is a differentiable function of x, z is a differentiable function of x and Y, and oz/oy i= 0,


then
dy oz/ox
dx - oz/oy'
62 CHAPTER 3

(b) The function g(w, x, y, z) = w + xyz + y2 has no critical p oints.


(c) For any f(x,y,z), we have o4 f loxo y 2 oz = o4flozoyoxoy.
(d) A fourt h -order T aylor series for f (x ) y, z ) w) = x 3 + y 4 - z2 + w - W z 2 is exactly the same
as f itself.
(e) Suppose D is a closed region on the xy plane. If f (x y) is cont inuou and has a minimllI!'
on D, t hen f (x , y) also has a m aximum on D.

2. Use Taylor 's theorem to calculate a fi rst-order and a second-order approxim ation for xy2z 3 a
th e point (1. 1, 2.03,0.98).

3. Consider the surface described by z = 5x 3 y - 2x y2. Discuss the concavity of the surface '~
cross-section in the plane y = 4.

4. F ind all of the critical poin ts of h(u, v, w , r) = v 3 + w 3 + u 2 + 7"2 - 3v - 12w + 4r - 8.

5. Let f be a function of x and yand of lax = 3x 2 yex


. Wh ich of t h following, if any, can Ix
aflay?
(a) x 3 -2y+cosy
(b) 3x 2 +y-8
(c) 3-e Y +x3
6. F ind the minimum and m aximum values of x + yon the cur ve where 2x 2 + y2 = l.
7. F ind the m in im um and m axim u m values of x 2 - x + y2 - Y on the followi ng sq uares and thei!
interiors:

(a) The square with vertices a t (0,0), (1,0), (1, 1) a nd (0,1), but not including the x or
axes.
(b) T he square with ver tices at (0, 0), (2,0), (2,2) and (0,2), bu t only t he border on the ~
a nd y axes is included in the region .

8. Let a particle move in a potential field in R 2 given by V( x, y) = x 2 + 4x y + y2 - 2x + 4y + E


Find the critical points of Vand classify t hem.

9. Consider the following system of equ ations :

x 2u+ yv + uv o
2
u + x v + yu k,

where k is a constant. If .T and y may be wri tten in terms of u a nd v, com pute o x l o v at th


given point and for the given k.

(a) (u, v, x, y) = (1 , -2, 0, 1); k = 2.


(b) (u, v, x, y) = (2 , 1,0, -2); k = -6.
10. Farmer Jones owns a golden goose farm and he wants to maximize p rod uc tion of gol den eggs
He knows that more geese will lay m ore eggs, but t oo m any geese will inhibit egg layin g as t h
geese instinctively will not overpopu late. On the other hand , too m any coyotes will prey upo
most of the golden geese and too fe w will cause the geese to overpopulate . Farmer Jones h&
determined that th e golden egg production is propor tional t o

E(g , c) = 10gexp( -O.lg) - 2cexp ( -0 .2c),

where 9 is the goose popul ation and c is t he coyote popu lation.

(a) F ind the critical poi nts of E.


(b) Classify the critical points of E.
63

4 VECTOR-VALUED FUNCTIONS

~.l: ACCELERATION AND NEWTON'S SECOND LAW


GOALS
1. Given a path, be able to compute the velocity vector , the acceleration vector and the speed at
a given point .
2. Be able to use Newton's second law and Kepler 's law.

STU DY HINTS
1. Velocity, acceleration and speed. Reca.ll that the velocity vector's components are the first
derivatives of the components of the path with respect to time. Speed is the length of the
velocity vector. The second derivatives make up the acceleration. Beware that t he derivative
of speed is not the accelerat ion . Note t hat speed is a scalar, while velocity and acceleration
are vectors.
2. Differentiation rules. No tice how the differentiation rules for paths are comparable to the rules
you learned in your one-variable calculus course.
3. Ne wton }~ second law. T his states that F = rna, where F is force and a is the acceleration . You
should remember this.
4. R egular path. If c'{t) 1= 0, t hen c{t) is a regular path and the image curve looks smooth.
5. K epler 's law. T his law relates an orbiting body's period to its radius. Vector alculus is used
to derive the equation. It is probably not necessary to remember the equation, but ask your
instructor to be sure.

SOLUTIONS T O SELECTED EXERCISES


2. T ake the first derivative of each component to get the veloci ty vector; that is, c'{t) == v{t) =
(t cos t + sin t , - t sin t + cos t, via). The acceleration is composed of the second derivatives, so
c"{t) = a(t ) = (- tsin t + 2cost , - tcost - 2sint,0 ). Therefore, v (O) = (O ,I,vIa), a (O) =
(2 ,0,0) and the tangent line is 1(>..) = c(O) + >"v(O) = 0 + >"(0, 1, via) = >"(0 , 1, via).
5. By the sum rule, we may add vector fu nctions first and th n differentiate , or we m ay d ifferen­
tiate first and then add the derivatives. By adding first,

! [Cl (t) + C2(t)] ~ (e t + e- t ,sint + cost , t 3 - 2t 3 )


(e t - e - t, cos t - sin t, - 3t 2 ) .
On the other hand,
d d d
dt [Cl (t) + C2(t)] dtc dt) + dt C2 (t)
(e t , cos t, 3t 2 ) + (_e- t , - sin t, - 6t 2 )
(e t - e- t , cost - sin t, -3t 2 ),
which is what we got by adding first and then differentiating.
64 CHAPTER

10. We wan F(O) = ma(O) , where m = 1. T he accelerat ion vector is given by a(t ) = r l/(t ) ­
(- cos t , - 4 sin 2t ), so a (O) = (-1 , 0) . Thus, F( O) = - i g-cm / s 2 = - O.OOli newton.
13. To show that the speed is constant, we differentiate it with respect to time and show that
deriva.tive is zero. Since speed is IIvll = .;v:v, it is more convenient to work with the squ
of the speed. (If th square of t he speed is constant, then the speed must be a constant, t
We calculate
d d dv
dt II v ll2 = dt (v· v) = 2v· dt = 2v . a,
but the acceleration a is perpendicular to the velocity Vi therefore v . a = O. Th
(d/dt)ll v (t )W = 0, so Il v (t)1I is constant.
17. Integrating each component of c'(t) = (t,e t ,t2) gives us c(t ) = (t 2/2+ A,e t + B,t 3/ 3+
where A, B and C are constants. When t = 0, we have e(O) = (A, 1 + B, C) = (0, - 5, 1),
so A = 0, B = -6 and C = 1. Therefore, the path is c(t ) = (t 2 / 2, et - 6 t 3 / 3 + 1).
19. (b) Let X = 2x, so X 2 + y2 = 1. Recogn izing this as a circle,
we let X = cost and y = sint, so x = X / 2 = (cost )/ 2. OUf
y
path e(t ) is described by (x,y) = ((cost )/2,sin t) . From the

original equation , we recogn ize 4x 2 + y 2 = 1 as an ellipse with

x-intercepts at ±1/2 and y-intercepts at ± 1.

1/2 x

20. By the cross product rule in the box preceding Example 1 of the text ,

d [c(t) x v(t )] = m [~
d [mc (t) x v(t)] = m dt
dt dt x v (t ) + e(t) x ~
dt ] .

But dc /dt = v(t), dv / dt = a(t) and v (t) x v (t) = 0, so the xpression reduces to

m[O + c(t ) x a(t )].


If k is a constant,
k (u x w) = ku x w = u x kw,
so
m [c(t ) x a (t)] = c(t ) x matt ) = c(t) x F(c(t )).
If F(e(t )) and c(t) are parallel, their ross product is 0, so the angul ar momentum i con
This is t he case of planetary motion; since
GmMc (t)
F(c (t)) = Il c(t)113 '
we see that F(c(t )) is a multiple of c(t), so it is parallel to c(t) .

4.2: ARC LENG TH


G OALS
1. Be abl to compute the arc length of a given segment of a path.

STUDY HINTS
1. No tation. Often 5 is used to denote a path in space, rather than c.
2. A rc length. T his is j ust t he length of a curv . Lengths may be added together, so we may
pute t he ar I ngth of curves which are not differentiable at finjtely many points by sum:
the lengths of the pieces .
VECTOR-VALUED FUNCTIONS 65

3. A rc length formula . You may find it easier to remember that arc length is the integral of speed
(n ot velocity !) . T his m akes sense because arc length gives the distance traveled . In any case ,
you should know that the for mula is

Itl
L(c) =
Itl

to
Ile'(t) 11 dt =
to
J (X'(t))2 + (y'(t))2 + (z'(t))2 dt.

4. Integration tricks.
(a) Due to t.he nature of the arc length formula , you m ay want to memorize the for mula

T his formula may be derived by trigonometric substitution if you do not wi h to memorize it .


Ask your instructor if this and similar form ulas will be provided on an exam .
(b) Look for perfect squares. Radicals can be elim inated from the integrand if a p r£ ct square
occurs.

5. Positive lengths only. If you compute a negati ve or a zero arc lengt h, then you m ade a mistake.
Arc length is always positive.

6. R iemann sum derivati on . If the derivation does not make sense now, you should return to this
discussion after R iemann sums are explained more thoroughly in C hapter 5. Understand ing
the derivation of fo rmulas gives much more insight into the t h ory.

7. Parametrization warning. If you need to parametrize a curve, be sure the curve is traversed
once and only once. T he orien ' at ion is also importan , and this will b come especially signif­
icant in chapter 7. T he bad consequences of an incorrect parametrization are illustrated in
example 1.

S OLUTIONS TO SELECTED EXERCISES

3. We compute e'(t) = (3cos3t, - 3sin 3t,3Vt), so


lie' (t) II = [(3 cos 3t) 2 + (-3 sin 3t )2 + (3vt) 2P/ 2 = ';9 + 9t = 3v'l+t.
T hus, the arc length is

1 1

o
3.Jf+t dt = 3 · -2 . (1 + t )3/2 11 = 2(23/2 -
3 0
1) = 4h - 2.

6. Given e(t) = (t , t sin t , t cos t) , we co mpute e' (t) = (1 , sin t +t cos t, cos t-t sin t). T hen lie' (t ) II =
=
[1 + (sin t + t cos t) 2 + (cos t - t sin t)2j1/2 ';2 + t 2. We want t.he arc length on the interval
[0 , 1t'], so we need to compu te:
fo re J2+t2 dt.

This m ay be integrated by he method of trigonometric substitut ion: One would let tan B =
V2t , aq,d t hen perform an integr ation involvingsec 3 8. This is left as an exercise.
An alternative
is to use the formula given in the integration tables (See study h int 4 above .):

i J2+t2
o
re
dt = '12 [tJi2+2 + 2ln(t + Ji2+2)] Ire0
~ [ 1t' J 1t'2 + 2+ 2 In(1i + ~) - 21nh] .
66 (HAPTE

9. Given e(t) = (2t , t 2 , logt), we compute c' (t) = (2, 2t, l/t) . Then
Il c'(t)11 = (4 + 4t 2 + l/t 2)1/2 = [(4t 2 + 4t4 + 1)/t2F/2 = (2t 2 + 1)/t = 2t + l/t.

Since c(l ) = (2,1,0) and c(2) = (4, 4, log2), we want the arc length on [1,2]' so we n ed
compute

1 2 ( 2t + ~) dt = (t 2
+ 10gt{ = 3 + log2 .

12. (a) Since T(t) . T(t) = II T (t)112 =


1, we can differenti ate both sides of T( t ) . T(t) l. By =
product rule fo r dot products (see the box preceding example 1 in section 4.1), (d/dt)( T
= =
T(t)) (d/dt )( l ), i.e., 2T (t) . T'(t) 0, which implies that T (t) . T ' (t ) = O.
(b) Beginning with T(t) = c'(t)/lle'(t)ll, we differentiate with respect to t, using the qUOl
rule:
'( ) = !!.. ( ~) = c"(t)ll c' (t)1I - c'(t)(d/dt)lle'(t)11
T t dt Ilc'(t) II lie' (t) 112 .
Recall that Ilc'(t )112 = c' (t) . c'(t), so

d
-lle'(t)11 = -d y'c'(t) . c'(t) = -1 (e' (t ) . e'(t))-1/2(2e' (t ) . e"(t)) = e'(t) . c"(t) .
dt dt 2 Ile'(t)11
Su bsti tution yields

, e" ( t ) , c' (t) . e" (t) , e" (t) II e' (t) 112 - (c' (t) . e" (t) )e' (t)
T (t) = lIe'(t )1I21Ie (i)11 - lI e'(t) 11 3 e (t) = lIe'(tlIl 3

17. (a) From the definitions of k and N in exercises 13(b) and 14,

dT
ds = T '() II' ( ) II T' ( s)
s = T s IIT'(s)11 = kN .

T he vectors T, N and B have un it length and form a right-handed system of mutually ort
onal vectors, so we have

T x N = B, N x B =T and B x T = N.
Differentiate , using the product rule fo r cross products , to get:

dN d dB dT dB dT
ds = ds (B x T) = ds x T +B x ds = ds xT - ds x B.

Using the fact that dB /ds = -TN (Exercise 15) along with the results derived earher i
exercise and then factoring out a constant from t he cross prod ucts, we get

-T( N x T) - k( N x B ) = -T( - B ) - kT = -kT + TB.


Finally,
dB d dT dN
ds
= ds (T x N) = ds x N + T x ds = kN x N + T x (- kT + TB)
= k N x N - kT x T + TT x B = -TN
since N x N = Tx T = o.
(b) Let w = WI T + W2N + W3 B. We shall use the results from part (a) to find scalars
and W3. We have

dT /ds = w x T = wt{ T x T) +w2(N x T) +w3(B x T)


= 0 - w2B +W3aN = kN .
VECTOR-VALUED FUNCTIONS 67

Here, we have used the facts that (1) B = T x Nand (2) T, Nand B form a right-handed
system of mutually orthogonal vectors, and so there exists scalars a and b such that B x T = aN
and N x B = bT. Thus,
=
W2 0 and aW3 k. =
Similarly,

dN/ds w xN = wdT x N) + w2(N x N) + w3(B x N)


w1B + 0 - w3bT = - k T + rB.

This gives us

Wl = r, bW3:; k and a = b.

Finally,

dB/ds w x B= Wl(T x B) + w2(N x B) + w3(B x B)


wl(T x B) + 0 + 0 = -TaN = -rN,
which gives a ::: 1, and this implies that b == 1 and W3 = k. Therefore,

w == rT+ kB.

4.3: VECTOR FIELDS

GOALS
1. Be able to sketch simple vector fields.

2. Understand the relationship between flow lines and a vector field.

STUDY HINTS
1. Vector field. This is a mapping from ]Rn to ]Rn . Note that the dimensions of the spaces are
equal. Each point x in the domain is assigned a vector. To depict a vector field, We draw the
assigned vector originating from the point x.

2. Scalar field. A scalar field differs from a vector field in that each point x in the domain is
assigned a scalar, not a vector. An example is the annual rainfall at each point on the earth's
surface. The wind velocity at any instant of time is an example of a vector field.

3. Gradient vs. vector field. All gradient fields are vector fields, but not all vector fields are
gradient fields. For example, the vector field i + xj is not a gradient field because you cannot
find an f such that of/ox = 1 and of/ oy = x. If a vector field Pi + Qj is a gradient field
then oP/ 8y = 8Q / ox. This statement comes from the fact that {j2 f / oxoy = 0 2 f / oyox for a
well-behaved function f.

4. Flow lines. This is the path a par ticle would take if it was free to move along the vectors in
the field. T hinking of the vector field as velocity, the flow lines would show displacement . A
formula description of a flow line can be obtained by integrating each component of a vector
field (or solving a system of differential equations). Flow lines c(t) must satisfy the equation
c'(t) = F(c(t)).
68 CHAPTER

SOLUTIO N S TO SELECTED EXE RC ISES

1. At each point (x, y) , we sketch the vector (2, 2) oriO"i­


nating from (x, y). Alternatively, we may draw a small
multiple of the vector field. T he orientation of the /' /' /' /' /'
vectors must be maintained at 45 0 fro m the positive x /' /' /' /'
axis , and all of t he vectors must have the same ma.g­
nitude. /' /' /' /'

/' /' /' /'


/' /' /' /' /'

5. At each point (x , y), we sketch the vector (2y, x ) orig­

inating from (x , y). T he magnitude of the vectors in­ y

creases as (x, y) moves away from the origin .

10 . At each point (x , y), draw a li ttle arrow in the direction (x , - y ), then connect the little arr
to ge t flow lines. Al tern atively, one can solve the system of differential equations dxldt =
dYI dt = -y and write y in terms of x, then plot th flow lines y =
C I x fo r various consta
C. Our compu ter-generated sket.ch is shown below.

13. If c(t) is a flow line of F , then c' (t) = F(c (t)) . The left-hand side is c' (t ) = (2e 2t , l i t , -lil­
t f::. O. On the right-hand side , we have F( c(t)) = (2e2t , lit, - 1It 2 ) since x = e2t and z =
We got the same result for bo th sides of t he equation, so c(t ) is a flow line of F .
VECTO R-VALU ED FUNCTIO NS 69

18. From the ch ain rule, we have

dV(c(t)) = \7V(c(t)) . c'(t).


dt
We are given that c(t) is a flow line of F = -\7V, so c'(t) = F == -\7V and

dV~~(t)) = -c'(t) . c'(t) = -llc/(t)112 :::; 0.

Thus, the derivative of V :::; 0, which shows that V is a decreasing function of t.

20 . First , we compute

At each point (x,y), we draw the vector -\7V. Notice that far from the origin, the denom ina­
tors of - \7V, (x 2+y2) 2, will be much larger than t he numerators . Thus , the magnitude of - \7V
is very sm all at points far from the origin . A few computations reveal that - \7V is very large in
m agnitude near the origin . For example, - \7 V(0.5, 0) == (4, -4) and -\7V(O. l , 0.1) = (50,50).
A sketch of - \7V is shown below at the left.
To sketch the equipotential surface V( x , y) = (x + y)/(x 2 + y2 ) =
1, we rearrange and
i)
complete the squares to get (x 2 - x + + (y2 - Y + (x - i) =
+ (y - t)2
t)2 = t.Therefore,
the equ ipotential surface V(x , y) = 1 is a circle of radius V"f, centered a t (~, t). This is shown
below at the right .

y
, 2
,
/
/
-2 -1 0 2
x x

-2 - ,

4.4: DIVERGENC E AN D CURL


GOALS

1. Given any vector field , be able to compute its divergence.

2. Given any vector field in 1R 3 , be able to compute its curl.

3. Be able to explain the physical significance of the divergence and the curl.

4. Be able to manipulate expressions involving the cross product, the dot product and the del
operator .
70 CHAPTER 4

STU DY HINTS

1. T he operation \1 . This operator , called "del," tells you to assem ble the vector of partial
deri vat ives: (%x, %y, % z).

2. Divergence. Note th at the divergence is a scalar , not a vector. Know the t wo notations:
\1 . F = di v F . You should know t hat the divergence is a rate of expansion or, if it is negative,
compression. T he term incompressible means that div F = O.

3. Curl. Note that the curl is a vector , not a scalar. Know the two not ations: \1 x F ::: curl F.
You should know t hat the curl is associated with rotations and that the term irrotational
means that curl F = O.

4. Valid space f or curl. Note t hat curl is a property of ]R3. We do not attempt to take the curl
of vectors in dimensions higher t han t hree. T he two-dimension al vector xi + xyj is taken to
mean xi + xyj + Ok if a curl is desired .

5 . Laplacian. The expression \1 2 f means \1 . (\1 f) , which is a scalar.

=
6. Theorem s. T he facts t hat \1 x (\1 f ) 0 and V . (\1 x F) =
0 are useful ; it is nice to commit
t hese to mem ory. If you need to know one of t hese facts and you forget t hem, you can always
do t he comp utation .

7. Formulas in JR3. Note that t he cross product or curl occurs in some of the basic identities of
vector analysis in the t able preceding exam ple 15; therefore, it is assumed that the formulas
are used in ]R 3 and not in higher dimensions .

8. Basic iden tities of vector analysis. T he for mulas in the table preced ing Example 15 are useful
for developing the theory of vectors; however, you should not memorize the t able. You can
refer back to the table as needed . It will become obvious which formulas are most important
as you refer back t o them frequently.

g. Exercise 30. These fo rmulas are referred to quite frequently in the examples. You should do
this exercise even if it is not assigned .

SOLUTIONS TO SELECTED EXERCISES

2. T he divergence is \1 . V = tx (yz ) + :y (x z ) + : )x y) = O.
= F(c(t)) .
7. Recall t hat flow lin es are defined by c' (t)
In this case , we have c' (t) = (dx / dt,dy/ dt) = (y, O) .
t

y
ince dy / dt = 0 , we know y is some const ant, k l .

Then dx/dt = y = k l , so X = kI t + k 2 , where kl


and k2 are constants. Thus, the flow lines have the
.=r=; - •...
1

form (kIt + k2 , kJ). If kl is positive, flu id flo ws from

left to right as t increases , and if kl is negative , fluid


--..
flows from right to left, as shown in t he sketch. We ....
x
com pute the divergence: \1 . F = 8(y)/8x +8(O) /8 y =

O. The sketch shows that flu id appears to be neither


expandin g nor contracting , which is consistent with
our calculations.
­
....
....
...
11. The divergence is

\1. F = :x (sin (xy)) - : y (cos( x 2 y)) = ycos(xy) + x 2 sin(x 2 y).


VECTOR-VALUED FUNCTIONS 71

14. The curl is


i j k
\7x F
() a 0
= i (~(Xy) - ~(xz)) - j (~(Xy) - ~(yz))
ax oy oz oy oz ax oz
yz xz XY

+ k (:x (xz) - :y (Yz) )


(x - x)i - (y - y)j + (z - z )k = O.
17. The scalar curl is the coefficient of k when \7 x F is computed for a vector field F in lR 2. Here,
we have
i j k
\7 x F =
a a a = [:x (cos x)- ; y (sinx) ] k = (-sinx)k.
ox oy oz

Sill X cosx o

Therefore, the scalar curl is - sin x.

22. First, we compute that \7 J = (y + z)i + (x + z)j + (y + x)k. Then

i J k
\7 x \7J
o o o
ax oy az
y+z x +z y +x

[ ~(y
oy + x) - ~(x
oz
+ Z)] i - [~(y
ox .
+ x) - ~(y
oz
+ z)] j

+ [:x (x+ z)- ;y (y + z)] k


(1 - 1) i - (1 - l)j + (1 - 1) k = o.

25. We know that the curl of a gradient is the zero vector. Thus, it suffices to show that \7 x F i= O.
We have
i j k
000
\7 x F = = (0 - O)i + (0 - O)j + (sin y - cos x)k i= o.
ax oy oz
y cos x x sin y 0

28. We refer to the table of basic identities of vector analysis, which precedes Example 15.
(a) From identity (5), div (F + G) = div F + di v G = 'V . F + \7 . G, which sums to zero,
according to the given hypot hesis . Therefore F + G does have zero divergence.
(b) From identity (8), div (F x G) = G· curl F - F·curl G, which , in general, does not equal
O. As a simple example, let F = xyi - z yk and G = i. We have \7 . F = \7 . G = 0 and
div (F x G) = z.

30. (a) We have \7(l/r) = \7(1/ Jx 2 + y2 + z2) . Begin by findi ng (8/ox)(1/r) or

ox
a (
Jx2
1 )
+ y2 + z2
-2x -x-x
= 2( x 2 + y2 + Z2)3/2 = (x 2 + y2 + z2)3/2 = -;:J'

By symmetry, (%y)(l/r) = _ y/ r 3 and ({)j oz)(l /r) = -z/r 3 • Putting these together,
V'{l/r) = -(xi + yj + zk)/r3 = - r / r 3 . In general,

\7 (rn ) = \7((x 2 + y2 + z2)n/2)


72 CHAPTER 4

and

(O /O X)( X2 + y2 + Z2r/2) = (n/ 2)(x 2 + y2 + Z2)n/2-1)2x = nx1'n - 2,

so by symmetry,

\7 (1'n) = nrn - 2(xi + yj + zk ) = n1' n - 2r.

Finally,

\7(logr) = \7(log( J x2 + y2 + z2)) ,

and
a
-(log( Jx 2 + y2 + z2 )) =
1 a
. - ( J x2 + y2 + z2) =
x
2'
ox J x 2+y2 +z2 ox l'

so by sym metry, \7(logr) = (xi + yj + zk)/1' 2 = r/1'2 .

(b) Using the results of part (a),

\7 2 (1/1') = \7. \7 (l / r ) = \7. (-r / 1'3 )

= - [: x C JX 2 +:2+ Z2 )3 ) +:y CJX2+~2+ Z2)3)


+ :z C + ~2 +
J x2 z2) 3) ].

The first partial derivative is

x (x 2 + y2+ z2)3/2 _ ~Jx 2 + y2 + z2 .2x


:x Cx2 + y2 + z2).3/2) (x 2 + y2 + z2 )3
x + y2 + z2 _ 3x 2
2
(x2 + y2 + z2)5/2 .

By symmetry,

a( y ) x 2 + y2 + z2 _ 3y2

oy (x 2 + y2 + z2)3/2 = (x2 + y2+ z2)5 / 2

and

a( z ) x 2 + y2 + z2 - 3z 2

OZ (x 2 + y2 + z2 )3/2 = (x2 + y2 + z2)5/2 .

Then

\7. (-r)
1'3
= 3(x 2 + y2 + z2) - 3(x 2 + y2 + z2 ) = O.

(x2 + y2 + z 2)5/2

Simil arly, part (a) tells us that in the general case,

\7 2 (1'n) = \7 . \7(1'n) = \7 . (nr n - 2r) .

We compute

: x « x 2 + y2 + z2) n/ 2-1 . x ) = n [( x 2 + y2 + z 2 t/ 2 - 1 + 2x2 (~ _ 1) (x 2 + y2 + z2) n/ 2-2] .


Again, by sy mmetry,

\7 . (n1' n - 2r) n [3(x2 + y2 + z 2t/ 2- 1 + (x 2 + y2 + z2)(n _ 2)(x 2 + y2 + z2r/ 2-2j


n1'n- z(3 + n - 2) = n(n + 1)1'n-2.

(c) The identity \7 . (r/r 3) = 0 follows immediately from part (b) since n = -1 in that case.
For the general case , use part (b) again to compute \7 . (1'''r). Note that we only need to divide
VECTOR-VALUED FUNCTIONS 73

the general result by n and ch ange n - 2 to k (Why?) . T hen \7 . (rkr ) = (k + 3)rk = (n + 3)rn.
(d ) By a dired computation, we get

i J k
0 0 8
'\7 x r = = 0.
8x 8y az
x y z

Using the fact that the urI of a gradient is the zero vector, the calculation j ust completed,
and t he general case in part (a), we get

\l x (rnr ) = rn ('\7 x r ) + \l(rn ) x r = 0 + m· n - 2 (r x r ) = o.

32 . (a)

1 j k
curl F = \l x F
o 8 o
ox2 oy oz
3x y x3 + y3 o
k(3x 2 - 3x 2 ) = o.
(b) Note that if F = \l I, then 3x 2 y = (%x) /(x, y) and x 3 + 11 = (% y)/(x, y). Integrate
each equation:
f( x, y) = J 3x 2 y dx = x 3 y+g(y),

where 9 is a fu nction of y only, and

where h is a function of x only. In bo th cases, I (x , y) must be the same , so compare both


sides. If we let g(y) = y4 / 4 and h(x) be an arbitrary constant , then f(x,y) = x 3 y + y4 / 4 + C
satisfies \l f = F.

33. (c) Let F( x, y) = e'" cos yi - eXsin yj , where the j component is the real part of ex - iy and th j
component is the imaginary part . We calculate div F = (0/ ox)( eX cos y) - (%y)( eXsin y) =
eX cos y - eX cos y = 0 and

i j k
curl F
o o a
ox oy 8z
eX cos y -e'" sin y 0
k( - ex sin y + e" sin y) = o.

Thus, F is both incompressi bl and irrota tional.

SOLUTIONS TO SELECTED REVIEW E XERCISE S FOR CHAPTER 4

2. The velocity vector is vet) = e'(t) = 2ti + (- 2t sin(t ))j + 4t 3 k. The acceleration vector is
2

aCt) = e" (t ) = 2i + [-2sin(t ) -2


4t cos(t )lj + 12t k. When t = ."fi, we have v( y'ir) =
2 2 2

2y'1ri + 47r."fik and a(y'ir) = 2i + 47rj + 127rk. The speed is the length of the velocity vector.
When t = ."fi, the speed is [47r + 1611"3]1/ 2 = 2-/11" + 4~ . The equation of the tangent lin IS
=
given by l(t) e(y'ir) + tv (y'ir) = (r. - 1, - 1, r. 2 ) + t(2."fi, 0, 4r.y'ir).

7. We use t he equation F = rna = me". Here, e" = (2, - sin t, - cos t) . At t = 0, a (O)
(2,0, -1), so F(O) = rn(2, 0, - 1).
_• ....a. _____ .

74 CHAPTER 4

9_ From x 2 = .; = Zll , we get y = X 2 / 3 and z = X 2 / 5 . Let x = t, so the path of the curve is


c(t) = (t, t 2/ 3, t 2/ 5). Here, we have 1 ~ t ~ 4 and c' (t ) = (1 , it-1/3, i t- 3/S). The arc length is

1 1
4I1C'(t)11 dt = t VI + i9t
11
-2/3 + i. t -6/ 5 dt.
25
13. We set the given equation equal to t and solve for x, y and z: x- I = 2y + 1 = 3z + 2 = t.
From x -I = t, we get r = t + 1. Similarly, 2y + 1 = t yields y = (t - 1)/2 and 3z + 2 = t
=
yields z (t - 2)/3. Therefore, we get (.7:, y, z) (t + 1, (t - 1)/2, (t - 2)/3) . =
15. We want to show that c'(t) = F (c(t) ). Here, x = 1/(1 - t) , Y = 0 and z = et /(1 - t). We ~et
dx / dt = 1/(1- t) 2, dy / dt = 0 and dz/ dt = (2e t - t et )/(l- t)2. T hus, dx / dt = .:z:2 , dy/ dt = 0
and dz /dt = z (l + x) = let /( 1- t )][l + 1/(1- t )l = (2e t - tet) / (l - t )2.
18. We compute div F = \l . F = fx( .:z:2) + /y (y2 ) + f. (z2 ) = 2x + 2y + 2z . Also,

i j k
888
curl F = \l x F = I - - ­
f}.:z: 8u 8z
.:z:2 y2 z2

= [-aya( 2 -8za( y2)] '


Z ) - 1- - (z 2) - -8 (x 2].
[8
8x f}z
) J + [-{} (y2) - -8(r 2]
8x
) k = o.
ay

21. The divergence is \l - F = f}y/ f}x + az/ 8y + 8x / 8z = O. The curl is


i J k
a 8 8 = ({}X_ {}Z ) i + ( f}y _ {}x ) j + (8Z _ 8Y ) k = -i-j -k.
\l x F =I {}x
ay 8z 8y {}z {}Z ax (}x 8y
y z x

25. We compute

\lJ = ( ~~ ) i + ( ~~ ) j + ( ~~ ) k = [2.:z: exp(z2 ) + y2 sin (zy2)]i + [2xy sin(xy2 )li + Ok.


Next, we compute
I J k
{} 8 8
\l x \lJ = 8x ay 8z
2x exp(x 2 ) + y2 sin(xy2 ) 2xysin(zy2) 0

[:Y (0) - :z (2xYSin( X y2))] i + [:z(2x exp (z2 ) + y2 sin(xy2 )) - : z (0)] j

+ [:x(2x y sin (x y2)) - : y (2xex p (x 2 ) + y2Sin(Xy2 ))] k


= (0 - O)i + (0 - O)j + [2y sin(xy2 ) + 2x y3 cos (xy2 ) - 2y sin (xy2) - 2xy3 cos(xy2 )] k
= o.
T hus, for J (x, y) = exp (x 2
) - Cos(xy2 ), we have \l x \l J = O.
28 . (b) If such J exists, then we know that F = \l1 = ({}f/{}x )i + (al/ ay )j + (8f/8z)k . Thue ,
we have {} 1/ 8x = 2xye Z , 01/ oy = eZ x 2 and 81/ oz = x2ye~ + z2. Integrating aI/ox = 2xye Z
=
with respect to x gives us I(x, y, z ) x2ye~ +A(y, z ), where A is some function involving yand
Z only. (You can check this result by computing (}f/o x. ) Similarly, integrating (}f/ ay with
respect to y gives us l (x, !I , z) = z2ye~ + B (x, z ), where B is some function involving x and z
only. Next, in tegrating oj/ 8z with respect to z gives us I( x , y, z ) = z2ye z + z3/ 3 + D(x , y),
where D is some function involving x and y only. Comparing the three resulte for I(z,y , z ),
we see that A(y, z ) = z3 / 3 + C, B(x , z ) = z3 / 3 + C and D(x , y) = C, where C is an arbitrary
constant. Therefore, f (x, !I, z ) = z2ye~ + z3 / 3 + C.
VE CTOR-VALUED FUNCTIO NS 75

32. (a) At the intersection, we know that y = 1, so t he intersecting curve has the equation
x 2+( 1)2+z2 = ;3, or x 2+z 2 = 2. Eq uivalently, we have x 2/2+z 2 /2 = 1, or (x/V2)2+(z/V2)2 =
1. Since we know that cos 2 t+sin 2 t = 1, we get x/V2 = cost and z/V2 = sin t, or x = V2cost
and z = V2 sin t. To get one revolution of the intersecting curve, we let t vary from 0 to
2r.. Thus, a parametrization of the in t ersecting curve is x = V2 cost, Y 1, z V2 sint, = =
o < t < 2r..
(b) From Sect ion 2.4, recall that an equation for a tangent line is l(t ) = c(to) + (t - to)c'(to).
Here, c(to) = (1, 1,1), where c(t) is the param etriza tion given in part (a). We want
(V2 cos to, 1, V2 sin to) = (1, 1, 1), so t = r./ 4. Differentiating each component of the
parametrizat ion, we get cl(t) = (-V2 sin t,O,V2 cost) . T hus, the tangent line is

l(t) = (1,1,1) + (t - r./4)( - 1, 0,1).

(c) In par t (b), we computed c'(t), so Illc'(t)11 = V2. Thus, the arc length is

f 2rr
io Ilc'(t)11 dt = 1

0
2rr

h dt = 2hr..
As expected, this is the circumference of a circle of radius V2.

36. (a) T he direction of the rotation vector w is the same as the axis of rotation, in this case, the
positive z axis, or k direction. vVe're given the magnitude as 4, so w = 4k.
,( b) When r = !5v'2 ~ i ­ j), the velocity is

j k
v =w x r = 0 0 4 = 20hi ­ 20hj .
5V2 -5V2 0

(c) The vector r is the vector from the axis to the point , so r is the vector from (0,0,5) to
(0, 5V3, 5), or r = .5V3j. For the point (0, .5V3, .5), we get

J k
v =w x r = 0 0 4 = -20V3i.
o .5V3 0

T EST FOR CHAPTER 4

1. True or false. If false, explain why.

(a) The curl of any vector field in ]R3 is another vector field .
(b) One revolution of the path c(t) = (cos2t,sin2t) has arc lengt h f;rr Ilc'(t)11 dt.
(c) If band c are vector-valued func t ions of t, then d(b x c)/dt = db/dt x c + dc/ dt x b.
(d) Two particles which have t he same acceleration must travel at the same speed.
(e) Suppose G(x, y) is the velocity field of a gas. If G(x, y) = xi, then the gas is expanding
and if G(x, y) = yi, then the gas is not expanding .

2. Let c(t) = (2t, t - 3, t 2 + 1) be a possible flow line for a velocity vector field . W hich of the
following, if any, could be such a velocity vector field?

(a) F(x, y, z) = (2, x - 2y + 7, x).


(b) F(x,y,z)=(yiz -l-y+5,1, 2y -6).

3. Suppose the velocity vector fields represent the flow of a fl uid. At what poi nts in ]R 3, if any,
does the flui d lack rotation?
-'---­ - --
76 CHAPTER 4

(a) F( x, y, z) = 4x yi + 2x 2j + k.
(b) F(x , y, z ) = xyi + zj + xk.
4. A particle travels counterclockwise along the ellipse 9x 2 + 4y2 = 36.
(a) Express the distance traveled going from (2,0) to (0, -3) as an integral of the form
va
J + b cos 2 t dt for constants a and b.
(b) Suppose t he part icle had flown off on a tangent line at (-l ,3 V3/2) . Where will the
particle hi t the x-axis?

5. For a given function , g(t), let a particle's posi tion be described by 2g(t)i + [g(t)j2j - k.
(a) What force is acting on the particle if its mass is 3 units?
(b) Wh at conditions must be imposed on g(t) for the force to be O?

6. Let F(x , y, z) = xi + yj + (z2 + y) k be the velocity vector field for a gas. Where , in ~3, is the
gas expanding?

7. In R 4 , a path is described by y = 2x, z = 4x 2 - 6 and w =e Z


• Find the arc length of the path
as an integral in y for 0 ~ x ~ 2. Do not evaluate.

8. Which of the following, if any, is the curl of some vector field G such that F = curl G?
(a) F(x, y, z ) = 2i + 3j - 2k.
(b) F (x, y,z ) =xi + 2j - zk.
(c) F (x , y,z )= yi + yj + x 2 k.
9. (a) Let v(t) = (sin 2 t)i + (t 4 - t 3 )j + 3k. What is t he deri vative of v( x 3 - x2 + 3x - 4) with
respect t o x?
(b) Let c(t) be a path in ]R 3 and let p(t) be a scalar function. Find a formula for the
acceleration of p(t )c(t) .

10. At the annual camel races, camel A 's position is given by (cos 5t, 2 sin 5t) and camel B's position
is given by (cos 4t , 2 sin 4t).

(a) Show that the camels are running on the same path .
(b) Suppose camel A 's mass is 500 . What force is being generated by camel A at t = 7r/5?
(c) Camel B saw an oasis at t = 7r/ 5. It ran off on a tangent line. What is the equation of
the tangent line?
(d) Suppose camel B had continued to run on the track. How far behind (in distance) was
camel B when camel A had com pleted one lap? Leave your answer in the form of an
integral.
77

5 DOUBLE AND TRIPLE INTEGRALS

5.1: INTRODUCTION
GOALS

1. Be able to calculate double integrals over rectangles.

2. Be able to use Cavalieri' principle.

STUDY HINTS

1. Some notation. (a) The cartesian product of two intervals in lR 2 is a rectangle. If a ~ x ~ b


and c ~ y ~ d, then it is denoted [a, b] x [c, d].
(b) Sometimes dx dy or dy dx is abbreviated dA, the "differential of area."

2. Review. Before continuing, you should re iew integration techniques for one variable. It is
essential that you remember how to integrate by parts and by subst itution .

3. Geometric interpretation. If f(x, y) ~ 0, then the double int gral lID


f( x, y) dA is the volume
=
under the surface defined by the graph of z f (x, y). Recall that with one variable, f(x) dx I
is the area under the curve y = f(x).

4. Computing a double integral. Just as with partial d rivatives , all but one variable is held
constant in each step. We integra te from the inside and work our way to the outside. For
example,
bi df(x, y)dy dx = Ja,b [F(x ,d)-F(x,c)]dx,
ia c

where Fis an antiderivative of fwhen x is held constant. Now , we com pute the second x-integral
using one-variable methods.

5. Cavalieri's principle. This principle is used in most one-variable calculus courses to derive
volume formulas. These fo rmul as are often referred to by names: disk or slice method.

6. Riemann sums. Be aware that in the sum m ation 2:7:=0 f(Ci)( Xi +! - Xi ), ci can be chosen
anywhere in [Xi , xi+d . In a Riemann sum, we take the limi t as n -+ 00, so in most cases, ! (c;)
is almost independent of Ci because X i +! and Xi eventually get close together.

SOLUTIO NS TO SELECTED EXERCISES

1. (b) First hold x constant and integr ate with respect to y, then integrate with respect to x:

ior/ Jo
t (ycos x +2)dy dx
2

r/ (12'
io
2

cos x+ 2) dx = ( 2'1 sin x + 2x ) 17f/2


0 = '12 + 'fr.
78 CHAPTER 5

2. (b) We can change t he order of integration . Thus , we hold y constant and integrate with
respect to x firs t:

r/ (y cosx + 2) dx dy
Jor Jo
i 2

Jot [(y sin x + 2x ) x=O dy 1"'/2 ]

1\y + dy= (y; + 7r I: = ~ +


7r) Y) 7r.

As expected, we get the sa me answer regardless of the order of integration.

3. By Cavalieri 's principle , the volume of a solid is


b
V = 1 a A(x ) dx, •

where A (x ) is the cross-sectional area cut out by a plane. Note that at the same height, a
cross-section of the left-hand side is a circle of radius 7' and a cross-section of the right-hand
side is also a circle of radius r . Since A (x ) is equal in both cases, the volumes must be equal.

5. With the setup in figure 5. 1.12, we slice W vertically by planes to produce triangles Rx of
area A(x ) in the figure. T he base b of the triangle is b = .Jr2 - x 2 and its height is given by
h = b tan (j = .Jr2 - x 2 tan (j, Thus A(x ) = ~bh = ~ (r2 - x 2) t an (j . Hence, the volume is

j r
_rA(x) dx = jT 21 (r
_r
2 2 1
-x )tan(jdx= 2(tan(j) ( r 2 x -'3
x )
3
I
r
-r

3 3
21 t an (j ( 2r 3 - 32r )
= 32r tan (j.

8. Note that when y is in the interval [-1 ,0], then Iyl = -y, and so
2
Jl (Iyl cos ~x ) dydx = Jor jO-1 (-YCOS 7r4X) dy dx
2 O
r [(_y2 7rX)I
Jo -2- cos 4" y=-l 1dx
(1 7rx) dx = -2. 7rX 2
1 o
2
- cos -
2 4
I
7r
sm -
4
12
0 7r

10. Since f( x, y) ~ 0 for all points in R, ffR f( x, y) dy dx is the desired volume. It is

1 2
1 \1+ 2x + 3Y)dY dx = 12 [(Y+ 2X Y + 3f )I~=J dx
12 (2X+~) dx= (x2+ 52X) I: 11
2

5. 2: THE D OUBLE INTEG RAL OVER A R ECTA N GLE


G OALS
1. Be able to com pute a double integral over a rectangular region.
2. Understand Fubini's theorem .
DOUBLE AND TRIPLE INTEG RALS 79

STU DY HINTS
1. Definition. The definition of the integral is more importa nt fo r theoretical rather than com­
putational work . It is defined to be a limit of Riemann sums:

J1 R
! (x, y) dA = nl~n;,
n -1
L
j ,k=O
f ( Cjk ) ~x ~y.
Although it is not required, it is usually convenient to use a regular partition, i.e., a partit ion
with ual spacings.

2. Properties. Many of the properties that hold for single integrals also hold for dou ble integrals.
Some of these include the integrabili ty of any continuous or even piecewise conti nuous fu nction .

3. Warning. As in one-variable integration, [fJRfdA] [fJR g dA] =1= JJR!gdA, in general. In


other words, the product of two integrals does not usually equal the integral of the product.

4. New terminology. T h nam s of the proper ties are known as lineari ty, homogeneity, mono­
tonici ty and additivity. You should know the statements even if you don 't know the names.

5. Fubini's theorem . Th is tells you tha.t, for most reasonable functions, you can int grate one
variable at a t ime and the or der of integration does not matter.

6. Double integrals and volumes. Recall that if !(x , y) 2: 0, t he double integral is simply the
volume of the region between the graph of !(x, y) and the xy plane. If f (x, y) < 0, then we
subtra t the correspond ing volume between the graph of !(x, y) and the xy plane.

SOLUTIONS TO SELECTED EXERCISES


1. (b) We compute t he do uble integral as an iterated in tegral:

Note that we chose to integrate in x first be ause that integral is simple (integrat ing with
respect to y first would requ ire integration by parts).

2. (b) T his is the integrated integral

fa 1[ (a~2 + (by + C)x) [=al dy


fa (~ + by + c) dy = [b~2 + (~ + c) yJI~
1

a b
"2 +"2 + c.

5. We will use t h fact t hat J cf(x ) d:c = c J f( x ) d:t for any cons tant c. If we integrate in x first,
then g(y) is held constant in the fir t st p, and so

J k[J(IX )g(y)]dx dy = ldib f (x)g(y) dx dy = ld [


i b 1
f(x )g(y) d:c dy

ld lb[g( y) 1
! (x ) dx dy ,
80 CHAPT ER 5

where J: I( z ) dz is a constant in y. Factoring out the constant integral gives us

[ib I(X)dXl [i d g{Y)dyl .

7. The fu nction x 2 + y is positive over n, so t he double integral represents the desired volu me.

111\x2 +y) dydx = 11[ (YX


2
+ y; ) [=J = 11(x2 + ~)
dx dx

(
x3 + 3X ) 11 =~ + ~ =~.
3 2 0 3 2 6
1
Thus, the volu me is 16 .

11. Begin by substituting y danO, so dy x sec 2 OdO and

J x 2 + y2 = X sec e. We get

x2 - y2 J x 2 - Z2 tan 2 ()
J (x~" + y 2)"
. .. d
y = 2

x' sec 411u


(xsec 0dO)

,>
y

J x3(1 - tan B) dB
X4 sec 2 B
2
= J 2 2

cos 0 - sin (J dB
z x
cos 2B dO = sin 2B + C.
x J
2x

z-::--+-y2
From the triangle, we have sin e = y/ ';'2 ~ and CO! 0 = z / J z2 + y2. Then

sin 2B +C = sin e cos e +C = ~ . __y x +C= Y +C


2x x x Vx2+ y2 J x2 + y2 X2 + y2 '
Evaluating, we get
[I x 2 _ y2 y 11 1
10 (x 2 + y2 )2 dy = Xl + yl y=0 = x2 + l '
Then

1 o
111x2 -
0
(2
x +y
y2
2)2 dydx =
11
0
-2~-dz
x +1
1 1
= tan - 1 xlo= -4 '
11"

For the second integral, we substitute z = ytan </>, SO dx = ysee 2 ¢d</> &nd J xl + y 2 = y see </>.
This gi ves

z2 -y2
-:-;:--~:-;::- dx
(x2 + y2 )2 J y2
2
tan </> - y2 ( sec 2 </>#)
y4 sec4 ¢ Y
J
= 11 (tan 2
¢ -
y4 sec 2 </>
1) d</>

-J11 (1 - tan
y4 sec 2 ¢
l
</» d</> .

Using the method above, t his becomes - x / {x 2 + y2 ) + C. Evaluating, we get

X2 - y2 - x 11 - 1

1
--~- dx - -~
o (xl + y2) 2 - x 2 + y2 .: =0 - 1 + y2 .
Finally,

1
111o 0
( 2
x2 - y2
x +y
2) 2 dx dy =
11
0
-
-1
- 2 dy
1+ y
= - tan -1 y I01 = - -4 .
11"

Fubini's theorem does not apply in this case because (x 2 - y2 )/(Z2 + y2 )2 is not bounded on
the entire domain , namely, at (0 , 0) .
DOUBLE AND TRIPL E INTEGR ALS 81

5.3: TH E DOUBLE INTEGRAL OVER MORE GENERA L R EGI ONS


GO A LS

l. Be able to compute a double integral over an elementary region on the xy plane.

STUDY HINTS

l. Region types. A y-simple region is bounded by the lines x = =


a and x b and two curves which
are functions of x, for a ~ x ~ b. Such a region is called y-simple because the curves that
form the boundary can be described by yas a "simple' function of x. Similarly, an x-sim ple
region is bounded by the lines y = c and y := d and two curves which are functions of y, for
c ~ y ~ d. Such a region is called x-sim ple because the curves that form the boundary can be
described by x as a "simple" function of y. A sim ple region may be classified as both y-simple
and x-simple regions. See figure 5.3.3 of the text .

2. Classifying regions. For purposes of integrating, it is more important to be able to recognize a


region type rather than being able to name it. Your primary concern should be learning how
to perform double integration.

3. Simplifying complicated regions. Most plane regions may be broken up into regions each of
which is y-simple or x-simple. For example, the region at the
left is divided into six peices, each of which is either y-si m ple
or x-simple. In fact, many of the subregions are both y-simple
and x-simple.

4. How to integrate. It is best to use the iterated integral J: J:12(~/ f(x, y) dy dx for y-sim ple
regions. For x-simple regions, it is best to use the iterated integral JcdJ:12(~/ f(x, y) dxdy.

5. Choosing integration limits. When you perform multiple integration, be sure that the limits
of integration do not include any previously integrated vari ables. In particular, no variable
should appear in the limits of the ou termost integral. For example, the integral of (x + y)2
:s
over the region D defined by 9 y ~ x 2 , 0 ~ x ~ 1, should be written as

not i x' 1

10 (x + y)2 dx dy.

Sketching the region often heJps in choosing your limits. Also, it helps to read the limits of
integration in this example as follows : "while y ranges from 0 to x 2 ; x ranges between 0 and I"
or "for each x between 0 and 1, y ranges between 0 and x 2 ." Draw a picture and think about
it.

6. Definition of the integral. As in the last section , this is important prim arily for theoretical
purposes. Up to this section, we only know how to integrate over rectangles, so we cover a
region D with a large rectangle and let f(x, y) = 0 outside of D.

7. Double integrals and area. If f( x, y) = 1 on a region D , then JIDf(x, y) dA is the area of D.


·
~.

82 CHAPTER 5

SOL U TIONS TO SELECTED EXERCISES

l. (b) The iterated integral is

1 [1 +
1
2
3x

2x
1
dy ] dx = 12 ( 13x+1)
1
y
y=2x

dx

y = 3x + 1

12 + (3x 1 - 2x ) dx

1\ x + 1) dx = ( ~2 + x ) I: = ~.
At each Xo in [1,2], the region extends from 2xo t o 3x o + 1,

so we get the region shown in the sketch . This is a y-si mple


x
region because it can be described by

1~ x ~ 2 and 2x ~ y ~ 3x + 1.

2. (b) First, we will sketch the region. At each Xo in [-1 , 1], the region extends from -2'lxol to
Ixol, so we get the region in the sketch . The integration is most
easily done by di\riding the region into two parts, so

l1 11
1X
-1 1x
1

- 1 - 2/x l
' eX +Y dy dx = 10i -X
- 1 2x
eX +Y dy dx+
0
1 x

-2x
eX +Y dy dx .

Note that when x is in [-1 , 0]' Ix I =: -x . The first in tegral of


the sum is

1 0
- 1
X

( ex+yl- ) dx
y=2x
= 1-1
0
(1- e3x ) dx

(x - e~X) [1 2 1
"3 - 3e3 '
The second integral of the sum is

11( 11 (e-,-2 + e- 11 = - + - - -.
2X 2
eX +Y IX ) dx = (e 2x - e- X ) dx :::: x) e 1 3
o y=-2x 0 0 2 e 2

Therefore, the entire integral is e2 / 2 + l/e - 1/3e 3 - 5/6.


(e) The iterated integral is

11 [l>x + n
ym) dX] dy t
Jo
[ ( _x n+1
n +1
+ xym) I
Y

X= y2
1dy
t y'n_+_1 + ym+1 _ _y 2_n +_2 _ ym+2) dy
(_
Jo n+1 n+1
yn+2 ym+2 y2n+3
ym+3 ) 11
( (n + l)(n+ 2) + m+ 2 - (n+ 1)(2n+ 3) - m+3 10
1 1 1 1
(n + 1):-:-(n-+----::-:-2) + -m-+-2 - (n + 1)(2n + 3) - -m-+-3'

To sketch the region, note that for every Yo in [0 , IJ , x extends from x = y~ to x = Yo.
DOUBLE AND TR IPLE IN T EGRALS 83

4. The equation of an ellipse with serniaxes a and b is x 2 / a 2 +


y2/b 2 = 1 or x 2 /b 2 + y2 / a 2 = l. In either case, the areas
are equal. We will use the first equat ion , find t he area of
the region in the ·first quadrant , and t hen multiply by 4.
The region is descri bed as

The area of a region D can be computed by A = ffD dx dy.

In this case, we have

1
-A=
4
11
a
a b
a
2 o
..j1- x /a dydx = 1" ( l
a
y
b..j1-x o/a»
y=O
dx =b 1 g2
a
a
1- 2 dx.
a
1
Let u = x/a to get b f01 a..jl - u 2du = ab f0 .Jl=U2 duo The integral f01 ~ du is the
=
area of a quarter of a circle of radius I, which is If/4. So A/4 ablf/4. T herefore , the area of
the ellipse is A = ablf .
7. The region D, shown in the sketch , can be described as

- V3/2 ~ y~ V3/2 and 0 ~ x ~ _4y2 + 3,


3 x so

J.JDr 3
3
j ,;3/2 r- + x 3y dx dy
4y O
x y dx dy =
-,;3/2 JO
=
,;3/2 (x4-y
1- 4y 2+3 )' dy = j ,;3/2 (_ 4y 2+ 3)4y
dy .
j- ,;3/2 4 x =O - ,;3/2 4
Let u = _4y2 + 3 and we get an integral whose limits of integration are °and 0, so the integral
is 0.
12. From section 2.1, we know that an equ ation of the
form z2 = -a (x 2 + y2) is a cone. To get t he t ip at
(0,0, h) as shown in the figure, the equat ion becomes z

z2 = h- a( x +y2) . W hen z = 0, we get h = a( x + y2 ).

2 2

If a = h/r2, we get r 2 = x + y2 , a circle of radius

r. Thus, the equation of the cone is z = f(x, y) =

Jh - (h/r 2)( x 2 + y2). T he region of integration is

the cone's base, which is described as

-- - - - - - -- ~-------------------.~---.
y
x
__a_ _ ~_~ _

84 CHAPTE R 5

Therefore, the volume of the cone is

J rJ~ V/ :2 (X2 + y2)


-r - -..!r'-x' h- dy dx .

Fortunately, we are only asked to set up the integral. We can "simplify" the integral by using
t rigonomet ric substitut ion or by finding the following equation fro m an integral table:

j Ja2-x 2 dx= ~Ja2 - x2 + (~)


2
0 sin- 1 for a>O.
2 2 a

By using the equ ation above, the volum e of the cone becomes

J:r [(rJh - V; x 2) sin - (~r21_ x2) 1dx.1

Changing the volum e integral to polar coordinates, a technique introduced in chapter 6, makes
t he calculation easy.

14. Let D be a y-simple region ; t hen D is described by

a ~ x ~b and ¢ 1(X) ~ yS4>2(X)

and the integral IID f( x )g(y) dA becom es

b l¢2(X)

l a ¢ , (x)
f(x)g(y) dy dx .

Now, we integrate in y. Let G be an antiderivative of g and note that f( x ) is constant in y.


We get
b[ i ¢2(X) g(y) dy1dx = l bf(X)[G(4>2(X)) -
la
f (x)
¢,(x) a
G(4)l(X))] dx .

G depends on x, so we can't consider it to be const ant and we cannot factor it outside of the
integral sign . Thus , in general, IID
f( x )g(y) dy dx is not t he p roduct of two integrals.

5.4: CHANGIN G T HE ORDER OF IN TEGRATIO N


GO ALS
1. Be able to evaluate a double integral by changing the order of integration.

2. Be able to st ate a nd understand t he mean value t heorem for dou ble integrals.

STU DY HINT S
1. Rati onale for changing order. Recall t hat Fu bin i's t heorem allows you t o change the order of
in tegration . Somet im es , changing the order of integrat ion simplifies a problem. Try doing ex­
am ple 1 without changing t he order of integration and compare the efficiency of both m ethods.
(You will p robably need to use trigonom etric substitution.) At other times, a double integral
can only be com puted if the order is changed .
2. B egin ning the change of mYier. It is useful to sketch t he region of integration from the given
lim its before choosing new limit s.

3. Mean value theorem for int egrals. If two conditions hold: (i) f is continuous amd (ii) D is an
elementary region , then the conclusion is

JIn f( x , y) dA = f(x o, yo) . area(D)


for som e point (xo, Yo) in D.
DOUBLE AND TRIPLE INTEGRALS 85

4. Mean value inequality. If m is the minimum value of ! (x , y) on D and M is the maximum,

Jl
then
m· area(D) ~ !(x, y) dA ~M . area(D).

This allows you to estimate the value of a dou Ie integral .

SOLU TIONS TO SELECTED EXERCISES


1. (b) Fir t, we recall that cos 2 B = (1 + cos 2())/2 and compu te the integral as written:

r/ ros
io io
2 9

cos() drdB =

iT

4
From the graph, we see that if we choose an TO, then B extends from 0 to cos- 1(ro) . Thus, the
region can also be described as

O ~ r ~ l and O ~ B ~ cos-l(r).

Therefore, changing the order of integration gives us

11COS-l(r) cos BdOdr = 11( sin() ICOs- (r)) dr = 11si n(cos- 1(r)) dr .

1
a 0 0 9=0 0

From the triangle, we see that sin( cos- 1(7))) =~. We recognize the integral s he area
of a quarter of a circle of radius 1, so
1

1
iT
~ dr =- .
o 4
2. (c) The region of integration is sketch here. T here is no obvious
function whose derivative is exp (x 2 ) , so we try changing the order of Y
in tegration. The region can be expressed as a y-simple region: 4

o~ x ~ 2 and 0~ y ~ 2x.

Thus , the integral becomes

(2rx
ioio
exp (x 2 ) dy dx = r [ yexp (x )1y=o
io
2
2X

] dx .

2 x
Now let u = x 2 and we get
12 2x exp ( x
2
) dx = 14 eti du = eti I: = e4 - 1.
86 CHAPTE R 5

3. This formula is an application of the mean value inequa.lity. T he region D is [-11",11"] X [-11" ,11"], eo
A (D ) is t he area of D, which is 411"2. The funct ion I(x, y) = esin (x+ y ) is largest when sin(x + y)
is largest, i.e., when x + y =
rr / 2 ± 2n1l" , where n is an integer. Similarly, I (x , y) is smallest
=
when x + y - 11" / 2 ± 2n1l" . Over the region D, we have -1 ~ sin (x + y) ~ 1, so m li e and =
M = e 1 . Therefore, substituting everything into t he mean val ue inequality gives us

~ ~ 4!2 J L l( x , y) dA ~ e.

6. The area of the triangle is t·


The function I(x, y) = 11(y - x + 3) is smallest when y - x + 3
is largest on D. Notice that y ~ x in D, so y - x + 3 is largest when y = x , and so m = 1/ 3.
Similarly, we see that M occurs at (1,0) where 1(1, 0) = Now, the mean val ue inequality t.
gives us

~~2JlI(x,Y) dA~~ or ~ ~JL/(x,Y) dA ~ ~ .


10. We need to divid e the triangle into two parts: one part for x in

[0,1] and the other part for x in [1,2]. When x is in [0 , 1], we have Y

x ~ Y ~ 3x, so the integral over this region is

[ 1 [3'"
J J.,
o
eX- Y dydx 11(- I::J ee - y dx

(2.2)

11(_ e- 2e + l )dx x
2
e- ", ) 11 1 1

( - 2-+ x 0 = 2e 2 + 2·

For the second region, we have x ~ y ~ 4 - x, so the integral over t hat region is

1 e 1\-e
2 [4 - "
1
4 X
1 J., eX - Ydy dx 2 ( -
X
_
YI - ) dz = 2X
-
4
+ l )dx
1 11 = " 1
2X 4
_e - ) 12 1 1
( 2 +X 1 = 2e 2 + 2·
Adding th e two integrals together, we get 1 + 1/ e2 .

13 . First, we sketch the region. For a.ll Yo in [0,1], x goes


from Yo to -/2 - y~ . To describe the region as an
y
x-simple region , we have to subdivide the region at
x = 1. When xis in [0 ,1]' we have 0 ~ y ~ x . W hen x
is in [1,)2], we have 0 ~ y ~ )2 - x 2 . Thus, we have

2 112 x

Jo[1 [1~ I( x , y) dx1dy = Jot [Jro I (x, y) dy] dx +


Y 1v'2 [[~
1 Jo 1
I( x , y) dy dx .

In many cases, the left-hand side will be easier t o work with .


DOUBLE AND T RIPLE INTEGRALS 87

15. T his proof requires using the chain rule. Let

G( x , u) = l jd x
f(u, y, z) dz dy .

We want to find dG/dx (the "total derivative ," if you will ) when u = x. By the chain rule ,

dG _ aGI + aG I
dx - ax u= x au u=x'

~~ is simple; it is equal to :x J: [t
f( u, y, z) dZ] dy. From the fundamental theorem of (one­
variable) calculus, we simply take the "integrand" (the dz integral here) and replace y by x
(because we are integrating with respect to y) . At u = x,

aG
~
uX
I
u=x
= jdf( x , x, z) dz.
c

~~ i s a little trickier. First,

aG
au au
l
a a Jc{ d f(u,y,z)d z dy
X

a
j
X d

l
a c auf(u,y, z )dzdy,

assu ming that f is a nice function such that the order of integration and differentiation can be
interchanged. Now we want to evaluate everything at u = x, so we simply replace u by x, and
get
aGI
~
uU u =x
= lj a
X

c
d ()
-;;-f(x,y,z)
uX
dz dy .

Add the two results and you've got it .

• 5: THE T RIP LE INTEG RA L


GOALS
l. Be able to compute a triple integral over geJleral regions in space.

2. Be able to change the order of in tegration for computing triple integrals .

STUDY HINTS
l. Notation . We use dV for the differential dx dy dz since it represents a volume.

2. P r·operties. Many of the properties of the triple integral are the same as those of the double
integral. T he triple integral may be considered as a t hreefold iterated integral and we integrate
from inside to outside. Fubini's theorem still holds and we can still integrate all piecewise
continuous functions.
3. Balls. Example 3 demonstrates one way to describe the unit ball. Another way to describe it
IS

Yet another description is

The solu t ion of example 3 uses a y-sim ple description of D. Using an x-simple description would
generate three more descriptions of the unit ball.
88 CHAPTER 5

4. How to integrate. As with double integrals , knowing how to set up the limits is important.
Drawing a picture of the region is helpful for finding the limits of integration, although in
many instances it is easier said than done.
5. Triple integrals and volumes. If I(x, y, z) = 1, then fffw I dV is the volume of W.
6. Integration trick. A time-saving device is shown in example 4. If we realize that an integral is
the area of all or part of a circle, then we don 't have to go through the process of fi...nding an
antiderivative.
7. Factoring out integrals. Some special t riple integrals can be easier to compute if you use the
following fact: If the limits of integration of the innermost integral do not involve a certain
variable and the limits of integration for that particular variable are constant, then that variable
may be integrated separately and multiplied by the remaining integral. For example,

11 1
1 2
:&
3
I( x)g(y)h(z ) dzdydx = (1 3
h(Z)dZ) (1 1'2XI( x)g(y) dy dX)
1

because the variable z does not appear in any of the limits of integration and its limits of
integration are the constants 2 and 3. We cannot integrate X' separately because it appears in
the limits of integration for y.

SOLUTIONS TO SELECTED EXERCISES


2. It is easier to integrate in x before integrating in y:

JJl e-xYydV = 111111 e-X'Yy dx dydz = 1111[ (e=:Y) yl:=J dydz


1111
o 0
(l-e- Y)dydz= 11[
0
(y+e-y)1 1]
y=o
dz=-1
e a
11 1
dz;;:::-.
e

ote that you would have had to use in tegration by parts if you had integrated in y before
integrating in x.
6. It is useful to make a sketch of the region. T he elliptic

cylinder runs parallel to the y axis and its intercepts

on the coordinate axes occur at x = ± 1/ .j2 and z =

±l. The ball is a sphere of radius 2 centered at the

origin. To describe t he region inside t he cylinder and

the ball as an elementary region, let x vary between

-1 / J2 jnd 1/J2. Next, for any given xo, we let z go

from - 1 - 2x5 to J I - 2X6. Finally, for any given

(x o, zo), we let y vary between two surfaces of the ball ,

namely, y = - J4 - x5- Z6 and y = J4-x5-z6.

Therefore, t he desired region is descri bed by

-1/V2 ~ x ~ 1/ V2, - Jl- 2x2 ~ Z ~ J l- 2X2 , - \1"4 - X2 - z2 ~ Y ~ J4 _X2 - Z2,


Another possible solution is

-1 ~ Z ~ 1, - J(l- z2 )/2 ~ z ~ J(1- z2)/2, - J4 - x 2 - z2 ~ Y ~ J4 - x 2 - z2.

9, The surface z = x 2 + y2
is a pa.raboloid opening upward. The surface Z = 10 - x 2 - 2y2 is a
paraboloid opening downward. When we equate the two surfaces, we see that they intersect
where x 2 + y2 = 10 - x 2 - 2y2 or 2X2 + 3y2 = 10, which is an ellipse. The ellipse can be
described by

- V5 ~ x ~ V5 and - J (10 - 2x 2 )/ 3 ~ y ~ J(10 - 2;2)/3.


DOUBLE AND TRIPLE IN T EGRAL S 89

As a triple integral , the desired volume is

l
Vsjv'(lO-2XJ) /3 1 10-X2 -y2 Vs jv'(1o-2X~)/3
dz dy dx = (10 - 2x2 - 3y2) dy dx
j -V5 -v'(10-2X ~) / 3 x'+y' - Vs - y "-(l-O---2x---::')/:-3
-:-

y3)1v'(lO-2X~)/3 l
Vs
l -Vs
[ (lOy - 2x 2 y _
-y(lO- 2x 2 )/3
dx

_4_ j Vs [10v'2~ _ 2v'2x 2 /5 _ X2] dx.


3V3 -Vs
From the integral table, we have J ~ d:c =
(x/2)J5 - x 2 + ~ sin-l (x/V5) +C and
J x 2 J5=X2' dx = (x/8)(2 :c 2 - 5)J5 - x 2 + 28 sin- (x/V5) + C. Therefore, we get
5 1

v = _4 [10..;2 ( ~/5 _ x 2 + ~ sin- l (~))


3V3 2 2 V5
V5
£
- 2v2 (x_( 2x2 - 5)Vr;;---.; sm- ( x) )]
5 - x + -25. 2 1
- 1
=- - =-
251l'V2 501l'.
8 8 v5 -V5 V3 y6

12. We will co mpute the volume of the region in the first 0 tant where x, y and z are positive.
Due to t he symmetry of the region , this result multiplied by 8 gives the correct answer. In the
first octant, we let x go fro m 0 to a. Since the region lies within the cylinder x 2 + y2 ::; a 2 ,
one boundary is y = v' a2 - x 2 • Since we are only looking at the first octant, we have 0 ::; y ::;
v'a 2 - x 2 . Finally, the z values of the region go from the xy plane to the cylinder x 2 + z2 = a. 2.
Thus, 0 ::; z ::; v'a2 - x 2 . The volume of the region in the first octant is the triple integral

l al~l~
a 0 a
dz dydx =

Therefore, the volume of tb entire region is 8(2a3 /3) = 16a 3 / 3.


17. The plane x + y = 1 can be written as x =1- y. T herefore, W can be described by

0 ::; y::; 1, O::; :c::; l-y and 0 ::; z ::; 1l'.

Therefore, the desired integral is

r r1l1-Y x 7rl 1 1
(cos z ) ( x331 -
Y

10 10 0
2
cosz dxdydz
1 o 0 x=o
) dydz

Alternatively, one can integrate in z first:

11 l 1 1 Y
-
1f

x 2 cos ;;dz dxdy = l 1


1
1 Y
- ( x sin z [=J
2
d:cdy = 111 1-Y Od:cdY=0.
90 CH APTER 5

21. Evaluate as an iterated integral:

112X1X+Y t 1 { 2X

1J
X
dz dydx = { 2X [ I +Y J dy dx = (x+y-x2_y2)dydx
1o 0 x'+y 2 J Jo
o z z=x 2 +y'
0
o

11 [((x - x 2
)y + y; - ~ ) [:al r
dx = Jo
1
(
4x
2
­ +3)
14
dx

4x
( 3
3_ 7X4) 11 = ~
6 o 6·

22. (a) Starting with the interval 0 ::; x ::; 1, sketch the area D

which extends from y = 0 to Y = Xo for each x o in [0, 1]. Then z &

over each point (xo, Yo) in D, extend t he region W from z = 0

=
to z Yo.

x (1,1,0)
(b) We will now fi nd an equivalent integral of the form

f f fw! (x, y, z) dx dy dz .
First, note that z goes from 0 to 1. Now , for each Zo, Y goes from Zo to 1. This gives us a
triangular area in the yz plane. Finally, for each (Yo , zo) in the triangular area, x goes from Yo
to 1. Thus , we describe Was follows:

o::; z ::; 1 and z::; y ::; 1 and y::; x ::; 1.

Thus,

111
1 X Y
!(x, y,z )dz dy dx = 111
1 1 1
! (X,y , Z)dxd Y dz.

As a quick check, we can see t hat both regions have the same volume by letting !(x , y , z) = 1.
In this case, the volume is i.
26. We note that x 2 + y2 + z2 ::; 1 describes the unit ball

centered at the origin. If z ~ 0, then the region W is

the upper hemisphere as depicted in the sketch . We z

are given t hat Ixl ::; 1 and Iyl ::; 1, but this does not

provide any extra information. To find t he limits of ,,


\
integration, note that x varies between - 1 and 1, so
-1 ::; x ::; 1. For each x, t he variable y extends fro m
-Vf=X2 to +Vf=X2. Finally, for each (x, y) in the
unit disk , the variable z goes from 0 to + VI -
x 2 - y2. "
).. --- --- - - --~~
Thus, W can be described by y

-1 ::; x ::; 1, - ~ ::; y ::; ~

and x
o :S z ::; VI - x 2 - y2.

T herefore ,

1
1 1~ lVI-X,-y2
jii W
! dV =
- 1 -~ 0
!(x , y, z) dz dy dx.
DOUBLE AND TRIPLE INTEGRALS 91

27. The volume, as a triple integral, is

11 [I
f( X' Y ) ]
dz dx dy
D 0

and integrating with respect to z yields

JL !(x , y) dx dy,

which is the double integral of f over D.

SOLUTIONS TO SELECTED REVIEW EXERCISES FOR CHAPTER 5

2. Evaluate this as an iterated integral:

6. The region is sketched here. As an x'-simple region, it


can be described as 0 ::; y ::; 1 and 0 ::; x ::; y'l , 0 we
get Y

[( X + y)3IY2 ] dy
1
a
1

3 x=o
(1,1)

9. Evaluate as an iterated integral:

l 1 [(yz + x;2 ) [=Jdydx


1 X

111 (y2 x~2)


X

+ dydx

= 11[(1+ ~) y; [=J dx

11 ( ~3 ~4 ) = (~;
+ dx + ;~) I: = 6~ '
92 CH APTER 5

13. From one-variable calculus, we expect t.o get


y

l b
(f( X) - g(x)] dz

if 1 ~ 9 for z E [a, b]. The region D can be described


as a ~ z ~ band g(x ) ~ y ~ I (x ) because it is a
y-simple region. Therefore, the area of Dis

a b x

ll lb [yl Ib[/(x)- g( x) ]dz .


b f
. (X) dy dx = f(X)] dx =
a g(", ) a !I =g( 3; ) a

15. (b) As a y-simple region, the unit circle can be described by

- l~x ~ l and - ~~ y~ ~.

Thus, we have

11v'1=X'2 x 2 y2 11-1[~3 1 ~ 1dx == -211x (1 - 2


X 2 )3 / 2
1-1- ~ dy dx = 2
3 y=_~ 3 -1

dx.

Use the trigonometric substi tution : ~ = cose , x = sin e and dz = cos edo, 80

J x 2 (1 - x 2 ) 3 / 2 dx = Jsin 2 Bcos 3 B(cos e dO) = J (sin2 e - 2sin 4 B+ sin 6 e) de,

which was obtained by using the identity sin 2 B + cos 2 B 1 and expanding. Now, use the=
identities: sin 2 e = (1- cos2e)/2 and cos 2 e = (1 + cos 2e)/2. We get

1 - 3 cos 2B + 3 cos 2 2fJ - cos3 2e] dB


J[
1 - cos 2fJ _ 1 - 2 cos 2e _ cos 2 2B
2 2 2 + 8

J( -~ ~
2 3
cos4B + cos 20 sin 20) dO= .i. _ sin 4B _ sin 2e + C.
16 16 8 16 64 48

Now , we use t he double angle identities and the following which

~x
we get from the triangle

e == sin - 1 :1: , sin e == z and cos e = ~.


~1 - x2
Substitut ing and evaluating at ± 1, we get

I
sin - z _
[ 16
4x ~(1-
64
Z2- x2) _ 8z 3 (1- Z2) ~lll
48
_~ .
16
-I

Therefore, the original integral becomes (2/ 3)(11"/16) == 7r/ 24.


DOUBLE AND TRIPLE INTEGRALS 93

20. As the integral is written, the region is described as 0 ~ x ~ 2 and

- 3v'4 - x 2 / 2 ~ y ~ 3v'4 - :z:2/ 2, which is a y-simple region . When we

=
rearrange y 3v'4 - x 2 /2, we get y2 / 9+ x 2/ 4 1, which we recognize
=
as an ellipse, so our sketch is as shown . We evaluate as follows:

r j3"f4-X
2 2
/2 ( 5 + y3) dy dx
io -3~/ 2 v'2+X

l' [(vA--. + ~') C:~:~<J d.


12 15~ dx = - 10(2 - x)3/21: = 20V2.
24. As written, the region de cribed by 0 ~ y ~ 1 and 0 ~ :z: ~

3y is an x-simple region . The region is sketched as shown.

We evaluate t he double integral as follows:

t [3
io io
y

e +Ydx dy =
X
11 ( X 1::0)Y
e + dy

= e4 _e+ ~.
4l1
1
t (e 4Y - eY)dy = ( e _e ll
Jo 4
)1 0 4 4
27. The region is sketched h reo Note that is consists of the following two regions:
o ~ :z: ~ 1 and 0 ~ y ~ _ x2 + X
1 ~ :z: ~ 2 and - x2 + X ~ Y ~ o.

We evaluate as a sum of integrals:

y tl-X2+X [2j O
io 0 !(x , y) dy dx + il _X 2+x I(x , y) dy d:z: .
x
The first integral is:

11­
+ +
1 X2 X
+ (:z:2 2xy2 2) dydx

l' [(x'Y+ 2xt + 2Y) C+<] dx

T he second integral is :
y3 0
[ 2j O (x 2 + 2xy2 + 2) dy dx = [2 [ ( x 2y + 2x + 2V) 1 ] dx
i1 - x'+x il \ 3 y=-x'+:r

(-1) 12 (-~:r/ + 2x6 - 2:z:5 - ~4 + X3 - 2X 2 + 2X ) dx

_x
(-1) ( 12 + 7
8
2x7
-3 -
x
6
x
5
X4
15 + 4
3
2x
- 3 +x
2)110 =(- 1) ( - 584
105 -
27 )
70 .

Add the two integrals together to get Ii .


.....

94 CHAPTER 5

31. T he function f (x, y) = x 2 + y2+ 1 has the value r2 + 1 where r is the distance from the origin.
Thus, on D, 1 ::; f( x , y) ::; 5. The area of Dis 411", so by the mean value inequality, we have
the desiIed result.
35. Dividing thIough by x2, the integrand becomes (l/x)/[1 + (z / x )2]. Let u = z/ x, so 3; du = dz
and the integral becomes

[1 [Y t / ../3 (
10 Jo 10
1 )
1 + u2 du dx dy
t
Jo Jo
r ( tan- 1 11/../3)
u u=o dx dy = "611" J[1o 10
[11
dx dy

-11"
6
11 (
0
xI
Y

x=O
) dy = -161" 11
0
y dy = -161" ( -y22110 ) = -1211" .

TEST FOR CHAPTER 5

1. True or false. IT false, explain why.

(a) The area between the parabola y = 4 - :r 2 and the x- axis can be expressed as the double
integral

[ j
4 _ X2 2
dx dy.
Jo -2

(b) If g(x , y, z) is integrable over a region Q in 1R3, then

JJk g(x,y,z )dx dydz = JJ kg (x,y,Z ) dy dZdX.

(c)

tj4 jS_
Jo y2
- 2
y3 (z + y)3/2 dz dy dx = (j4
- 2
15- y2
y3
(z + y)3 /2 dz dY) .4.

(d ) For any integrable function f (x, V), the double integral lID
f (x, y) dx dy is the volume of
the region bounded by t he sUIfaces z = f( x, y) and z = 0 over D.
(e)

1 2jl (X2 + 4x
o - 1
+ .JX)(y3 _ y2 + 4y) dydx

[1 1 [1 1
2 1 2 1
1 ( 3;2 + 4x + .JX) dy dX ] 1 (y3 - y2 + 4y) dy dX] .

2. Change the order of integration and rewrite the triple integral 101:2Io ydz dy dx in five ways.
1 1
-
Compute its value .

3. Find the volume of the region bounded by z = x 2 + 1, z = 0, y = 0 and y = x 3 for -1 ::; x ::; 2.
4. Evaluate the double integral I/ - COS(I ) Ic1os- ( I _ y 2) y dxdy.
1
I

5. An am ateUI magician's hat is described by z::; 9 - x 2 - y2 and z ~ O.


(a) Use Cavalieri's principle to calculate how much milk he can pour into his hat ifhe wants
to bake a magic cake.
(b) Express the volume of the hat as a double integral in t he form II f(x, y) dx dy.
(c) Use parts (a) and (b) to quickly calculate I~3(9 - t 2)3/2 dt.
DOUBLE AND T RIPLE INTEGRALS 95

6. Evaluate the double integral JJT Ix - yl dy dx, where T is the triangle wit h vertices at (2 ,0),
(0,0) and (0, 2) .

7. (a) Suppose f : W -t ~ is continuous and W is an elementary region in IR . Le t V( W) be


t he volume of W, m the minimum value of fon Wand M the maximu m value of f on W.
Make a statement relating m, M, V and the triple integral of f over W.
(b) Show that
v'22 :S
16 e
l 1l"/4
0
1 Y

a 0
cos(xy)
eY
7r
+z dx dz dy :S -8 '

8. Eval uate J04JolJ3z exp (x3 ) dx dydz.

9. Eval uate JJD x dx dy where D is the region shown.

T he bOWldary on the left is y = 4 - x 2 . All of the

other boundaries are straight lines.

(4.- 3)

10. A cake-eating monster saw a pyramid-shaped cake which he easily devoured. T h cake had
a square cross-section and at a height h, the side of the square had length 8 - h. T he cake
monster's son could only eat the top ~ of a. similarly shaped cake.

(a) Use Cavalieri's principle to calculate the cake monster's minimum stomach capacity.
(b) How t all is t he rem ainder of the cake that the son started eating?

. .

97

6 THE CHANGE OF VARIABLES FORMULA


AND APPLICATIONS OF INTEGRATION

6.1: THE G EOMETRY OF MAPS FROM]R2 TO JR2


GOALS
1. Be a ble to determine whether a m ap is one-to-one.

2. Be able to graph a region D which is the image of a region D* under a mapping .

STUDY HINTS
1. No tation. The equality D = T (D* ) expresses that fact that T t akes a point in D* and maps it
to a point in D, and t hat all such points of D are obtained in this way. T is called a mapping
or a transformation. T hinking of T as a fu nction, this is similar to y = f(x) for f : JR -t JR.

2. One-to-one. If two distinct points always get mapped to distinct points, i.e., distinct points
never get m apped to a single point, then the function is said to be one-to-one . A domain may
need to be chosen carefully for this property to hold. (See example 3.) For integration, this is
a nice property ; otherwise, the double integral over D may not equal the double integral over
DO.

3. Onto. If every point of a range D can be obtained fro m some point in the domain D" , then the
m apping is onto. Onto does not imply one-to-one because t wo points of D* may get mapped
to one point of D. By t he sa me token, one-to-one does not imply onto.

4. Finding D f rom D* . In many cases, the range D can be determ ined by simply mapping the
boundary of D* . Then decide whet her the map takes D* to points inside or outside the
bound ary of D. For exam ple , let D be the unit disk. If

then T m aps the uni circle to the unit circle, and T maps a point such as (~, ~) to (2,2), a
point outside t he circle. So T maps t he inside of the circle to the outside [and is not defined
at (O, O)J .

5. Important exercise. The result of exercise 8 is used in the examples of section 6.2. It states
that if T (x) = Ax, then T is one-to-one if and only if det A =f:. O. The result is true for all n x n
determinants.

6. L inear transformations. If T( x ) = Ax + b and A is a 2 x 2 m at rix, t hen T takes parallelograms


to parallelograms. If D is a parallelogram, one can often obtain it from the unit square D*
with such a T.
-

98 CHAPTER 6

SOLUTIONS T O SELECTED EXERCISES

2. The transformation is given by the following equation :

T( x·, y* ) = Ax = [1/-/2 1/-/2] [ X:


1/-/2 -1/V2 y ].

We compute det A = 1 i- 0, so by theorem 1, the linear m apping T maps vertices to vertices.


The origin of D* gets mapped to T(O, 0) = (0,0) in D. Similarly, the points (1,0), (1, 1) and
(0,1) of D· get mapped to (1/ V2, 1/V2), (0, -/2) and (-1/V2, 1/)2) , respectively, in D. It is
easy to show that the length of each side of the transfor mation is 1 and the dot product can
be used to show that the angles are 7r /2. Thus , T rotates the unit square by 45 degrees.

y •• y!

1 x* x

4. The four vertices (0,0), (1,0), (1,1) and (0,1) of D should be mapped from the four vertices
(0, 0) , (~, ~) , (152 , \6) and (t, 152 ) of D·. We want to find a matrix A so that

T( x*, yO) = Ax = [~ ~] [ :: ] .
=
When C~, 1) gets mapped to (1,0) , we get 8a/5 + 4b/5 1 and 8e/5 + 4d/ 5 0, so d = -2e. =
When (i, °
Y52 ) gets mapped to (0,1) , we get 4a/5+ 12b/5 = and 4c/5+ 12d/5 = -4c = 1, or
a = -3b and c = -i,d = t. Substituting a = - 3b back into 8a/5+4b/5 = 1 gives us - 4b = 1
or b = -t, =
and so a ~. T herefore,

T (x • , ~I *) = (34'x *- 4'y 1 *+ '12 y*) .


1. ,- 4'x

7. Since sines and cosines appear, we will try to use the identity sin 2 t + cos 2 t = 1 to eliminate
the parameters and obtain a recognizable fo rm: x 2 + y 2 = p 2 sin 2 ¢ (cos 2 B+ sin 2 B) = p2 sin:! ¢
and x 2 + y2 + z2 = p2(si n 2 ¢ + cos 2 ¢) = p2 . We recogn ize that x 2 + y2 + z 2 = p2 is a sphere
°
of radius p; since :s p :s 1, D is the unit ball.
The map T is not one-to-one. For example, (0, 7r, 7r/2) and (0 , 7r /6, 7r / 5) both m ap to the
origin. Also, (I,O,7r/2) and (1, 27r , 7r/2) both map to (1, 0, 0).
Since p=° always gets mapped to the origin , we want to elimin a.te that point . Also, si nce
°
B = and () = 27r give the same mapping, we want to eli minate either one of those points.
Hence , T can be made one-to-one by using the following intervals: ¢ E [0 , 7rJ, B E (O ,27r] and
pE(O ,l].

10. Using the hint and applying T to the parallelogram described by q = p + .xv + /-,W, we have

T( q) = Aq = Ap + >.Av + /lAw .
Since A is a 2 x 2 matrix and p, v, w are all vectors in JR2, then Ap, Av and Aw are all vectors
in JR 2. Since A is nonsingular (det A i- 0), Av cannot be a scalar multiple of Aw if v is not a
=
scalar multipleofw. T his is because if Av aAw , a scalar, then Av -aAw = A(v - aw ) 0 , =
°
which implies t hat either det A = of v = aw . Thus, the image of T is a parallelogram in JR 2.
TH E CHANGE OF VARIABLE S FORMULA AND APPLICATIONS OF INTEGRATION 99

6.2: THE CHANGE OF VARIABLES THE OREM


GOALS
1. Given a transformation T, be able to compute its Jacobian.
2. Be able to use the J acobian to change variables in do uble and triple integrals.
3. Be able to state and use the change of varia bles formula for polar, cyl indrical and spherical
coordinates .

STUDY HINTS
1. Review. You should review the definitions of polar, cylindrical and spherical coordinates from
section 1.4.
2. J acobian. The Jacobian is a determinant, not a matrix. In this section, we introduce the
notation
a(x , y) o(x,y,z )

and
a(u, v) o(u, v, w) ,

which are the determinants

ax/au ax/av ax/a u ox/ov ax/ow


ay/au oy/ov and ay/au oy/av By/ow
l az/au 8z/Bv Bz/8w
respect ively.
3 . Changing variables . As stated , the change of variables theorem requires th at T be one-to-one.
However, the theorem still holds if T is not one-to-one on the bou nd ary of D· , a sit uation th at
occurs in a number of examples . A similar theorem holds for triple integrals.
+ y 2 occurs in the integrand, try using polar or cylindrical
4. Useful change of vari ables. If x 2
coordinates , which have the Jacobian r. If x 2 + y2 + z2 occurs in the integrand, try using
spherical coordinates, which has t he Jacobian p2 sin 1jJ . These two Jacobians should be memo­
rized .
5. Integrating over ellipsoids and ellipses. If you are integrating over an ellipsoid with equation
(x /a) 2 + (y/b)2 + (z/e) 2 = 1, the subsitutions u = x/ a, v = y/ b and w = z/e may be made
to transform the ellipsoid into a sphere. Spherical coordinates may now be used to integrate
over u 2 + v 2 + w 2 = 1. Similar methods can transform an ellipse into a circle a nd then polar
coordinates may be used.
6. Limits of integration. W hen you set up the limits of integration, remember that ¢; is mea­
sured from the "north pole," not fro m the "equator ." Also, recall that () is measured in a
coun erclockwise direction.

SOLUTIONS TO SELECTED EXERCISES


2. We know that x = u + v and y = u - v. Adding the two equations gi ves us (x + y)/2 = u and
sub traction gives (x - y)/2 = v . By mapping the boundaries of the triangle in the x1Tpl ane ,
we ge t another triangle in the uv-plane as shown .
y v

(~.~)

u
100 CHAPTER 6

The Jacobian is the absolute value of

a(X,y ) I_I ax /a u ax /av 1_11 1 1-- - 2,


la(u, v) - ay/au ay/av - 1 - 1

so the Jacobian is 2. In this case, we integrate x + y = 2'11 as follows:


t/2 [I - V { 1/2 { I-V

J
o [

JD (x +y)dxd y = Jo i" (2u)(2)dudv= 4


Jo
J" u du dv

1/2 (u11-V) dv =2 11/2 (1 - 2v) dv=


2
2(v-v 2)1
1/2 1
4
1o
-
2 ti = v 0 0
= 2'

Don 't forget t hat the Jacobian is the absolute value of the determinant. A direct calculation
gives us

1111 o y
(x+y) dx dy = 11 - +
0
[( x2
2
xy) 11
x=y
1dy =11(1-2 + y - -32y2 ) dy = ( -2y + -y22 - -y3)
0 2
11 = 1
2 0
-0

3. (b) We compute th at x - y = 2'11 - 3v and the Jacobian is

a(x, y) = I 14 a I= 12.
Ia(u, v) 2 3
Therefore , the integral becomes

12111\2U - 3v) dvdu = 1211 [ ( 2UV - 3~2 ) [=J du = 1211 ( 2U -~) du

12 (u 9;)1: = 12 (1- ~) = - 42 .
2
_

Notice that we did not even need to know what D looks like to perform the integration. By
theorem 1 of section 6.1 , this linear transfor m ation maps the rectangle D· to a parallelogram
D. We map the vertices and then connect them to get D.

v y
(4,8)

2 (4,5)

u x

7. We compute t hat x 2+ y2 = (u 2 _v 2 )2+4u 2 v 2 = u4 + 2u 2 v2 + v4 =


v
(u 2+ v2)2, and so ';x2 + y2 = u 2+ v2. In this case, the Jacobian

IS

la(X'Y)
a(u, v)
1= 1
2v
2u
- 2v
2u
I= 4u 2 +4v2 .

T herefore,

J1 D.
4(u2
2
u
+ v22 ) dudv = 4
+v
Jj
D.
dudv .
1 u

This is just 4 times the area of D* , which is a quarter of a unit


circle, so the answer is 7r .
TH E CHANGE OF VARIABLES FORM ULA AND APPLICATIONS OF INTEGRATION 101

11. The transformation takes (x,y) to (rcosO,rsin O) , so

the Jacobian is

y
8 (x,y) =
8(u, v}
I I c~s f)O
Sill
-rsinO 1= r .
rcos f)
2
1
Recall that t he area is t he double integral fID dx dy.
In polar coordinates, this becomes

2lrl1+sinB rdrdO
J10. r drdO =
1 o 0

f21r (r2 11+Sin B) dO


Jo 2 r = O x
~ 1 271: (1 + sin 0) 2 dO.

Use the half angle formula to get

-11271: ( 1 + 2·
sm 0+ I- COS O) dO-- -1 (30- - 2 cos II
rJ -
sin20 )1
--
21r
_
-
311"
--.
20 2 22 40 2

14. When D- = [0, 1] X [0 , 211"] gets mapped. to a unit circle, we know we are using polar coordinates.
Substitute u = 1 + r2 to get

17. Here, we want to take the uni t square and find a transformation which maps it 0 the giv n
region R, which is a parallelogram. Note that we can map (0 , 0) 0 (1,0) , (1,0) to (~ , ~),
(0 ,1) to (0 ,1) and (1,1) to (~, ~ ) . The transforma ion should be linear, so we should get
(x, y) = (au + bv + c, du + ev + f) . When (u, v ) = (0 ,0) gets mapped to (x, y) = (1 , 0) , we
see that c = 1. T he mapping of (1, 0) to ( ~,~ ) gives us a = ~,and he mapping of (0 , 1)
to (0 , 1) gives us b = -1. Similarly, we can determine d, e and l The transformation is
(3u/2 - v + 1,3u/2 + v), so the Jacobian is

8
(x , Y)
1 8(u,v)
I= II
2
-11 I = 3.

(Yz-X)

--~------------,~
1 u 1 x

Also, we compute x + y = 3u + 1 and x - y =1- 2v, so the change of variables gives us


102 CHAPTER 6

Since the integrand can be factored into separate factors which contain only one variable and
the limits of integration are all constants, we get

(1 (3u + 1)2 dU) (1 e 1)31:) ( e~;V I:)


1 1
3
1
-
2v
dV) 3 ( (3U;

3(64; 1) (e-1_~ e = 221 (e _~) . 1


)

20. The Gaussian integral tells us that

1: exp(-x
2
) dx =,fo.

We substit ute u = 2x and du = 2dx to get

1: exp( _ 4x
2
) dx lim
a-+oo J
aexp(-4x
-a
2
) dx == lim
a-+oo
J2a exp(-u
-2a
2
)(du/2)

-21 a-+oo
lim Jaexp( _u
_
a
2
) du = ,fo
2 .

Since the function exp( -4x 2 ) is symmetric about t be y axis, we deduce that

1 00

exp (-4x2) dx =~ 1: exp(-4x


2
) dx = V;.

23. We will use spherical coordinates. The region S can be described by

a ~ p ~ b, 0 ~ B ~ 211" and 0 ~ ¢ ~ 11".

The Jacobian for spherical coordinates is p2 sin ¢, so the integral becomes

l 11
b lr 2lr
!......--3 ¢
p2 sin...:.... dB d¢ dp.
a 0 0 P

Since all of the limits of integration are constant, we can "factor" the integral as follows:

(l b
~) (1'" sin ¢ d¢) (1
2
". dB) = log (~) . 2 . 211" = 411" log ( ~) .

26. T he Jacobian for cylindrical coordinates is r.


(a) After sketching B, we see that B lies over the unit disk (see figure at left, below) and z
extends from 0 to y'x 2 + y2 = r. T herefore ,

21
f f l zdV = 1"'11
2 1 r

rz dz drdB= 1"'1 (r;2[=J drdB

12"'1
o 0
1
-r3 d7' dB
2
=. 12".(r411)
0
-
8 8
2
dB = -11 ". dB = -11" .
4
r=O 0
T HE CHAN GE OF VA RIABL ES FORMUL A AN D APPLIC AT IONS OF INTEGRATION 103

y y

x x

(b) The bound ary x 2 + y2 + Z2 ::; 1 is the same as r2 + z2 ::; 1 (see the sketch of W above).
Solving for 7', we get r ::; ~. Also, the integrand is l/vx2 + y2 + z2 = 1/v'r 2 + z 2.
T herefore, the chan ge of variables gives us

r t
2
7< r~ I' drdzd().
10 11/210 vr2 + z2

Let u = 1'2 + z2 in the inner r integral to get

2"1111--dudzd() 27<11 ( v'ul 1 ) = 1 27< fl (1- z) dzd()


1 1
o 1/2 z ~ 2VU
=
1 o 1/2 u= z~
dzd()
a J 1/2

{ 27r [ (
10 z -
z2 ) ll
2 z =I/2
1d() = "81 10{27< d() ='
1T
4'

27. We want to fi nd a mapping of the unit square B* to the rectangle B. We want to map (0,0)
to (1 , 0), (1, 0) to (4, 3), (1, 1) to (3, 4) and (0 , 1) to (0,1). We want a linear transformation
which maps a parallelogram to a parallelogram . O UI desired m apping is (x,y) = (au + bv +
c,du + ev + f) . Mapping (u,v) = (0,0 ) to (x,y) = (1,0) gives us c = 1. Mapping (1,0) to
(4, 3) gives us a = 3. And mapping (0,1) to (0,1) gives us b = -1. Similarly, we get d, e and
f We get (x, y) = (3u - v + 1, 3u + v). T he J acobian is
- 1
IO(X'Y)I=1 3 1 1=6.
8(u ,v) 3

Y
(3,4)
v
(4,3)

1 u 1 oX

Therefore, the integral gets changed into

..

104 CHAPTER 6

30. In the first octant of JR3, the spherical coordinate B goes from 0 to rr/2. T he desired region of
integration is sketched below. Recall that the J acobian for spherical coordinates is p2 sin ¢, so
the desired integral is

7f/ 21tan-1 (2) 1.../6 (1)


- /sin¢dpd¢dB

1o 7f/4 a P

7f / 2 jtan- I (2) 1.../6 p sin ¢ dp d¢ dB


1a 7f/4 a
7f/21tan-I(2) ( 2
~ sin ¢
1.. /6 ) d¢ dB
1a 7f/4 p=O

7f/21tan-I(2)
=31 7f/2 ( Itan-I(2))
3
1 a 7f/4
sin ¢ d¢ dB
a
- cos ¢
1>=rr/4

dB.

From the triangle below showing ¢ = tan- 1 (2), we get the follo wing:

3 (-
1
vis + Vi.1 o
1 ) r /2
dO =2
3rr ( 1
Vi - v'5 .
1 )

y
1

6.3: APPLICATIONS
GOALS
1. Be able to use double and triple integrals to compute averages, centers of mass . moments of
inerti a and gravitation al potentials.

STUDY HINTS
1. Averages. In general, an average is defined to be the integral of fd ivided by the integral of 1.
Hence, the two average formulas given in this section are
IffII' fdV ffD f dA
and
fffll'dV ffD dA '
2. Cente1' of mass. This is a weighted average. In general, the center of mass of a region in JR n
is (Xl,X2,X3, ... ,X n ), where Xj is
fR x j c5 dV
fR c5 dV
and dV = dXl dX2 ... dX n and R is the region. Since c5 is the density, fR c5 dV is the mass. If
the region is in JR2 use double integrals, and if the region is in IRs use t riple integrals.
THE CHANGE OF VARIABLES FORMULA AND APPLICATIONS OF INTEGRATION 105

3 . Moment of intertta. A good way to remember Ix = IIIw (y2 + z2) 8 dV is to notice that the
integrand lacks an x term. Similarly, Iy and l z lack a y and a z term , respectively.
4. Gmvitational potential. Its formula is V = - OM m/ R. Although you do not ne d to under­
stand the physic 1 t heory behind the discussion the mathematics shou ld be clear to you . You
probably will not need to reproduce the discussion for an exam .

5. Geometry. Recall that if the integrand is 1, then the triple integral IIIw dV gives the volume
of W. Sim il arly, the double integral lID dA gives the area of D.

SOLUTIONS TO SELECTED EXERCISES


3. The fo rmulas used to compute t he center of mass (x, y)

for a region in the plane are

_ IIDxtS(;z;,y)dxdy _ lID ycS(x, y) dx dy


x = lIDtS(x, y) dx dy and y­
- lID 8(x , y) dx dy .

In this case, we sketch the region D and see that it can

be described by

o ~ x ~ 1 and x 2 ~ y ~ x.
x
T he numerator of x is

[11" x(x +y) dydx


Jo rl

T he deno minator is

( l "(X + y) dydx
Jo r'
11[(Xy + y; )[=".] dx

11(3;2 _ ~4 ) x3 _ dx =
(
X3 _
2
X4 _
4
X
10
5
)1 1

0
18
120

Therefore, x = i! . We leave it to you to calculate y = 126'


65

6. Th mass of the plate is lID 8 dA, which is

[21< r 2
Jo Jo (y2 sin 4x + 2) dydx = Jo[27< (y3 )1" {27< ( 3
'3 sin 2 4x + 2y y =O = Jo ~ sin 2 4x + 21r
) dx.

Recalling t hat in 2 t = (1 - cos 2t )/2, we have

1r3
'3 Jo
{ 2" 1 - cos 4x
2 dx
{21r
+ 21r Jo
(x sin 4X )
1r3
dx = '3 '2 - - 8 - a
2

1 "
+ 41r
2
=
1r4
'3 + 41r
2
.

The area of the plate is 21r x 2r. = 4,,2 Thus the average density is mass/ area, or r. 2 /12 + 1.

7. (a) Let 8 be t he density of the box, then the mass i· IIfw If dV , which is

tS Jo
t /2 Jot Jo[2 dz dy dx = If (1 )
'2 (1)(2) = 8.
~. -­
106 CHAPTER 6

(b) Again, the mass is JJJw6 dV , which is


[1/ 2 t [2 2

10 1010 (x + 3y2 + Z +1 )dzdydx

1 10 [ (( x 2 + 3y2 + l)z + z:) [=J dydx


1/2 1

t l2[1 t/2 [ 1 ]
10 10 2(x 2 + 3y2 + 2) dydx = 2 10 ( (x 2 + 2)y + y3 ) Iy=o dx
(X3 + 3x ) 11/2 ~ 2 ( -1 + -3)
2
1
o
1/2
(x 2 + 3) dx = 2 -
3 0 24 2
37
= - .
12

11. The average value of f over W is JJJwf(x, y, z ) dV / JJJw dV. The numerator is

111 2 4 6
2 2
sin 7rZ cos 7rX dz dydx .

Since f(x , y, z ) can be factored so that each factor contains only one variable and the limits of
integrat ion are constants , we get

(1 (1 dy) (1
2 4 6
2 2
cos 7rX dX) sin 7rZ dZ ) .

Using the half-angle fo rmulas, we get

[1 2
C c~s
+ 27rX) dX] [1 d [1 4
y]
6
C- c;s 2rrz) dZ]
6

= 21 [( x + sin
~ 27rX) 102] [ Yo14] 21 [( z - ~
sin 27rZ) 10 ] = 21 (2)(4)2(6)
1 = 12.
The denominator is just the vol ume of W, which is 2(4)(6) = 48 , so the average is !~ = i.
14. T he moment of inertia of Waround the y axis is given by

Iy = f f fw 6( x2 + Z2) dx dydz.
Since W is the ball of radius R, we will use spherical coordina.tes. Since x 2 + y2 + z2 = p 2, we
can rewrite the integrand as 6(x 2 + z2 ) = 6(p2 - y2). T hus , the moment of inertia is

6
[21f
10 10
r [R 2
(p2 - p2 sin ¢ sin 2 B)p 2 sin¢ dp d¢ dB
10
[ 21f r [R
6 10 10 10 (p4 sin ¢ - p4 sin ¢ sin B) dp d¢ dB
3 2

(j 1 21f 1" [(P55 1~=J 3


(sin ¢ - sin ¢ sin B) ] d¢ dB
2

<iR r " r
5
-5- 10 10 (sin ¢ - 3 2
sin ¢ sin B) d¢ dB.

To finish the in tegral , use the identities si n 2 t + cos 2 t = 1 and the half-angle formula sin 2 t =
(1 - cos 2t) /2 . The above integral becomes

<iR 5 [2"
510 10
r [sin¢ - sin¢(1 - cos ¢)sin 2 2
B] d¢ dB
T HE CH AN GE OF VAR IABL ES FORMULA AND APPLICATIONS OF INTEGRATION 107

3
5
r (
6R Jo21r [( - cos if> + sin 2 B cos if> - - 3- fjJ) ) ¢= o dO
-5- COS I" 1
6R51 21r (
-5- 0
4. 2 )
2 - 3'SlD B d8 =-5-
6R51 21f
0
[2 - 3'2(1 - cos2B) ] dB
6R5 [2B_ ~ e + sin 2B ] 121r 86R5 rr
5 3 3 0 15

19. From t he discussion in the text, we know tha t the gravitational potential acti ng on a mass m
lying ou tside ofC .M.W . is - GmM/ R , where G = 6.67 X 10- 11 N·m 2 / kg 2 , m is t he mass of
the smaller object in kg , M is t he mass of the planet C.M.W. and R is t he distance between
the object and the planet's center. When t he object lies outside of a spherical planet , the
gravitational potential acts as if the mass M is concentrated a t the planet's enter. Given the
planet's density as J, we use spherical coordinates to calculate

'rJw J(x,y,z)dxdydz = Jr21r Jor Jr X 108

M = JJ o o
6(p ,fjJ, e)/s in <jJ dpd¢ de.

Since 6 is independent of ¢ and e, we have

Note that 10 12 and 10 8 are insignificant when added to a number with the magnitude of 10 17 .
Therefore , the potential is

v= -(6.67 x 10- 11 )(4.71 x 1019) m/ R = -(3.04 x 109 )m/ R (meter/ second)2

.4: IMP R O P ER INTEGRALS

G OALS
1. Be able to defi ne and compute an im proper in tegral.

STU DY H INTS
l. CompaTis on. One-variable integrals are called improper if the fu n tion becom s infinite at
so me point or if the limits of integr ation are in fini e. The difference here is that a function
m ay become infinite on a curve or surface .
2. Computation. To find an improper integral, compute a limiting value rather than substit uting
the limits directly. Fubini's theorem is valid for improper integrals of positive functions.
3. f becomes infinite. We remind you t hat f m ay become infinite betwe n t he li mits of integration.
In this case, we need to divide the region at the points where fis infinite. For exampl ,

1o1 ]1-Yx dy dx =11 ~Y dy dx + 1111 ~Y dy dx .


1 0

- 1 0 - 1 0 0
• .. .&­

108 (HAPTER 6

See what happens if you evaluate the antiderivative at y = ±1 and do not consider y = 0; you
would get the wrong answer.

SOLUTIONS TO SELECTED EX ERCIS ES


3. Th is integral is improper at x = O. We sketch the region and see y
that it is a y-simple region described by
(1,1)
° ~ x ~ 1 and x/2 ~ y ~ x.
(1,1/2)
Thus, the integral is

[11X~dydx
Jo x/2 X

11 (~: [=x/ J dx
1 x

1(x2- - -X2/4)
- dx -
_113x _ 3x211
- dx - ­
3
1o 2x 2x a 8 16 0 16
6. (a) Assuming D is a y-simple region, it would be reasonable to think of b = 00 as an "endpoint."
Then we can define f fD fdA by
°0 1 ¢>2(X)

a ¢>,(X)
f(x, y) dy dx.

(b) The region D is like the one described in part (a) . Since the limits of integration are con­
stant, we can integrate each variable separately and then multiply the results. The integrand
is factora ble as fo llows:

xy ex p(-x 2 _ y2) = [xexp(-x2)][yexp(-y2)],


so

JI f (x,Y)dA [1 1
yex p (-y2)d Y] [/00 xex p (-x 2 )dX]

(
exp(-y2)
-2 a
11) b-t oo
p
-2
2
lim (ex (-x ) I )
1
b
= (. ~)
4
(! _~) .
e e2

7. Fubini's theorem tells us t hat multiple integration can be performed in any order as long as
the integrand is positive. In this case, e- xy > 0, so we have

/2 1
00

e-XY dx dy = 100 /2 e-
xy
dydx .

For the left-hand side, we compute

rJo[00
J1
e-xy dx dy
1 1
2 (
lim ~
b-too
-xy Ib
y x=O
) dy

/ ,1
2 (
lim
b-too
- -+-
_e­
y
by
1) dy= 12 dy
y 1
- = logy 12
Y 1
~log2.

On the right-hand side, we compute

1a
1
00
1
2
e- XY dy dx = 100 -_e--
o :Z:
XY
12
y=1
dx = 100
0
e-
X
­ e-
2x
dx .

Since Fubini's theorem tells us t he two integrals are equal, we have the desired result.
THE CHANGE OF VARI ABLES FORMU LA AND APP LI CATION S OF INTEGRAT IOI\l 109

9. The integrand sim plifies to 1/(x+y). T he integral is improper whenever y == -x, wh'ich occurs
in our region at t he point (0,0). We compu te

1_] =11
1111+
o 0
_1_ dy dx
x y
= 11[
0
(In Ix + yl)1
y-O
dx
0
[In(1 + x) -In(x)] dx,

which we interpret as

lim
~ --+o+
11 ~
[In (1 + x) -In(x)] dx .
Integrating by parts, we get

E~+ { [((x + 1) In(x + 1) - x) - (x Inx - x)]I: } ·

Using I'Hopital 's r ule, we know hat

·
I1m f
In f = I'1m -
In/f = 0 ,
E--+O+ [ --+0+ 1 f

and so the integral is 2 1n 2.

11. Since x 2 + y2 + z2 appears in the integrand, try using spherical coordinates. In the first octant,
we have
o ~ () ~ 7r/2 and 0 ~ cp ~ 7r/2.
Since x 2 + y2 + z 2 ~ a 2 , we also have 0 ~ p ~ a. Recall that the Jacobian for spherical
coordinates is p2 sin e/; . Subs itute p2 = x 2 + y2 + z2 and p cos cp = z to get

1f/2 1 ff /2 1 Q pl/2.
=l 1f/2 1 1f/2 1 Q p2 sin cp

lo 0 0
----r====p2 sm cp dp de/; d()
J p cos cp + p4 0 0 0 J cos cp + p3
dp dcp d().

Substitute u = cos cp + p3:

1f /2 1 1f /2 j COS ¢+a d u SI·n '+'


'+' dcp d()
lo 0 cos¢ 3y'U
Jo
r/1 0
2 1f

/
2
~ sin e/; ( y'Ul cos ¢+a
3 u= c os¢
3

) dcpd()

1f/2 r/2"3 sin CPJcos cp + a3 dcp d()


2
1r/ r/
o Jo
2 2
2
- Jo Jo 3" sin CP Jcos cp dcp d()

r/
Jo
2
[_i(coscp+a 3 )3/zl1f/ + i(cosCP)3/2'11f/2] d()
9 ¢=o 9 ¢=o
2

27r [(1 + a 3 )3/2 _ a9/ 2 _ 1]


9 .

14. W hen 0 ~ g(:I:, y) ~ f(x, y), integration over D should give us

.lin 0 dA ~ .IL g(x , y) dA ~ .IL f(x, y) dA.

However , since the integral of 9 is sm aller than the integral of f and lID g(x, y) dA does not
exist , we m ust conclude that fiDf (x , y) dA also does not exist.
110 CHAPTER 6

16. Since x 2 + y2 + z2 appears in the integral , we will try to use spherical coordinates. The region
D can be described by
I:=;p and 0 :=; O:=;27r and 0 :=;¢:=; 7r.
In spherical coordinates, the integrand is 1/ p4 and recall that the J acobian is p2 sin r/!. There­
fore, we get
27f 1 7f Joo p2 sin ¢ dp d¢ dO.
o 0 1 1 P
'----;-4-'-

Since all of the limits of integration are constant , we may integrate each variable separately
and multiply the results:

{27f
Jo Jo
f"Joo ~
1
sinriJ
dp d<f; dO =
( { 27f
Jo dO
) (f" . ) ( roo
Jo sm ¢ d¢ Jl p2
dP)

27r ( - cos ¢ I:) b~~ ~ I: ) = 27r(2)(I) = 47r.


( -

SOLUTIONS TO SE LECTE D REV IEW EXERCISES F O R CHAPTER 6


2. (a) This is a linear transformation of a paralellogram, so by t heorem 1 of section 6.1 , all we
need to do is to map the vertices . We compute T(O, 0) = (0,0) , T ( 1,0) = (2, 1), T( 1, 1) = (2 , 4)
and T(O, 1) = (0,3) . The transformation is depicted below:
y, v, (2.4)
(1,1)
(0.3)
T •

(2,1)
r ~
x u
(b) Using the notation shown in t he sketch in part (a), our formula is

J10 f(u, v) du dv = J10. f (u(x, V) , v(x, V) ) 1 ~~~: ~~ 1 dx dy .

In this case, u(x , y) = 2x and v(x, y) = x + 3y, so

a(u , v)
1a(x, y)
1 = 1 au/ox
av/ax
au/ay
av/ay
1 = 1 21 °=
3
1 6
.
Thus, our formula is

J10 f(u, v) du dv =6 J10. f(2 x, x + 3y) dx dy.

4. (d) Recall that t he Jacobian for spherical coordinates

is p2 sin qS and sin 2qS = 2 sin qS cos qS. We factor out

the Jacobian from the integrand and we are left with

2p cos qS, which converts in rect angul ar coordinates to

2z. From our sketch of the region , we see that the

region of integra tion is over the disk of radius ~, cen­

tered at the origin, so we have

-I/V2 -< x <


-
I/V2 and -J~£ ~ x-2 -< Y <- J~2 -
x2.
y

x
T HE CHANGE OF VARIABL ES FORMU LA AND APPLICAT IONS OF INTEGRATION 111

Now z extends from the cone to the sphere of radius 1. The equation of the cone is z = x 2 + y'l
and the upper hemisphere has the equat ion z = x 2 - y2 so / 1-

T herefore, the integral is converted to

]
1/-/2 ] v'1/ 2 - E~ l v'I-X 2 Y2
­
2z dz dydx.
-1/-/2 -Jl/2- x' E' +y'

5. The desired region is shown here. The surfaces inter­


sect where z = x 2 + y2 =2 - z2 or z2 + z - 2 0 =
or z = 1. We disregard z =
- 2 as a sol ution z
since we want z ~ O. Looking at our drawing, we
can describe it in cylindrical coordinates as follows .
Notice t hat the entire region lies over the unit disk.
Also, the region lies between z = x 2 + y2 = r2 and
= /2 -
z x 2 - y2 = -J2=T2, so the r gion can be
described by
y
o~ r ~ 1, 0~ e ~ 27l'
and

Since the Jacobian for cylindrical coordinates is r, we get

8. For convenience, we will rotate C 2 so that its axis lies on the z axis. T he volume remains
unchanged. Using the res ul t of exercsie 12 in section 5.5, we substitute a = 1 since the
diameter is 2 and the radius is 1. Thus, the answer is \6 .
11. When the cu t is made by the plane x + y + z = a, the
volume of the (shaded) solid below t hat plane is

r
fa a- x f O- x- y
io io io dzdydx =
fa fO-x
io io (a- x -y)dy dx

r [( (a -
= io x)y - ~2 )\ Illy=o
_Xl dx
r (a~ x r- dx .
= io
1
y

Substitute u =a - x to get

T hus, the volume for the entire solid, when a = 1 is i.


If the solid is to be cut into n equal
volum es, then t he volume under ;I: + y + z =
a should be k / 6n , where k is an integer such
that 1 ~ k ~ n - 1. Therefore, a3 /6 ::::: k/6n implies that cuts should be made in the planes
x + y + z = (k/ n )1/3.
112 CHAPTER 6

15. Let u = x + y, v = y - x or x = (u - v)/ 2, y = (u + v)/2. The reader should veri fy that the
J acobian 18(u, v)/8(x, y)\ is ~ . From the fig ures, 0 ~ u :S 1 and -u ~ v ~ u.

v y
(1,1)

u
> ~

(1,-1) x

T hen

Jl exp [~ ~ : J dx dy tjU
10 - u
exp(v/u) dv du
2
=~ t
2 10
u ( ev/u l
u
v =-u
) du

~ (e - ~) 11 du = ~ (e- ~) .
U

16. T he tot al mass is the integral of the density. Since we are integrating over a sphere , use
spherical coordinates. The sphere of radius R is described by

o ~ P :S R, 0 ~ r/J ~ 1r and 0 ~ 0 ~ 21r.


T he dist ance d from the origin is the same as p and the Jacobian is p2 sin ¢. T herefore , the
total mass is
2" 1"lR p2 sin r/J
3 dp dr/J dO .
'----=-
1 o 0 0 l+ p
Since all of the limits of integration are constants, we can integr ate each variable separately
and multiply the results. We get

2
(1"dO) (1" sin r/J dr/J) (1 R1:2p3 dP) = 21r [ - cos¢ I:J [ ~ 1n(I+ p3)Cl = ~1r ln(1+R2).
19. (a) Let T( x, y, z ) be the temperature at (x , y, z ). T hen the average temperature is
fffe TdV/ fffe dV . By definition, d = J x 2 + y2 + z 2, so d2 = x 2 + y2 + z 2 and T(x, y, z ) =
32(x 2 + y2 + z2). T he numerator is

32 [ 1 [ 1 [ 11(x 2 + y2 + z2) dx dydz 32[:[ 1 [(~3 + + z2 )x ) [=-J dydz


(y2

64[11[11(~+ y2 +Z2 ) dy dz
64[ 1{ [(~ + z2 ) y+ Y:J[=-J dz
1

128 j 1(2
-1 3 + z 2) dz = 128 (2
"3Z+ 3Z3 ) 11 -1 = 256.
T he denominator is just the volume of the cube C, which is 8, so the average temperature on
C is 2~6 = 32.
(b) T he average temperature is attained wherever 32d 2 = 32, =
i.e., d l. Thus the average
temperature is att ained at all points of the cube on a sphere of r adius 1, centered at the origin .
THE CH AN GE OF VARIABL ES FO RMU LA AND APPLICATION S OF INTEGRATION 113

t
20. The inequality y2 + z2 ~ describes the inside of a circular cylinder of radius centered on t
the x axis . T he inequali ty (x - 1) 2 + y2 + z2 ~ 1 describes a ball ofradi us 1 centered at (1, 0, 0).
We also want x ~ 1, so we get he region shown. By symmetry, we easily see that fi = z = O.
For conven i nee , we will shift the region in question so that its axis of symmetry is the z'
axis and we will place the ball's center at the origin . For the center of mass of the new region
D+ , it is the integral of z· over D" divided by he volume of D- . T h region D* is described
in cylindrical coordinates by

o <- 7' -< !2' 0 <_ B _< 211" and 0 < z· ~.


< V! - r~

y z
.

-1/2 112

The Jacobian is r, so the integral of z · over D· is

211"1 1/21 Yl-rl r z" dz· drde = r 1l" [ 1/2 (r(z.) 2 1 ~) dr dB


1o 0 0 Jo Jo 2 z. =O

27r1 1/2(r- -2 r- 3)
drde
1
o 0

11
-
2 0
2
". [(r2- - -r4 ) 11/2] dB -_ - 7 1211" de -_ -711".
2 4 r=O 128 0 64
The olu me f DO is

1
27r1 l/2 1~ 211"1 1/2 ( rz· I~) dr dB
r dz· drdB
o 0 0 1o 0 z · =O

1 211"11/2 rvh - r2. drdB = 2" 1 [-(1-r2 )3/2Il /2] de


o 0 0 3 r=O

( !3 _ v'83) Jo[2'" dB = 211" (! _.;3)


3 8 .

D* was shifted down t uni t, so for the original region, x is

24. (a) Since we are integrating a spherical region , we will use spherical coordin tes and the
Jacobian is p2 sin ¢. We get

ffl (:1: 2 + y2 + z2 )xyz dV = 1"1"'l


2 R
p2(p cos {Jsin 4J )(p sin esin ¢ )(pcos ¢ )p2 sin ¢ dpd¢ de
114 CHAPTER 6

( r. r [R
10 10 10 lcosOsinOcos¢Jsin3¢Jdpd¢JdO.
Since all of the limits ofintegration are constants, we can integrate each variable separately
and multiply the results. Thus, the integral is

(l R
p7 dP) (1
27r

COS 0 sin 0 dO) (1 cos ¢J sin ¢J d¢J )


7r
3

( p; C)( 0C) ( sin


2

2
sin: ¢J [) ~ O.
28. Since x 2 +y2 +z2 appears in the integrand , we will use spherical coordinates, and the Jacobian
is p2 sin ¢J. To integrate over all of ~a , we will integrate over a sphere of infinite radius. Thus,
we describe ~3 by
o ~ p < 00, 0 ~ ¢J ~ 1T, 0 ~ 0 ~ 21T .
Therefore,

JJ (Xi r [21f p2 sin ¢J


!a /(x,y,z)dXdYdZ 10 10 10 {,. ~\~/" dO difJ dp
1f 21f
~3)3/2 dP) (1 sin ¢J d¢J) (1
00

(1 (1 + dO)

(1 :2:~3/2) ( - cos ¢JI:) ( oe)


00

(1
00 p';idp
41T
1
+ o (1 p3)3/2
.

Substitute u = 1 + p3 to get

41T
100 1
du
-3-2
3u /
= -41T.hm
3 b-+oo
1b 1
u-
3/"
~ du = -41T3 b-+oo
.
hm (-Ju)I: = 81T/3.

31. (a) This improper integral is evaluated as follows:

1001 100 (2
11 x2 Y
x exp ( _y3) dx dy 1 ) exp( _y3) dy
o 0 0 x=O

11
-
2
00

0
y2 exp( _y3) dy = lim
b-+ oo
(exp ( _y3)
-6
b
I
0
)
1
- 6'

33. Notice that x 2 + y2 occurs in the integrand , so we will use polar coordinates, which we recall
has Jacobian r. Consider ~ 2 to be a disk with infinite radius, so ~2 is described by

o ~ rand 0 ~ 0 ~ 21T.
Thus,

J~ ~ ! (x, y) dA
[21f
10 10
[00

r ~. -' ~ dr dO

-bl~n~(1+r2)-1/21~=J dO = 10r
2
r.
121f [ dO = 21T.
THE CHAN GE OF VARIABLES FORMULA AND AP PLI CAT IONS OF INTEGRATION 115

TEST .F OR C HAPTER 6

L True or false. If false, explain why.

(a) In polar coordinates, the volume bounded by the paraboloid z =a - (x 2 + y2) with a > °
°
and the plane z = is given by t he integr al

1""l
2 Fa
r(a - r 2) dr dB =-
rra2.
8
00

(b) T he xistence ofthe improper integral J1


2Jooo co:x dx dy is guaranteed by Fubini 's theorem.
(c) When changing fro m Cartesian coordinates to pol ar coordinates, we have

= J ,,"/6 [ 2 r2 cos B dr dB.


1-l1lorv'4-y2 x dx dy
-,,"/6lo
(d) The volume of the unit sphere centered at the origin with the cone z :=:; J x 2 + y2 deleted
IS

[ 211" [ 1
lo lo l1f/4
r p2 sin if; dif; dp dB.

(e) The parallelogram with vertices at ~2, 2), (4,3), (0,5) and (2,6) can be transformed to the
square [0,1] x [0 , 1] by T (x, y) -+ ( x+2l -10, -x+;y-2) and its corresponding Jacobian is
1

2. Compute the integral of 1/(1 + x 2 + y2 + x 2 y2) over all of JR 2 • (Hint : Factor the denominator.)
°
3. A region in space is described by :=:; x :=:; 3, - 2 :=:; y :=:; 2 and
coordinates to om pute the average of z over the region .
°:=:; z :=:; ~. Use cylindrical
4. F ind the volu me of the region which lies between the surfaces z = 10 - x 2 - y2 and
z = _( x 2 + y 2 ) 2 and which lies within the cylinder x 2 + y2 = 4.
5. Let S be the transform ation defined by S : (x , y , z) -+ (u 2 , uw, V + w). Suppose this transfor­
mation takes D into the region [0,1] x [0 , 2] x [-1,1]. Compute JJJD(y + z) dx dy dz.
6. As wit h parallelogra ms, a linear ran form a tion will map vertices of a parallelepiped to vertices
of another parallelepiped. Let T be the transfo rmation T : (x , y, z) -+ (3u + 2v + w, 2u ­
w , v + 2w) and let P be the parallelepiped with vertices at (0,0,0)' (3,2,0), (2,0, 1), (1, -1,2) ,
(5 , 2, 1), (3, - 1,3), (4,1,2) and (6 , 1,3).
(a) Convert JJJp (x - y - z) dx dy dz to tJvw coordinates. [Hint : Map the points (x, y, z) ==
(0,0,0) , (3, 2, 0), (2,0,1) and (1,-1,2).]
(b) Evaluate the triple integral.
7. Integrate x + 2y over t he interior of the ellipse (x - 2)2/4 + (y - 1)2/9 = 1 by making an
appropriate substitu tion and then using polar coordina tes.
8. The shap of a swimming pool can be described by °
:=:; x :=:; 2, ~ 3Jl - x 2 /4 :=:; y :=:;
3J 1- x /4 and - J 1- x /4 - y2/9 :=:; z :=:; O. Compute the volume by first substituting
2 2

u = x/ 2 and v = y/ 3 and then changing to spherical coordinates.


9. Explain what is wrong with this calculat ion:
Y
[lJY"!. dx dy = lot
lo -y x
In Ixll _
X_-Y
dy = [ 1(In lyl- lnIY!) dy =
lo
to dy = 0.
lo
10. A near-sighted mosquito saw a t a ttoo on a man's arm which appeared to have the combined
form of a rect angle, [- 2, 2] x [0,1], and a semiellipse described by x 2 / 4 + y2/9 :=:; 1, y:=:; 0.
Where should the mosquito aim its probiscus to consume blood out of the tattoo's center of
mass? You may assume t hat the mosquito sees a planar surface and t he tattoo has a constant
density.
117

7 INTEGRALS OVER PATHS AND SURFACES

1.1: THE P ATH INTEGRAL


G OALS
1. Be able to compute a path integral.

STU DY H I N TS
1. Not ation. T he path integral is denoted by Ief ds, where ds = Ile'(t)lldt and e(t ) is a path .
2. Definition. The path integral is the integral of real-valued functions over a curve. The integrand
f only needs to be defined on c and f must be piecewise continuous on e. In addition, e needs
to be piecewise continuously differentiable. T he integration of vector-valued functions will be
discussed in section 7.2.

3. Computation. If e(t) is defined for a ~ t ~ band f is defined on e, then the path integral is

lb f( c(t )) lie' (t)11 dt.

4. Relation to arc length. If f = 1, then the path integral is the arc length formula of section 4.2.

SOLUTIONS TO SELECTED EXER CISES


2. (b) The path in egral is defined by

lb f(e(t)) Ile'(t)11
1dt.

In th is case, f( e(t)) = cos t , e' (t) = (cos t, - sin t, 1) and lie' (t)11 = [cos 2 t + sin 2 t + 1P/2 == V2.
T herefore, the path integral is

1 2..
cos t . v'2 dt = v'2 sin t
1271'
0 == O.

3. (b) Let x = t, Y = 3t and z = 2t. Here, f = yz = (3t)(2t) and ds = Ile'(t)11 = [12+3 2 +22 P / Z =
V14. Then the path integral is

1 =Jf ds
3
(3t )(2t)V14 dt "'" 6v14 j 3 2
t dt ::::; 2v14t 3
1
3

1 = 52Ji4.

6. (a) Recall that in polar coordinates,

x = r(O) cos 0 and y = r(O) sin O.

Heie we treat r as a function of 0, and so

dx = (r'(B) cos B- r(B) sin B) dB and dy = (r'(B) sin 0 + r(B) cos 0) dB.
118 CHAPTER 7

Also, we know that

ds = V dx 2 + dy2 dO
= ([7·/(0) cos 0 - r (O) sin (W + [1'1(0) sin 0 + 1'(0) cos 0]2}1 /2 dO
= + +
[( 1" (0) )2(cos 2 0 sin 2 0) (i (0))2(cos2 0 sin 2 0)j1 /2 dO +
= Vi2+ (dr j dO) 2dO .
Hence, t he path integral of f (x , y) = f (1' cos 0, rsin 0) is

1 1(x , y) ds = 1~2 f(r cos e, l' sin O)Vi


2
+ (~~ r
de .

7. (a) From t = - 1 to t = 0, the path is a straight line going from to (0,0,0); from t =(1,1,1)
to t = 1, the pat h is a straight line going from (0 , 0, 0) to (1,1, 1), so act ually the same path
°
was covered twice. Hence, the path integral is

lb f (x (t ), y(t ))y'( x'(t ))2 + (y' (t) )2 dt = 211 (2t4 - t 4)V {4t 3)2 + (4t 3)2 dt

= 211 4t 4Y'2t6 dt = 8h11 t 7 dt = h.


The answer could be thought of as the area that needs to be covered if one wants to paint the
= =
two sides of a fence erected along the path x t4 , Y t4, of height f (x (t), y( t)) .
(b) The arc length function is the pat h integral when f = 1, so it is

lb f( c(r))llc'(r)ll dr ,

where a is the st arting point. To evaluate s(t ), we need to break up the interval into [-1,0]
and [0 , 1] . For - 1 ~ r ~ 0, Ilc'(r)1I = [( 41"3 )2 + (4 r 3) 2j1 /2 = 4V2I1"13 = -4y1r 3. Then

s(t) = - [tl4V2r 3 d1" = - V2(t 4- 1) .


T hus , t 4 = - s j y1 + 1 and 4t 3 dt = - ds/ y1. When t = - 1, s = 0; and when t = 0, s = y1.
For °:: ; r ~ 1, Ilc'(r)11 = =
[(41"3)2 + (41'3) 2]1 / 2 4y1 r 3. We also need to add the length from
- 1 to 0, so we get, for °:: ;
t :::; 1,

s(t) = V2 + 1t4V21"z d1' = -/2(t 4 + 1).


So t 4 = s/V2 - 1 and 4t 3 dt = ds/y1. When t = 0, s = V2; and when t = 1, s = 0. Putting
all this together, the path integral becomes

1V2 (- ~ + 1) ds+J~ (~ -l) ds =(- 2~ +s)l: + (2~ - s) I~~h.


10. (a) W hen density is a constant k, we have

Mass =k x length of wire.

The length of the wire is

lbIlc'(O)11 dO ,
INTEGRALS OVER PATHS AN D SURFACES 119

so the mas is

(b) Recall th at a coordinate of the center of mass, Xj, is

Here, we have a thin wire in the yz plane. By symmetry, x = z = O. In this case, y = a sin 0,
so the center of mass for y, with k = 2, is

f ky ds
-f--
k ds
=
fo" a sin (J • 2a dO
2a7r
_ a
- -
71'
1" .
0
0 _ 2a
smOd - - .
71'

Thus, the center of mass is (0, 2a/ 7r, 0) . Note that the center of mass is not on t he wire.

13. The t rick of this xercise is to use the correct coordinate system. T h intersection of the
sphere and th plane is a unit circle, but in an unfamiliar orientation. T he unit normal vector
of the plane x + y + z = 0 is (i + j + k) / J3, so we need to find two orthogonal vectors in the
plane x + y + z = O. For example, (1,0, - 1) and (1, - 2, 1) are a pair. Normalization of h e
two vectors yields u = (1,0, - 1)/0 and v = (1, - 2, 1) / V6. T he circle is now param etrized
as c(B)= =
(x, y z) cos Bu + sin Ov, 0 ~ 0 < 271'. Take the first component of c to get
x = (cos B) / /2 + (sin B)/ )6. Then the mass is

1x 2
ds = 1 (~coso + ~sinor Ilc' (B)l ldB
2

"

1 (~ + ~
2
" cos B sin B) 2 . 1 dB

1 (12 + 2 v'i2B 0+ 6 B)
o
2 ..
cos
20 sin cos 1.
ill
2
dB .

You should verify that Ilc'(B)11 = 1. Now use the half-angle formulas to get

7.2: LINE IN TEG RALS


GOALS

1. Be able to compute a line integral.

2. Be able to explain the difference between a line integral and a path integral.

STUDY HINTS

l. Definition . The line int gral deals with vector fields whereas the path integral dealt with
scalar fu nctions. It is required that c be Cl and F be continuous on c (or at least piecewise
continuous for both c and F) .

2. Physical interpretation. The line integral is most commonly interprete as the work performed
by F along a path.
120 CHAPTER 7

3. Computation. If F is defined on c and c is defined for a ~ t ~ b, then the line integral is

lb F(c (t )) . c' (t ) dt.

Note that the integrand is a dot product, so the integral is a scalar . T he formula involving
T (t ), the unit tangent, may be more difficult to use since you need to compute T (t) as well as
Ilc'(t)ll ·
4. Sign interpretation. Posi tive work means that t he force fi eld did a net amount of work on t he
object; such is the case when the motion of the object is in the direction of the force. If work
is negative, then this amount of work is done by the object on the force field.
5. Reparametrization and orientation. If c is reparametrized and the orientation is preserved,
the value of a line integral does not change. If t he orientation is reversed, only the sign
of the value of the line integral changes. You can substitute for t he end points to be sure the
direction is correct. Orientation is not important with the path integral of the previous section.
Reparametrization allows us to break up a curve and integrate over each segment separately
and with convenient parametrizations (see example 11 ). Be sure the path is traversed the
correct number of times.

6. Gradients. The line integral of a gradient field depends only on the endpoints . We will see
how to make use of this fact in section 8.3 . Given this fact, we can see tha t t he line integral
of a gradient along a closed path is O.

SOLUTIO N S TO SELECTED EXERCISES


1. (a) T he line integral of F along cis

11 F (c(t)) . c'(t ) dt .

We have c(t} = (t, t, t), so c'(t) = (I , I, 1) and F( c(t )) = (t, t, t ). Therefore, the desired line
integral is

11 + +
(ti
o
tj tk ) . (i + j + k) dt = 11+ +
0
(t t t) dt 3 2
= _t
2
1

0
= -3 .
2
2. (c) We need to break up the integral into two parts. For the segment j oining (1, 0, 0) to (0, 1,0),
th e easiest way to parametrize the line is to find c(t) = u(t) + tv (t ) such t hat c(O) = (1, 0,0)
and c (l) = (0,1, 0) . Letting u(t ) = (1, 0,0) and v (t ) = (0 ,1, 0) - (1,0,0) = (- 1, 1, 0) gives us
C1 (t ) = (1, 0, 0) + t (- 1, 1, 0) or (x, y, z ) = (1 - t, t, 0) for 0 ~ t ~ 1, so dx = -dt, dy = dt and
dz = O. Substitute these values to get

1 Cl
(yz dx + xz dy + xydz ) = 110
[t · 0· (- dt ) + (1 - t) · 0 · dt + (1 - t) . t · 0] = O.
For the line segment joining (0, 1, 0) to (0 , 0,1 ), use the same method . We find C2 (t)
(0 ,1, 0) + t(O , - 1,1) or (x, y, z) = (0 ,1 - t, t ) for 0 ~ t ~ 1, so dx = 0, dy = - dt and dz = dt.
Substitute these values to get

1 ~
(yz dx + xz dy + x y dz ) = { I [(1 - t) . t ·0+ 0 . t . (- dt ) + 0 . (-t) . dt] = O.
h
Adding the two line integrals together, we get 0 + 0 = O.
4. (a) By definition, we have
1 F· ds = 1 F( c(t) ) . c'(t) dt .
INTEGRALS OVER PATHS AND SURFACES 121

We are given that F and e' are perpendicular, so F· e' (t) = O. T herefore, the line integral is O.
(b) Again, we use the definition of the line integral. In addition , we use the definition of the
dot product to get

1 1F · ds = F(e(t )) . c'(t ) dt = l IIF (e(t ))lllle'(t)II' cos(O) dt = l IIF " ds.

This result because the cosine between parallel ve tors is 0 and by definition ds = Ilc'(t )11 dt.

7. We are given that c(t) = (x,y,z ) = (t,tn, O), 0 $ t $ 1, where n = 1,2, 3, .. .. Differentiation of
each component gives us c' (t) = (1, nt n - 1 , 0) , so dx = dt , dy = nt n- 1dt and dz = O. Therefore,

1[ydx+(3 y3 - X)d y + z dz1 = fa \ tn dt +(3t3n _t) ntn -1 dt+0)

fa 1(t n + 3nt 4n - 1 - ntn ) dt

( 1l +- nn tn+1+ ~t4n)ll
4 0
3 n -l
4'-I+n '

11. Given that e(t) is a path and T is a unit tangent , we have

T=~
Ile' (t) 11

by definition . Since v · v = Il v 11 2, we get the line integral

r r e'(t)
Jc T · ds = Jc Il e' (t )11
,
. e (t) dt =
rlIIle'(t)112
Jc e' (t)11 dt =
l '
c li e (t )/I dt .

The last integral is simply the arc length of c(t).

14. If a vector field is a gradient, then the integral only depends on the endpoints. For a closed
curve, the starting point is the same as t h ending point, so theorem 3 tells us that t he integral
is O.

17. From the fact that V(I/7') = - r/ r3 , we see that F = Vf , where f = l/ r. According to
theorem 3, the line integral is

f(c (b)) - f (e(a)) = -R1 :!


1
- -R .
1

7.3: PARAMETRIZED SURFACES


GOALS
1. Be able to parametrize a given surface.
2. Be able to determine if a surface is regular and /or differentiable.
3. Be able to compute a tangent plane for parametrized s urfaces.

STU DY HINTS
1. Parametrized surfaces. Recall that curves in the plane can be parametri zed by wo functions
of one variable, namely, x = =
f (t ) and y g(t). A curve in space is given by t hree functions
of on variable. Now, we extend this idea to surfaces. We do his by letting J,', y and z be
fune ions of two variables, i.e., now we hav t hree fu nctions of two variables, x = f (u , v),
y = g(u, v) and z = h(u, v).
122 CHAPTER 7

2. Differentiable surfa ces. Such a surface is parametrized by (f( u, v) , g(u, v), h (u, v)) and j, g and
h are all differentiable functions.

3. Review. Recall t hat a vector n which is normal to a plane gives the coefficients of the equation
of that plane. See Section 1.3.

4. Tangent vectors. Like a partial derivative, we can hold u constan t to get T I) (fl) , g", hI) ).
Likewise, holding v constant gives us Tu =
(fu, 9u, hu ).

5. Tangen t plane. By holding either parameter, u or v, constant, we get a curve on the surface
and if the cur ve is "nice," it will have a t angent line. Sim il arly, we can get another t angent
line by holding the other parameter constant. Those two tangent lines determine the tangent
plane, and t he normal vector to the plane is Tux T".

6. Regular. A surface is said to be regular if Tux T" f:. O. When the cross product is not
0, a tangent plane exists and consequently, the surface has no pointed regions (li ke a cone).
Differenti ability does not imply regular (see exercise 1i ), but regularness requires differentia­
bility, for otherwise T u and T" would not be defined.

7. Importan t para m etrizations. Know the following parametrizations:

(a) Circle of radius r: x = r cos O, y= rsine, 0 ~ f) ~ 21T.


(b) The hyperbola x 2 - y2 = 1: x = cosh t, y = sinh t.
(c) Sphere of radius p: x = p cos Osin ifJ , y = p sin BsinifJ, z = pcosifJ, 0 ~ 0 < 21T, 0 ~ ifJ < 1T.
This is the same as the definition of spherical coordinates.
(d ) A surface z = g(x, y ): x = u , y = v, z = g(u , v ).

8. Normal to a graph. If z = ! (x, y), the normal to the graph is (- ~, - is useful %&' 1). This
to know for the coming sections. Note that if we let g = z - !(x, y), then the (unnormalized )
normal to the surface is n = 'il g.

SOLUTIONS TO SELECTED EXERCISES

3. The normal to the desired tangent plane is Tux T I) ' We compu te T u = (/Jx / /Ju , /Jy/ /Ju , /Jz/ /Ju) =
(2u , sin e" , ~ cos e" ). Simil arly, T" = (0, e"u cos e V , - ~evusi n e" ). We compute

i j k
Tu x Tv 2u sin e" 13 cos e"
o ue" cos e" (-u / 3)e" sin e"
2
(( - u/ 3)e" sin e" - (u/ 3)e V cos 2 eV)i + ((2u 2 / 3)e" sin eV )j + (2u 2 e" cos e" )k
(u/ 3)e" (- i + 2usine"j + 6ucose"k).

To find the tangent plane, we only need to evaluate - i + 2u sin e" j + 6u cos e"k at t he given
point (uo,vo) . However, we are only given (ZO, yo, zo ), so we must be clever . We know th a
usine" = - 2 and (u/ 3) cose U = 1 at the given point , so 2u si n e" = - 4 and 6ucose" = 18.
Hence, the tangent plane is

-(x - 13) - 4(y + 2) + 18 (z - 1) = 0, or 18z - 4y - x = 13.


4. (1) A surface is regular ifTu x T" f:. O. We calculate Tu = 2i + 2uj +Ok and T" = Oi + j+2vk,
so
j k
Tu X T" =I 2 2u 0 I = 4uvi - 4vj + 2k,
o 1 2v
INTEGRALS OVER PATHS AND SUR FAC ES 123

which is never O. Therefore, the surface is regular.


=
(2) Again , we calculate T ... 2ui + j + 2uk and Tv = - 2vi + j + 4k, so
i J k
T ... x Tv = 2u 1 2u = (4 - 2u)i - (8u + 4uv)j -+ (2u + 2v)k.
- 2v 1 4

If u = -v = 2, then T ... x Tv = O. Therefore, the surface is not regular at (0,0, -4).


7. From t he partials of x, y and z with repsect to tI , we compute T ... = j, and similarly, Tv
(cos v) i - (sin v)k. T hen
j k
T ux T v = 01 0 =-(sinv)i-(cos v)k.
cos v 0 -sm v

By our luck, the magnitude of this vector is 1 already, so a unit normal is n = -(sin v)i ­
(cos v )k. [Note that another unit normal is (sin v)i+(cos v)k, depending on which cross product
(T ... x T v or 1\ x T... you take.]
To identify the surface, note that x 2 + z2 = sin 2 v + cos 2 V = 1. As v varies between 0 and
21l" , this becom s a circle of radius 1 centered at the origin on the xz plane. Since y = u, we
have -1 ~ Y ~ 3. Therefore, the surface is a cylinder of radius 1 centered around the y axis
and it extends between y = -1 and y = 3.
9. (a) Parametrically, the surface is described by x = h(y , z), y = y, z = z . In this case,
Ty = (hy, 1, 0) and Tz = (hz, 0,1), so
1 j k
Ty x Tz = hy 1 0 = (1, -hy, -h z ) .
hz 0 1
The vector T y x T z is normal to the tangent plane, and so the tangent plane is

(x - xo) - hy(Y - yo) - hz{z - zo) = 0,

where hy and hz are evaluated at (xo, Yo, zo).

11 . The surface is parametrized by (x, y, 3x 2 + 8xy), so Tx = (1,0 , 6x + 8y) and Ty = (0,1 , 8x ).


At t he point (1 , 0, 3), T x = (1,0,6) and Ty = (0, 1,8). T he normal vector is given by

j k
n = Tx x Ty = 1 0 6 = -6i - 8j + k.
018

Thus, the desired equation is (x - Xo, Y - Yo , z - =0) . n =


0; that is, 0 (x - 1, y - 0, z - 3) . =
(-6, -8, 1) = -6x - 8y + z + 3, i.e., z = 6x + 8y - 3.
T hinking of the surface as the graph of t he equa tion f( x, y) = 3x 2 + 8xy, we calculate
Oflox = 6x + 8y , ofloy = 8x , which become 6 and 8, respectively, at the point (x , y) = (1,0).
Then the tangent line is z - 3 = 6(x - 1) + 8(y - 0); that is, z = 6x + 8y - 3.
13. (a) W hen 0 = 0, 4>(1',0) is the line segment from (0 , 0,0) to (1 , 0, 0) . As 0 increases, the line
segment rotates around the z axis and the segment also moves to the height z = O. Thus , we
get the sketch of a helicoid . The sketch in figure 7.4 .2 shows a helicoid for 0 ranging from 0 to
21l". T he helicoid for 0 ~ 0 ~ 41l" is similar except the graph extends to z = 41l" and makes an
extra revolution around the z axis.
(b) Differentiating the components of <I» (r, 0) gives us Tr (cosO,sinO,O) and
To = (- r sin 0, r cos 0, 1) . A normal vector to 4> is
1 j k
Tr x To= cos 0 sin 0 o = (sinO,-cosO, r).
- rsinO r cosO 1
124 CHAPTER 7

Bence, t he unit normal vector is (T r x T9) /II T r x T 911 = (sin B, - cosB , 7')/~.
(c) Multiply the vector Tr x T o calculated in part (b) by r to get another normal vector:

(rsinB, - rcosB,r 2) = (y,-x, x 2 +y2).

Then the equation of the tangent plane at (xo , Yo, zo) is

Yo(x - xo) - xo (y - Yo) + (x~ + Y6)(z - zo) = 0.


17. (a) It is obvious that the imageof~ l is the xyplane (or the uvplane, if you prefer) . Since u 3
and v3 can have any real value, and the third coordinate is 0, the image of ~2 also is the xy
plane.
(b) A surface is not regular is liT" x T vll = 0. For ~1' we have T" = i and Tv = j, so
li T " x Tvll = Ilkll = 1. Therefore, ~ 1 descri bes a regular surface . For ~ 2' T" = 3u 2 j and
°
Tv = 3v 2 j. If u = or v = 0, then T" x Tv = o. Bence, ~ 2 is not regular. (T his problem
illustrates t hat "regularness" depends on parametrization, not necessarily the image.)
(d) The answer is no. A "regular" parametrization cannot "round out" the "corners" of a
graph. Indeed, by (c), a regular parametrization gives a smooth surface.

7.4: AREA OF A SURFACE


GOALS
1. Be able to find the surface area of a given surface.

STUDY HINTS
1. Riemann sum argument. If you understand the Riemann sum argument , you should be able
to deri ve the surface area formula.
2. Form ulas. You should know at least one surface area for mula. One is

A (S ) = J10 li T" x Tv ll du dv.

Equation (3) follows immediately from this formula, and (4) is a special case of (3). If you
choose to remember equation (3), note that the integrand has the Jacobian matrix of each
possible pair chosen from (x, y, z), i.e., (x, y), (y, z ) and (x, z ).
3. One-lo-one. J ust as with curves, the chosen parametrization must be one-to-one; otherwise,
the surface may be covered more than once. See exercise 2.
4. Su r/ace area 0/ a graph. If z = !(x, y), use the parametrization x = u, y = v and z = !(tt, v).
Then the surface area formula becomes

A (S) = J10 /11 + /~ + 1 dA.

T his is important to know or you should be able to derive it from ffD li T" x T vll dudv .

SOLUTIONS TO SELECTED EXERCISES


2. Using formula (3), we compute the following determinants :

8( x, y) 8x / 8B Bx/ B¢ I_I -sin Bsin ¢ cos Bcos </> / _ . ¢; </>


8(B, ¢) /
By/8B By/ B</> - cos 8 sin¢ sinBcos ¢ - -sm cos,
8(y, z)

8(B, ¢) /
° _
cos 8 sin ¢ sin Bcos ¢ / _
sin ¢
. 2 A, (j
- - sm If' cos ,
INTEGRALS OVER PATHS AND SURFACES 125

and

8(x,z) _ 1 - sinBsin¢J cosOcos¢J 1 _ . 2 ", . l1

8( e, ¢J) - 0 - sin ¢J - sm 'I' sm (l.

Squaring the above determinants, adding them and taking the square root gives us
4 2
liTe x T 1>11 = j sin 2 I,b cos 2 ¢J + si0 tP(cos 2 B + sin B) = j Sin 2 ¢ (cos 2 tP + sin 2 I,b ) = 1sin tP l·

If we allow ¢J to vary from - 7r/ 2 to 7r/2, then we get

1
2'1fl1f/2
o - 1 " /2
Isin 4> 1d¢ dO = 27r [1 0

- 'If/2
(- sin¢J ) d¢J + r/ SintPdl,b]
io
2

27r [ cOS¢J IO + (.- cos 4» 1'If/ 2] = 47r.


- 'If/ 2 0

T he answer is 47r because when 4> varies from -7["/ 2 to 7r/ 2, t he parametrization covers the
upper hemisphere twice, so the surface area is the same as one compl te sphere .
If we vary I/; from 0 to 27r we should get 87r , since the sphere is parametrized twice:

1 1 211"
2
1[ Isin ¢ I d¢J dB = 27r [111" sin tP d¢ + 12'1f(- sin ¢J) d¢]

27r[ (- cos¢J)I : + costP C ] = 27r(2 +2) = 87r.

If we had fo rgo ten the absolute values and simply integrated sin 1/;, we would have computed
a surface area of 0 in both cases, but tills is not possib le. Surface areas have to be positi ve .
5. The ar a of of/ (D) is lID II Ttl x Tv II du dv. Given of/(u, v ) = (u - v, u + v, u'v) , we get

8(x, y) = I 1 1 1= 2; 8(y, z) = 1- 1 1 1= - u _ v; 8(x, z) = 11 1 1= u - v.


8(u, v) -1 1 8(u, v) v u 8(u, v) v u
Squaring t he de terminants, adding them and then taking the squar root gives us the integrand

II Ttl x Tv ll = V2 2 + (- u - V)2 + (u - v)2 = V4 + 21.12 + 2v 2 = V2Vu 2 + v 2 + 2.


Since we are integrating over the unit disk, use polar coordina tes. Let 1.1 = r cos B and v =
r sin () , 0 ~ r ~ I , 0 ~ B ~ 27r. Then

JLh Vu 2 + v2 + 2dA = 111211" h Vr2 + 2rdBdr = 2V27r 11v,z + 2r dl'

2V27r' i (r + 2)3/ 2
2
1: = 2V;7r (33 / 2 - 2 3/2 ) = i (6V6 - 8).

8. We choose the parametrization x = ~, y = t, Z = z, 0 :S Z ~ 1, - 1 ~ t :S 1. To compute


the sur face area integrand, we first compute

1 8(x,y~ 1= 0, 8 (y,Z) I- l 18(x,Z )I - t


8(t, z } 1
8(t, z) - , 8(t, z) - Jt2+T'
then,

II T t x Tz II = J t l dt dz
02 + 12 + --Z-
2 = [ffi
t2 +
- 2- -
1
dt dz.
t + t + 1
Thus , the surface area is

1111 [ffit
a
2+
-1
- 2-
t +1
-
1 dt dz -_ 11 [ffi
t2+
-1
- 2-
t
-
+1
1 dt.

We do not a ttem pt to simplify further. An alternative parametrization is to use the hyperbolic


functions sinh t and cosh t; the integral only gets nastier.
126 (H APTER 7

10. Let x = ucos v, Y = f(u ), z = usin v, a :::; u:::; b, 0:::; v:::; 2r. . The reader should verify that
8 8 (X,Z) 1= u.
(X,y) 1= - U!, (U) sin v, 8(y, z) 1= u!,(u ) cos v, and 8(u, v)
18(u, v) I8(u,

v) 1
Thus, the surface area is

A(S) = JIn .ju2 + u 2(f'(u)) 2 du dv.

Since the integrand does not depend on v, the v integral can be performed, and we get the
desired formula:
A(S) = 211" lb lul.jl + (f'(u))2 duo

We are rotating a. curve about the y axis, so consider the distance from the y-axis to the curve
as t he "height," which is IxI. Thus, a cross-sectional circumference of the surface at a fixed
Yo is 211" 1xl· Next, describe the cur ve y = f (x ), a < x < b as a path c (t) = (t, f (t )) . T hen an
infin itesimal arc length can be expressed as J1 + (f'(t ))2 dt or simply ds . T he surface area
is obtained by integrating the cross-sectional circumferences along the path c and the above
Ie
formula reduce to A (S ) = 2r.lxl ds.
13. We are interested in the area of the surface z (x, y) = f(x, y) = 1 - x - y, inside x 2 + 2y2 :::; l.
First, compute
VI + r; + fJ dx dy = v'3 dx dy .
To compute the surface area, we need to parametrize the disc z = 0, x 2 + 2y2 :::; 1 using polar
coordinates: x = rcosB , y = (r / V2) sinB, 0 :::; r:::; 1, 0 :::; e :::; h, and the Jacobian is r/V2.
Our integral then becomes

~r. v'3 dr dB =
2" fl
1 o Jo r.V6
2 .

17. Completing squares, the equation x 2 + y2 = x becomes

(x 2 - x+ t )+ y2 = t,
i.e. , (x - t )2+ y2 = ( ~ t Th is

equation represents a cylinder whose base circle is centered

at (~, 0) with radius ~, as shown . To find the surface area

of 5 ] , we need to consider where the cylinder "sticks out"

".. ,­ ~, ~
of the sphere. Consider the positive octant. The surface
x
area is IID ) 1
+ r; + f3dx dy , where D is half of the base

circle (shaded), and z = f (x, y) = J1 - x 2 - y2 is the


sphere. Since we will be integrating over a circular region, we

can use polar coordinates: x 2 + y2 = x is the same as r2 = r cos B or r = cos B. From the
figure , one can see that D is described by °:: ;
r :::; cos Band 0 :::; 8 :::; r. /2. Also , we com­
2
pute f x = - x /J 1 - x - y2 and by symmetry, fy = -y/Jl- x 2 - y2. So + + f~ = )1 n
1/ J1 - x 2 - y2 , which becomes 1 /~ in polar coordi nates. Remembering that the J aco­
bian is r and that SJ consists of four equal surfaces, we get

A(S t} 4
1f/21cos e r
v'1=T2 drdB = 4 1 0 7r/ 2 ( - ~
Icos e) dB
1o 0 1 - r2 r=O
f7r/2 7r/2
4Jo (l-sinB)dB = 4 (8+ cosB)lo = 21T' - 4.

By high school geometry, we know that A (S:d = 411" - (211" - 4) = 211" + 4, so A(52 )/ A(SI) =
(r. + 2) /(r. - 2).
INTEGRALS OVER PATHS AND SUR FACES 127

20. First, we are going to figure the volume of the material removed to make the hole in the sphere.
Use cylindrical coordinates to describe the hole:

0::; r ::; 1, 0 ::; B ::; 21!' - ~ ::; z ::; ~.


The limits for z were fou nd from the equ ations of the upper and lower hemispheres: z =
J4 - x 2 - y2 and z = - /4 -
x 2 - y2 and substituting 1'2 = x 2 + y2. Remembering that the
Jacobian is r, we get

Vhole = ('" tl~ l ' dz drdB = t "t 2r/4 - r 2 dr dB


Jo Jo -~ Jo Jo
1 2
" ( -2{4 ~ 1'2)3/2 [=J dB = ~ (8 - 3\1'3) 1 2
" dB = 4; (8 - 3\1'3).

Since the volume of the sphere is 3211" / 3, th volume of the coupler is 3211" /3 - (3211"/3 -4rrvS) =
4rrV3.
For the surface area, it suffices to calcula e the surface area of one "cap" of the hole. In rect­
angular coordinates, the surface area of I (x, y ) = J 4 - x 2 ­ y2 over D, the circle of radi us 1,
VI
is ffD + Ii + I~ dx dy. We calc ulate Ix = - x/ J4 - x 2 - y2 and Iy = -y/ / 4 - x 2 - y2,
so VI+ Ii + n := 4/J4 - x 2 - y2 , Changing to polar coordinates, the surface rea of one
cap is

12"11 h 1 4~llr=O ] 1
2 2
drdB = " [ - dB = (8 - 4V3) ". dB:= rr(16 - 8V3).
o 0 4- r w
0 0

Si.nce the surface area of the sph re is 411"1'2, or 16rr , and the surface area of the two caps is
2rr(16 - 8V3), the outer surfa.ce area of t he coupler is 161!'(V3 -1).
22 . (b) We computel:r: = y + l/( y + 1) and Iy =x - x/(y + 1)2. Thus

VI + Ii + fJ dA =
1 + (y2 + y + 1)2 + (x(y + 1)2 - x )2 dA
(y + l )2 (y+l)4
1
(
y +l
)2 J( y + 1)4 + (0 + 2y2 + 2y + 1)2 + (x (y + 1)2 - xF dA ,
and the surface area is

1412
1 1
(
Y + I,
1
\2 / (y + 1)4 + (0 + 2y2 + 2y + 1)2 + (x (y + 1)2 - x)2 dx dy.

7.5: INTEGRALS OF SCALAR FUNCTIONS OVER SURFACES


GOALS
1. Be able to compute the integral of a given scalar function over a given surface.
2. Understand why the integral is defined the way it is and to interpret it physically.

STUDY HINTS
l. Notation. In t his section, we introduce dS, which stands for li T" x T v II du dv , which was
discussed in section 7.4.
2. Importance. This section and the next will be used extensively in chapter 8. In this section ,
we integrate scalar function as we did in section 7.1, and in the next section, we will integrate
vector-valued functions .
128 CHAPTER 7

3. Computation. The form ula for the scalar surface integral of a scalar function f is

fisfdS = flf liTu XTtl ll dudV.


Here, fis usually given as a func tion of (z, y, z ) and we rewrite it in terms of t he parameters
tI and v by substituting x, y, z as functions of tI, v.

4. Physical interpretation .

(a ) If f = 1, then we get the surface area. T his may help you to remember t he formula.
(b) If fis the mass density per unit area at each point of t he surface, we get the m ass of the
surface.
5. Scalar integral over a graph. If z = g(x, y), then the formula becomes

fl f (z , y,g (x'Y ))V 1 + (:!r + (~:r dx dy.


You should remember this or be able to derive it from lIDf li T.. x Tvll du dv.
O. Integra ting over a plane. If S is a plane, we can simplify the integrati on formula in equ ation
(5) of the text :

fis f dS fLc~ = () dx dy ,
=
where cos () D . k and D is the unit vector normal to the plane. (Review the geometry of the
dot product, section 1.2.) The region D is the projection of S onto the xy plane .

SOLUTIONS TO SELECTED EXERCISES


3 . Since we're integating over a hemisphere, it is wise to use spherical coordinates. For th
hemispherical surface, we have p = a, so x = a cos () sin </>, y = a sin () sin 4> and z = a cos </> fo r
o ~ () ~ 271" and 0 ~ </> ~ 71"/2 . Thus
liTex T¢> Il II (- a sin () sin </>i + a cos () sin </>j - Ok)
x(a cos ()cos 4>i + asin () cos </>j - asio </>k)11
a 2 sin </> .
Then

'{J {2'1f r l2

f1 zdS
J D a cos </> II T e x T 4> 11 d4> d8 = Jo Jo 2
a cos </> . sin ¢ d4> dB
a

3
271"a Jo r /2 sin 4> cos ¢ d¢ = 21ra 3 ( . ry
sm;
¢)1"/2 = 7I"a
0
3

5. (a) The equation of the sphere is x 2 + y 2 + z2 = 2Rz, R > O.

Upon com pleting the squares, we see t hat it is equivalent to z

the equati on z 2 + y2 + (z - R)2 = R2 , wbich is a spbere of

radius R centered at (0, 0, R ). The tip of the cone z2 = x 2 + y2

intersects (by design) the sphere at the origin. To fin d the other

intersections, note that x 2 + y2 + (z - R )2 R2 and x 2+ y2= z2


=
implies R2 - (z - R)2 = z2 or z = R. Parametrize t he cone as

follows: x =
p cos B, y = p sin 8, z =
p with 0 ~ p ~ Rand

o ~ B ~ 271" . You should verify that IITp x Tell V2p. Thus ,


= y

the area of the portion of the cone that is inside the sphere is x

12"lR
o 0
V2p dp dB = 271"V2 -p2 1R = 7I" V2R2.
2 0
INTEGRALS OVER PATHS AND SURFACES 129

(b) By "area of the portion of the sphere inside the cone," the authors presumably mean the
area of the piece of the sphere that is the "ice cream" part of this configuration. T his is si mply
the area of the hemisphere of radius R, or 21l'R2 .
7. Parametrize S usin polar coordinates: x = r cosB , y = 1'sinB, 0 ~ r ~ 1, 0 ~ B ~ 21l'. Since
the su rface is described by z = x 2 + y2 , we substitute in x and y, and get z = 1'2. Then,

T r x T 8 = (cos B,sin B, 21') x (-1' sin B,rcosB,O) = (- 2r 2 cosB, -2r 2 sinB, 1'),

and so

dS = IIT r x T elld1'dB= V41' 4 +r 2 d1'dB=1'V4r 2 +1d1'dB .

Calculat,jng IIs ;; dS:

This integral can be done using integration by parts (or the tables) : let
1
u = r2, dv =rV4r 2 +1dr; du=2 1' d1', v= 12 (41'2+ 1)3/2 .

Then t he integral becomes

8. First, integrate over the portion of the cube in the plane z = 1,


which we will call 51 . We have -1 ~ x ~ 1 and -1 ~ y ~ 1, so
T x x T y = i x j = k and II T x x T y II = l. Then

J1 z211Tx x Tyll dx d y

11Jl
-I -1
1·1· d x dy = 4. y

The integral over t he portion of the cube in the plane z = -1, which we wIll call 52 , is done
in the same way, so

J.{
z2 d52 = 4.
is,
ow, for S3, which is in the plane x = l. Let D be the square -1 ~ y ~ 1 and -1 ~ z ~ 1,
and no e that Ty x Tz = j x k = j and IITy x Tzil = l. Then

Similarly,

J.{J~ z2 dS4 = J'J~{ z2 d55 = J'hsf


3
z2 dS6 = ±,
where S4 is the par t of the cube that is in the plane x = -1, S5 is in the plane y = 1 and 56
is in the plane y = -1. Therefore, IIs
z2 d5 is the sum of the integrals over the six surfaces,
which is 4 + 4 + ~ + ~ + ~ + ~ = ~o.
130 CHAPTER 7

11. (a) The equation of the spbere is x 2 + y2 + z2 = R2. One can substit ute x for
y, y for z, and
z fo r x, or any other perm utation , and still get the same equation. T hus, t he three integrals
ought to be equal. (This is what "by symmetry" usuaUy means.) Geometrically, a sphere
"looks the same" no matter how you look at it.
(b) Using part (a) , we have

Jis (x
2
+ y2 + z 2) dS = Jis 2
x dS + Jis y2 dS + Jis z2 dS =3 Jis 2
x dS.

Substitute x 2 + y2 + z2 = R2 and recall that ffs dS is the surface area of 5 to get

11 s
x 2 dS =-1
3
fl (x 2 + y2 + z2) dS = -1
s 3
fl s
R 2 dS =R2
3
411" 4.
- ·411"R 2 = -R
3
(c) Due to symmetry of the sphere, if we integrate x 2 + y2 over the entire sphere, we should
get twice the mass desired in exercise 10. So the desired mass is

~ Jis (x + y2) dS = ~2
2
Jis x dS
2
= ~11" R4.
14. By exercise 12, the average z coordinate is

A tS) Jis z dS.

In this case, A (S), the surface area of a hemisphere ofradius r, is 2 11"1' 2. By exercise 3, ffs z dS
is 11"1'3 , so Z = 1I" r 3/ 211"1'2 = r /2 . Since there is as much of the sphere on one side of the z axis
as there is on the other (by symmetry ), x = y = O.

20. If z = g( x,y), then we can use t he fo rmula stated in equation (4), namely,
Jis !(x, y, z) dS = JIn f( x, y, g(x, y) )J1 + (8g/ ox )2 + (og/ oy)2 dx dy.

Since 5 is defined implicitly by F (x , y, z) = 0, we can use implicit differentiation to get


8z 8g -(oF/o x ) oz og -(oF/oy)
8x ox of/ oz and oy oy of/oz .

Given that ! (x, y, z) = loF/ 8z l, we get

f·r10F IdS
Js oZ
= JrJD 10F8z Iy r::
1 i
[- (OF/8 x)]2
8F/ 8z
+ [-(8 F/8 y )]2 dxdy
8F/8 z
"f 18F I J (8F/8z )2 + (8F/ oX)2 + (8F/8yF dx dy
f JD OZ J(8F/8z)2
JIn J(8 F/ 8zP + (oF/8 x )2 + (8F/8y)2 dx dy.

7.6: SURFACE INTEGRALS OF VECTOR FIELDS


GOALS
1. Be able to compute a surface integral of a vector fu nction.

2. Understand its derivation and physical in terpretation.


INTEGRALS OVE R PATHS AND SURFACES 131

STUDY HINTS
1. Notation. The symbol n( ~ (u o, vo)) is used to describe the un it vector which is normal to (a
param trized surface) ~ at (uo, va ); note that ~(uo, va ) is the base poin t of n.
2. Orientation. As with line integrals, the orientation is important. T he sign of the integral
changes with the opposite orientation.
3. ParametrizatlOn. As with line integrals , we can reparametrize a surface. As long as the
orientation is retained, the value of the integral is unchanged .
4. Definition . The surface integral of a vec or field F over a parametrized surface ~ with domain
D is
ff~F . dS= flF.(Tu X TtJ ) du dv .
The integral of a vector field is a scalar.
5. Generalizations. _'otice that scalar integrals (sections 7.1 and 7.5) do not depend on orienta­
tion. However the sign will change in the integration of a vector fie ld (sections 7.2 and 7.6)
if the orientation has been reversed . Finally, note that scalar integrals involve the length of a
vector and the integral of vector fields involve the dot product.
6. Reduction to calor integrals. If we know he unit vector to t he surface ~ , t hen the surface
integral reduces to IIs(F .
n) dS. Letting f = F . n, we get the scalar integral of section 7. 5.
If he orientation switches, n changes sign and so does this f.
7. Surface integral ouer a graph. If z = g(x, y) and F = (FI ' F2, F3 ), then a normal vector 1S
(- ag/ox , - og/oy,l) and the surface integral becomes

fin [Fl (- ~!) + F2 ( - ~:) + F3 ] dx dy.


8. Physical interpretation. If F represents the velocity field of a flu id , the surface integral of F
over ~ gives the rate of flow of the fluid across the surface ~. T his is known as fl ux .
9. Un it normal to unit sphere. It is useful to know that for the un1t sphere, n = r = (x , y, z).
10. Good example. Example 6 shows three ways of computi ng the same integral.
11. Summary. The formulas following example 6 give a nice summary of sections 7.5 and 7. 6. An
understanding of how to use these formulas is important. If any of t he formulas do not make
sense to you . it is time to review .

SOLUT I O NS TO SELECTED EXERCISES


1. As in example 4, the heat flux across the surface S is IIs
- 'i1T · dS . Parametrize the surface
x + z2 = 2. Let x = V2 cos 8, z = v'2 sin 8 with 0 ~ 0 ~ 211". The surface can be parametri zed
2

by S (8,y) = (V2cosO,y,V2sin8) , 0 ~ 8 ~ 211" and 0 ~ y ~ 2. Then the outward pointing


normal to S is To x Ty = V2cosBi + v'2sinBk. Given T (x,y,z ) = 3x 2 + 3z 2 , we compute
- 'ilT = - 6(.1:,0, z) = - 6( v'2cos 8,0, v'2sin B). Thus

f l - 'ilT . dS = JL - 'ilT· (T o x Ty ) dydB

-6 2""12 [ J2 cos 8) 2+ ( v'2 sin B) 2] dy dB


10 0 (

-1212""12 dydO = -4811".


If the Ty x To cross product (hence the opposite orientation ) is used, the answer would be
+ 4811" .
132 (HAPTER 7

6. First, we compute
j k
\7xF = %x %y 0/8z = - 2zj + (3y - l) k .
x 2 + y - 4 3xy 2xz + z2
Use spherical coordinates to parametrize S: x = 4 cos Bsin </J, y = 4 si n Bsin </J, z = 4 cos </J with
o ~ </J ~ 7r/ 2 and 0 ~ 8 ~ 27r. Then To x T ¢ = 16(- sin2 </J cos Bi - sin 2 </J sin OJ - sin </J cos </Jk).
Thus

J1 (\7 x F ) . dS J1 (0 , - 2z , 3y - 1) . (T e x T¢ ) de d</J

J
- 16 1 [(0, - 8 cos </J, 12sin Bsin</J - 1)

. (sin 2 </J cos B, sin 2 </J sin B, sin </J cos 8)] dB d</J

ior ior/
2 2
1'(
2
- 16 (4sinBsin </Jcos</J -sin </Jcos</J) d¢ d8

-1 6
r [(4
2
1'( 1 )1 1'(/2 ] dB
3sin3</Jsin8 - 2"sin2</J <1>=0
io
- 16 ior (43 sinB - 2"
1) d8
21r

-4 8 - 2"8J1 1'( = 167r.


-16 [Tcos
2

8. (a) The wall lies under the circle z = 4R 2, x 2 + (y - R) 2 = R2 and above the mountain
x 2 + y2 + z = 4R2 . From the top view, we may parametrize the circle by

x = Rcos e, y- R = Rsin8, 0~ 8~27r.

Then the mountain becomes


z = 4R2 - (x 2 + y2) = 4R2 - [( Rcos 0)2 + (R + Rsin 0) 2]
4R2 - [2R2 + 2R2 sin BJ = 2R2 - 2R z sin O.
To find the surface area of the "cylindrical" wall of the restaurant, we parametrize the wall by
x = RcosO, y = R + Rsin8 , z = z
o ~ B ~ 27r,
2R2 - 2R2 sin 8 ~ z ~ 4R2,

and the surface area becomes

271'14Rl
1o 2R~-2Rl sin e

li Te x T211dzd8.
The reader should verify that liTex Tz il = R. Thus, the integral becomes

2"14Rl
= 12"- R (4R2 - 2R2 + 2R2 sin 8) dO = 47rR3 .
1o 2Rl - 2R'l s in e
Rdz dB
0

(b) Parametrize the restaurant interior by


x = r cos 0, y = l' + 1" sin B! z = z,

where 0 ~ 1" ~ R, 0 ~ 8 ~ 27r, 4R2 - (21"2 + 21"2 sin B) ~ z ~ 4R2 . The Jacobian is

os B - rsin B 0
8(x, y, z)
1 + sin B r cos 0 o =r + rsin 8.
8(1", 8,z )
o o 1
INTEG RALS OVER PATHS AND SURFACES 133

Thus, the volume of the restaurant is


2

f2" fR
Jo Jo 1 4R
4R2 -(2r2 +2r~ sin 0)
2
(1' + l' sin B) dz d1' dB

1 " 1 R (1' + l' sin (;1)( 4R2 - 4R2 + 2r2 + 2r2 sin B) d" d(;l

1 Jo 211" fR
(2r 3 + 4r2 sin 8 + 27,3 in 2 8) d1' d(;l

211" (R4
a 2 + R4 sin (;I + 2R4 2)
sin 8 d8.
1

Since f02" sinBdB = 0 and fo sin 2 8d8 = rr , the volu me is rr~ . Thus, as long as R > 0, the
27f

volume rrR 4 is always greater than rrR4 j 2.


(c) The heat flux is JIs
V . dS, where V = -k(6x, 2y - 2R, 32z ). The roof of the restaurant
can be parametrize by x = l' cos f) J y = " sin (;I + R, Z = 4R2 for 0 ~ l' ~ Rand 0 ~ (;I ~ 2rr .
We have T r = (co 8)i + (sin 8)j and T o = (- 1' sin (;I) i + (r cos 8)j , so T r x To = 1'k. T herefore,
he heat flux through the roof is

-k Jis (6:z:, 2y - 2R, 32z )· (0 , 0, 1' ) drdB =- k1 211"lR 32zr d1'd8

-k 1
2
a
"l a
R
128R 2r drdB =- k f
Jo
Z
1I" (64R2r 21R ) dB
r=O
= -k(64R4 )(2rr )
- 128rr R 4k.
The side which makes contact with the mountain can be parametrized by x = l' cos B, y =
rsinB + R, Z = 4R2 - (r2 + 2RrsinB + R2) for 0 ~ r ~ R and 0 ~ 8 ~ 2rr. We have
Tr = {cosB)i+ (sin 8)j + (- 21' - 2Rsin B)k and T o = (-1'sin (;I) i + (reos 8)j + (- 2Rr cos 8)k, so
T r x T 8 = (21'2 co O)i + (2Rr + 21'2si n 8)j + rk. T herefore, the heat flux through the mo untain
side '

- k J is (6x, 2y - 2R, 32z ) . (2r2 cosB , 2Rr + 2r2 sin 8, r ) d1' dB

- k Jo a (12 r 3 cos 2 8 + 4Rr zsin e + 41'3 sin 2 e + 128R2 "


{R1 27T
- 32,,3

- 64R 2 7'Sin e- 32R2 1' ) dB d1'

-k 1R(12rr,,3 + 4rr1'3 + 256rr R 2r - 64rrr 3 - 64rr R 2 ,, ) dr

-k 1 R
(- 48rrr3 + 192rrR 2r ) dr = - 841rR4k .
Finally, th curved gJass wall can be parametrized as in part (a) . We have T o x T z
(Rcos B)i + (Rsin B)j. The heat flux through the wall is

- k Jis (6x,2 Y - 2R,32z )· (RcosB ,Rsin8,0) dzd8

- k 1Z1I'14R2 (6 RZ cos 2 8 + 2R2 sin 2 8) dz d8


o 2R2 - 2R' sin8
-k 1211' (2R2 + 4R Zcos 2 B)(2R2 + 2R z sin e) d8
211"
-k (4R4 + 8R4 eos 2 B + 4R4 sin 8 + 8R 4 cos 2 Bsin B) d8
0
1
- k(8rrR4 + 8rrR 4) = -16rr R 4k.
134 CH APTER 7

Adding these results, we find that the total flux is -2287TR4 k.

11. The surface S is the unit sphere, so S can be parametrized by x cos 0 sin ¢, y = sin 0 sin ¢ =
=
and z cos ¢ for 0 ~ 0 ~ 27T and 0 ~ ¢ ~ 11". Differentiate each component with respect to 0
to get T o = (- sinOsin¢,cosBsin¢, O) . Similarly, T~ = (cosO cos¢, sin Ocos ¢, - sin ¢) . The
normal vector for S is T o x T ¢ = (sin 2 ¢ cos 0, sin 2 ¢ sin 0, sin ¢ cos ¢). Fact or out sin ¢ to get
(sin ¢)(x , y , z) = n = (sin ¢)r . Suppose F = (Fr , Fe, F¢), then F . n = F . (sin ¢r) = Fr sin ¢.
Here, we make use of t he fact that (er , ell, e¢ ) form an orthonormal basis (see section 1.4).
Hence
J1 F . dS = 1 21r L'" Fr sin ¢ d¢ dB.
The corresponding formula for real-valued functions is

Jl!dS = 1 21rl" !sin ¢d¢ dB .

14. A parametrization of the surface ~ ( u , v) = (u, v, 0) , so liT" x T vll = Ii i x j ll = Il k ll = 1. The


scalar integral is
J1 ! (x , y , z ) dS = JIv ! (tI , v, 0) li T " x T vll dudv.

By substituting (x, y) for (u , v), we get ffD! (x , y, 0) dx dy. For a vector field F = (F"" Fy, Fz )
(NOTE: the subscripts here do not denote partials!),

Jis F . dS = JIv F . kdz dy = JIv F. dx dy.

That is, only the z component of F enters into the formula.

16. (a) Parametrize the cone as fo llows: x = reos e, y = rsin O, z = (x 2 + y2)1/2 = r, 0 ~ r ~ I ,


o ~ B ~ 211'. T hen T o x Tr = (- rsin O, r cos B, O) x (cos O, sinB,l ) = (rcos O, r sinB,-r) . The
flux is the surface integral of F over S, which is
21r
r Jro (0, 0, - 1)· (r cosO , rsinB,-r ) drdB = Jor Jot
Jo
21r 1
rdr dO=71'.

For the hot shots, note that the same amount of rain falling through the cone must also go
through the disk x 2 + y2 ~ 1, z = O. The problem then could be done in your head .
(b) Using the same parametrization as in part (a), the total flux through the cone is

Jo
21r
r Jor 1
(-Vi -Vi)
- 2- ,0' - 2- · (rcosB,rsinO , - r ) dr dB = r r .J2 (- r
2tr

Jo Jo 2
1
cos B + r) dr dB

Vi Jo{21r ~ (1
2 2
_ cos O) dO = ~ V2.

7.7: A PPLIC ATIONS TO DIFFERENTIAL G EOMETRY, P H YSICS AND FO RM S OF


LIFE
GOALS

1. Be able to compute the Gauss curvature for a given surface .

2. Be able to compute the mean curvature for a given surface.

3. Be able to explain when the Gauss-Bonnet theorem applies for calculating the total curvature
of a surface.
INTEG RALS OVER PAT HS AND SURFAC ES 135

STUDY HINTS
1. Old calc ulations . In the process of computing the Gauss and mean curvatures, you already
learned how to do the calculations for many of the ingredients earlier in the course. In fact ,
you should consider the calculations a review.

2. New funct ions. If your instructor expects you to memorize how to com pute E, F , G, i, m, n,
it m ay be helpfu l to know that E, F and G involve manip ulations of the first partials of C).
Also, note that N is dotted with the second partials of C) to compute i, m and n.

3 . Calculating Gauss cur vature. Note that E , F and G are not needed to compute the G auss
curvature since W =
IIC)u X If1 v112.
4. Curvature of special surfa ces. You should know the res ults of examples 1 and 2. Example 1
shows that a plane has no Gauss or m ean curvature. Example 2 shows t hat the curvature is
the same at any point on a sphere.

5. Gauss-Bonnet theo r m. For a "sphere-like" surface which is closed and bounded, we have
(1/271')lIs =
K dA 2. Furthermore, a surface of genus g results in he integral (1/271') f{ dA == Ifs
2 - 2g .

SO LU TIONS T O SELECTED EXERCIS ES


1. We begin by computing all fi rst and second partial derivatives of C) (u, v) = (u cos v, u sin v, bv),
that is,

C)u = Tu = (cosv,sinv,O); C) U = Tu = (-usinv,ucosv,b);

C)uu = (0,0 , 0) ; lf1 "v = (-sinv,cosv,O); and C) uu = (-ucosv , -usinv,O).

Using these results, we can compute

T u x Tv = (bsinv,-bcosv,u); E = IIC)"W = cos 2 v +sin 2 v +


F = C)U . C)v = -u sin v cos v + u cos v sin v + = 0; ° °= 1;

G = II C)v 112 = u 2 sin 2 v + u 2 cos 2 V + b2 = u 2 + b2; and


W = BG - F = (l)(u 2 + b2 ) - 02 = u 2 + b2 •

T hen, we calculate N = (T " x Tv) /v'W = (b si n v, -b cos v, '1.I.)/Ju 2 + b2 . Now, we calculate


i, m and n:

f N . C)"U = N ·0 = 0;
m N ' C)uv =(-bsin2v-bcos2v)/)u2+b2=-b/)u'J+b2; and
n N . C)v v = (-ubsin v cos v + ub sin v cos v)/Vw = O.
Finally, t he Gauss curvature is

K = in - m
2
(0)(0) - m 2 -[b 2/(U 2 + b2)]
W W u2 + b2
and the mean curvatu re is

H =: Cf + En - 2Fm
2W

= °

since i = n = F = 0.

5. Let x = u and y = v, so z = u 2 /a 2 - 1I2/b 2 . T hus , a parametrization of the hyperbolic


paraboloid is lf1( u, v) = (u,v,u 2 /a 2 - v 2 /b 2 ). We now compute the first and second partial
derivatives of 1f1:

C)u = T " = (1,0,2 2u/a 2 ); C)V = Tv = (0, 1, -2v/b 2 );


lf1u" = (0 , 0, 2/a ); C)"V = (0,0,0); and c)"v = (0,0, -2/b 2 ).
.. .

136 CHAPTER 7

Using t hese results, we can compute

Ttl X Tv = (- 2u/ a2 , 2v/ b2 ,1) and W = liT" X Tvl1 2 = 1 +4u 2/a 4 +4v2/b 4 .
Now, we calculate e, m and n:

e = N · ()uu = [(Tu x T )/VW] · (0 ,0 , 2/a 2 ) = 2/a 2 /W.


lI

m = N · ()uv = N . (0 , 0,0) = 0; and


n = N · 4'1111 = [(T " x T,, )/Vw] . (0 , 0, _ 2/b 2 ) ;::: -2/b 2 J'{V.
Finally, t he Gauss curvatUIe is
2
){ = en -W m = [( a2 J'{V 2 ) ( 2-2)
JW - °2]
b
.W
'7"

-4 - 4a6 b6
a 2 b2 W 2 - (a 4 b4 + 4b4 u 2 + 4a4 v 2 )2 .
Notice th at we did not have to compu te E, F and G t o find the G auss curvature.

8. T he torus T is defined by

~ ( ¢, () ) = (x ,y,z ) = (( R + cos ¢ ) cos B, (R +cos¢ )sin(),sin cP)'


so

~ ¢ = T ¢ = (- sin ¢ cos f), - sin rf; sin () , cos ¢ ) ;

4' e = T e = (- (R + cos cP ) sin If, (R + cos .p) cos () , 0).

T hroughout this exercise, we use the identity C05


2
A + sin 2 A = 1, so we get the cross product

T tl> x T o - (R + cos ¢ ) cos ¢ cos ei - (R + cos ¢ ) cos ¢ sin Ifj


-( R + cos ¢) sin ¢k.
Nex t , we calculate

W = li T</> X Tol12
(R + COS ¢ ) 2 cos 2 1; cos 2 e + (R + cos cP )2C05 2 ¢sin 2 e + (R + cos ¢)2 sin 2 ¢
(R + cos ¢)2 cos 2 ¢ + (R + cos ¢ )2 sin 2 if;
(R + cosif; f .
Then, we calculate

N = (T ¢ x T 8)/ J'{V = - cos 1> cos (lj - cos ¢ sin €Ij - sin .pk.

Now, we com p ute e, m and n:

£ = N · ()¢¢ N . (- cos ¢ cos If, - cos ¢ sin If, - sin rP )


cos 2 ¢ cos 2 B + cos 2 ¢ sin 2 () + sin 2 .p
cos 2 if; + sin 2 ¢ = 1;
m = N · ~¢ o N . (sin ¢ sin () , - sin ¢ cos 0, 0)
- sin ¢ cos ¢ sin () cos (I + si n if; cos 1> sin () cos + 0 e = 0;
n = N ·4' OB N . ( - (R + cos ¢ ) cos () , - (R + cos ¢ ) sin (), 0)
(R + cos if; ) cos if; cos 2 B + (R + cos if; ) cos ¢ sin 2 If + 0
(R + cos ¢) cos ¢ .
F inally, the Gauss curvatUIe is

Cn - m 2 (1)( R + cos ¢ ) cos ¢ - 0 2


cos if;
K = HI (R + cos¢ )2 R + cos¢
INTEGRALS OVER PATHS AND SURFACES 137

For the torus, which has genus 1, the Gauss-Bonnet theorem tells us that we should get
(1/ 271") ffs K dA = 0. In this case, we have

= ~ {27r {27r os¢l II Tt,6 x T ell dBd¢l


271" io 10 R + cos¢l
1 127r127r cos ¢l
= -271" 0 0
R
+ cos ¢ (R + cos ¢) de d¢l
= {2r. {27r cos ¢ de d¢ = 271" {2rc cos ¢l d¢l
io 10 271"10
= sin¢ C = o.
9. We begin by computing all of the first and second partial derivatives of (I(u , v):

(I" = Tu = (l,h'(u)cosv,h' (u)sin v);


(I ~ = T~ = (0 , - h(u) sin v, h(u) cos v);
(I"u = (0, h"(u) cos v, hI! ('U ) sin v) j
c)u ~ = (0, - h'(u) sin v, h' (u) cos v); and
(I"" = (0, - h(u) cos v, - h(u) sin v) .

Using these result we can compute

Tu X T il = (h'(u)h(u) ,-h(u)cos v, - h(u) sin v).


Furthermore, we calculate

w = IIT u X T~1I 2 = (h'(u ») 2(h(u» 2 + (h(U ))2 cos 2 V + (h(U» )2sin 2 v


= (h(u»2[(h'(u)2 + 1].
Next, we calculate t. m and n:

e= N · c)uu = [(Tu x TtI) / Vw]· (0, h"(u) cos v, h"(u) sin v)


= (- h(u)h// (u) cos 2 V - h(u)h"(u) sin 2 v)/Vw = [-h(u )hl!(u)]/Vw;
m = N . (lU ti = [(Tu x Tu )/Vwj · (0, - h'(u) sin v, h' (u) cos v)
= (h(u)h' (u)sin vco v - h(u)h'(u) sin v cos v)/Vw = 0;
n = N·(ln = [(TuxTu) / Vw]·(O, - h(u ) cosv,-h(u)sin v)
= [(h(U»2 cos 2 v + (h(U))2 sin 2 v]/Vw = (h(u))2/Vw.
Hence, the Gau curvature is

K
= en - m 2 = en -
W W
° = (-h(h
VW
ll
) ) ( ~) ~
VW ' W
= ­ h3 (h")
W2
= - h3(h" ) _ - hI!
[( 1 + (h' )2)h2J2 - [(1 + (h' )2)2h] '

SOLUTIONS TO SELECTED REVIEW EXERCISES FOR CHAPTER 7

l. (b) First, we calculate c'(t ) = (- sint,cost, 1), so lI e'(tl ll = .;2. Then the path integral is
{{2'1r {27r
1c fds = i o xyz ll e' (t) ll dt = io V2 (cost )(sint)t dt .
138 CHAPTE R 7

Use integration by parts with 1.1 = t, dv = cos t sin t dt, du = dt and v = ~ sin 2 t. T he integral
becomes

V2 t
-sin 2 t 127r _ 127f ~sin2tdt ) = _~
In ( 2 7rl- cos2fdt =-7rV2
( 2 0 0 2 2 io 2 2 .

2. (b) As in exercise 1(b), we have ds = II c' (t) II dt = V2 dt . Then, the path integral is

{27f {27f ( 1 + cos 2t ) dt

1f
c ds i o (sin t + cos t)V2 dt =
2
V2 i o sin t + 2

V2 ( - cost + "2 +
t'
SI: 2t) 1027r = V27r.
3. (a) Compute t he line integral over each segment of C and add them together. Be careful

with the orientation. Recall from chapter 1 that the equation of the line I(t) = (x, y , z) =

(1 , 0,0) + t( - 1,1,0), 0 :5 t :5 1 satisfies the condition 1(0) = (1 , 0,0) and 1(1) = (0, 1,0). On ...
this segment , x = 1 - t, Y = t and z = O. Also dx = - dt, dy = dt and dz = O. Substitute
these values into the given line integral to get

11
o
[sin 7r(1 - t) dt - (cos 7rt)(O) ] = 11 0
sin 7r(1 - t) dt = -1 cos 7r(1 - t)
7r
10
= -1 (1 - (- 1)) = -2 .
7r 7r
T he next segment can be parametrized by I(t) = (x , y, z) = (0,1,0) + t(O, -1 , 1) = (0,1- t, t )
for 0 :5 t :5 1. Also, dx = 0, dy = - dt, and dz = dt , so we get

11 o
1 11

[sin 7r(0)( -dt ) - cos 7r( 1 - t ) dt ] = - sin 7r(1 - t) = O.


7r 0

Finally, the last segment can be parametrized by the equation

I(t ) = (x, y, z) = (0, 0, 1) + t(l, 0, -1) = (t , 0,1 - t)

for 0 :5 t :5 1. Also, dx = dt, dy = 0 and dz = - dt , so we get

11 [sin 7rt (O) - cos 7r(0)( - dt )] = 11 1 dt = l.

T herefore, the line integral around the triangle is 2/7r + 0 + 1 = 2/7r + 1.

5. We start at (0, 0) and compute t he line integral over each side of the square. For t he line

segment from (0,0 ) to (a, 0), we have y = 0, dy = 0 and :5 x :5 a. Thus


°
J F . ds =i o r [( x
2
-
2
0 ) dx + 2x (0) . (0)]
r
=i o 2
x dx = a-
a
3

For the line segment from (a, 0) to (a, a), we have x = a, dx = 0 and 0 :5 y:5 a. Thus,

J F· ds = l a
[( a
2
- y2)(0 ) + 2(a) ydy] = l a
2ay dy = ay2 1: = a3 .

For t he line segment from (a, a) to (0 , a), we have y = a, dy = °and x goes from a to O. Thus,

J F · ds = iar (x 2 - 2
a ) dx =- l 0
a
(x
2
-
2
a ) dx = - (a
3
a- - = 32a '
3
a )
3

Finally, for t he line segment from (0, a) to (0, 0) , x = 0, dx = 0 and y goes from a to O. T hus

J F · ds = O.
By addition, we find t hat the line integral around the square is a3 /3 + a3 + 2a3 / 3 + 0 = 2a 3 .
INTEGRALS OVER PATHS AND SURFACES 139

7. (b) First complete the square: (2x2 - 8x + 8) + y2 + z2 = 1 + 8 or 2(x - 2)2 + y2 + Z2 = 32.


This has the form X 2 + y2 + z2 = p2, so use spherical coordinates: For the x parametrizat ion,
let X = =
v'2(x - 2) 3cos Bsin<,h , so x = 2 + (3 cos 11 sin <,h )/ v'2. Thus, the parametrization is

x 2 + (3 cos hin <,h)/v2


y 3 sin 11 sin <P
z 3 cos <p,

with 0 ~ B ~ 271' and 0 ~ <P ~ 71'. This is a general strategy in at tacking many parametrization
problems: changing cartesian coordinates into either cylindrical or spherical coordinates by
completing the squares.
10. The surface area of the graph lying over D is lID
II Tx x Ty II dx dy and the area of D is lID dx dy.
The' parametrization" of t he graph is x = x, y = y, z = / (x, V) . Thus,

Tr = i + (8// 8x)k and T y = j + (8!1 8y) k.

T herefore,

Tx x Ty = (8/18x)i + (8//8y)j + k and II Tr x Ty ll = [( 8/18x )2 + (8 //8y )2 + l f/2.

Since (8/18z)2 + (8//8y)2 = c, II Tx x Ty ll = JI+C. Returning to t he original formula, we


have lID IITr x Tylldxdy = lIDJI+Cdxdy. Since c is a constant , we factor the constant
from the integral to get VI+C lIDdx dy = v'I+C . (area of D) .
12. (b) Use cylindrical coordinates . Let x = rcosB, y = r sinl1. In addition, Z = x = "cos l1 , and
the intervals are 0 ~ l' ~ 1, 0 ~ 11 ~ 27l' since we want to be inside the cylinder x 2 + y2 = 1.
We calcuJate

(cosBi + sin OJ + cos 11k ) x (- rsinOi + l' cos OJ - l' sin Ok)
(-ri + rk)=r(-i + k ) so IIT r xTell= v2r .

Therefore,

15. We want to compute lIs x


dS , where S is the trian­
gle with vertices (I, I, 1), (2, 1,1) and (2 ,0,3). First ,
we need to find the normal to the triangle: Two vec­
tors on the triangle are (1 , 0,0) and (0 , I, - 2) (found by
subtracting the coordinates of the vertices). Take t heir
cross product and normalize it. We get the unit normal (1,1,1)
n = (0 ,2, l)/v'5, so cosB =
n · k = 1/ v'5. Next, the
projection of S onto the X!t"plane can be described by I y
, (1,1,0)
- y + 2 ~ x ~ 2, 0 ~ y ~ I , as shown. Thus,

Jis xdS = v'sJ'rJD x dx dy = v's1 1


0
1 2

-y+2
X dx dy x (2,1,0)
1
v'2 5 1 4 - (2 - y)2 dy
-
0

v'5 [4+ (2- y)311] = 5V5.


2 3 6
o
140 CHAPTER 7

19. Here , we have F = e'i + tj + t 2k and ds = etj + j + 2tk. Thus

1 F . ds = 10
1
(et . et + t . 1 + t 2 . 2t ) dt = 10
1
(e2t + t + 2t 3 ) dt

( 2
e
2t
t2 t4) 11 = 21 (e
+ "2 +"2 0
2
+ 1) .

24. Our surface is z2 =


1 - x 2 - y2 . We want the part above the xy plane , so we want z ~ 0, or
= =2
f (x, y ) z (1 - x ­ y2 )1 / 2. For the surface area of a graph over D, the formul a is

A= fL V 1 + f ; + f;dxdy .

In this case,

_ - 1 2 2 - 1/ 2 _ -x _ -y ( )
fx - - 2 . 2x (1 - x - y ) - ( 2 2)1 / 2 and fy - ( 2 2 )1/2 by sym m etry .
I - x - y I - x - y
Hence , the integrand is

x2 +
y2
1+ 1_ x 2 _ y2 - (1 _ x 2 _ y2 )1/ 2 '

f afa
The area we want is
1
_ 2 _ 2) 1/ 2 dydx.
Y - a -a ( 1 X

We can eval uate one of the integrals. Let u = (1 - x 2)1/ 2. Since


2 tI is independent of y, the
integral in ybecomes

f a(u
- a
2 _
1
y
2) 1/ 2 dy =~
1 fa
-a
I. "
1
'" dy.

Substituting v = y/u with dy = tI dv, we get


a ti

f /

- a / ti
dV
v'f=tj2 =
1- V
sm
. -1
(v)1
a/ ti

- a/ti
a
= 2sm. -1 (-)
U
= 2sm
. - 1 (
~.
a
1 - ;z;
)

P lug this back into the original integral and get t h e form ul a for t he surface area:

2 I: sin-
1
(b) dx.

27. (c) Use cylindrical coordinates: x 2cosO, y 2sinO , z =


z, where 0 :5 (J :5 21r and = =
o :5 z :5 x + 3 = 2 cos (J + 3. (Note the difference in parametrizations, as compared t o exercise
12). We calcula te IIT8 x Tzil = IIC- 2sinOi + 2 cos OJ) x kll 112cos Bi + 2sinBj il 2 . T hen , = =
the surface integral beco mes
211' 12 cos 8+ 3 f211' 2

1o 0

2z2 dzd() lo
2 f211'

a (2cos O+ 3)3 dB

alo (8 cos3 () + 36 cos 2 () + 54 cos B + 27) dO


2 f21r [
al
.
8( 1 -S10 2
(J)cosO+ 36
(1 + cos2 2B ) + 54cosB+ 27J dO
o
3 2

a2 [ 8 ( smO
. - -Sin3 - (J ) +18 ( ()+ -Sin20)
2- . O+ 270J/0 11'
+ 54sm

2
a [18 . 21r + 27 . 21r] = 601r.
IN TEGRALS OVER PATHS AND SUR FACES 141

TEST FOR CHAPTER 7

1. True or false . If false, explain why.

°~ u ~ 2. Then the line integrals


°
(a) Let c(t ) = (e t + 1, sin 2t , - 2t + 4), ~ t ~ 1 and let b (u) = (eu / 2
Ie
F . ds and Ib
F . ds are equal.
+ 1, sin u, -u + 4),

(b) On a given domain, jf g(x , z) ~ °and f is a positive scalar, t hen the integral of f over the
surface y = g(x, z) is negative.
(c) Over a triangle with vertices at (-1 , 1), (2, 3) and (5 , - 3), the surface area of the graph of
=
I( x, y) 2xy+y+5 is the same as t he surface area of the graph of g(x , y) x 2 + y2 + x +3. =
(d) Consider t he cone z = J x2 + y2. Cl t( u , v) = (u , v,";U2 +v2) yields a one-to-one
parametrization if u ~ 0 and v ~ 0, but tl2 (r , 0) = (r cos (I , r sin (I , Jr)
for 0 ~ (I < 211',
r ~ 0 is not one-to-one.
(e) If the line integral of F along one closed curve is 0, then F is a gradient.

2. An ellipsoid centered at the origin wi th intercepts at x = ± 1, Y = ±2 , z = ± 3 can be


paramet rized by tI (B,</J ) =
(cos (1 sin </J,2sin(1sin</J,3cos</J), °
~ (I < 211', °
~ ¢ ~ 11'. If this
ellipsoid has surface area A , how much larger is the surface area of an ellipsoid centered at the
origin wi th intercepts a.t x = ± 2, Y = ± 4, z = ± 6?
3. Find the work done by a force F = xi + x yj along the path beginning at (0, 0), going to (1,1 ),
to (2,0) and t hen back to (0,0) all along straight line segments.

4. Let F(x, y , z) = xi + yj + (x + z) k. Compute t he surface integral of F over the elliptical cylinder


x 2 /4 + y2 = 1, 0 ~ z ~ l.

5. At a point (x , y, z) on a spherical shell of radius 2, the density is Iz I.


(a) Find a paramet rization for t he sphere using the spherical coordinates (I and ¢.
(b) Write the mass of the sh II as a surface integral.
(c) Compute the mass.
6. (a) Compute the gradient of I (x ,y,z) = eZ + 2x z sin y.
(b) Let p be the path from the origin to t he point. (1r/ 2, 1, 0) along the curve y = sinx . From
there, p continues along a st raight line to (5, e, - 3). Then p cont.inues on a straight
line fro m (5, e, - 3) and ends at (2, 37r/ 2, 1). Compute the line integral of F (x , y,z)
2z sin yi + 2xz cos yj + (eZ + 2;1; sin y)k over p.
7. Let C) (u v) = (u2 , V - 2u , 3uv 2 ) be a parame trized surface.
(a) Find the equat ion of the tangent plane when (u v) = (1, 1) and (x, y , z) = (1, -1, 3) .
(b) Write an expression for the surface area of tI over the region (u, v) E [0, 2] x [-1,1]. (Do
not evaluate the integral. )

8. Find the fl ux of F(x , y , z) = ;ri - 3yj + 2zk across the surface z = x 3y + y2 on [- 1,1] x [0, 2].

9. Let r (t) = (t 2 , et , t + 3, 2t ), 0 ~ t ~ 1 b . a path in 1R4 and let F(w, x , y, z) = (z , 3, w + y, 2x ).


Compute the line integral of F over the path r .
10. A new amusement park is being built in town . Its perimeter has an elliptical shape and can be
described by x 2 /9 + y2 /4 = 1. A roller coaster ride is to be built on top of the fe nce enclosing
the park . Its height is IxYI. Use the path integral to compute the surface area of the park's
fencing.
143

8 THE INTEGRAL THEOREMS OF VECTOR ANALYSIS

8. 1: GREEN'S THEOREM
GOALS
1. Be able to state Green's t heorem .
2. Be able to use Green's theorem to compute a line integral or a double integral.
3. Be able to use Green 's theorem to fi nd an area.

STUDY HINTS
1. Notati on. Recall that a,
t he same symbol used for part ial derivatives, means boundary. Thus,
aD is the boundary of D.
2. Green's the07'em. Under c rtain conditions, a line integral is converted into a double integral:

L (PdX+QdY) = iL (~~ -:) dxdy .

Of en, one ide of the equation will be much easier to evaluat than the other side.
3. Required conditions .
(a) C must be a closed curve wit h an ou tside counterclockwise orientation. Interior "holes"
have a clockwise orientation . In both cases, the region is on your left if you walk around
C with the correct orientation.
(b) P and Q must have continuou first derivatives.
If Green's theorem does not apply di rec ly to a region, the region can usually be subdivided
so th at the theorem can be applied. See figure 8.1.5 of the text .
4. Green's theorem and area. T he are of D m ay be computed by the formu la

A = ihD (xdy-ydX).

Again, aD is oriented counterclockwise. This for ula is most useful if the boundary has a
simple parametrization ; otherwise , double integration is usually si mpler.
5. Vector form. If F = Pi + Qj , then \7 x F = (aQ / ax - ap/ ay)k, so another formulation for
G reen's theorem becomes
hD F . ds = iL (\7 x F ) . kdA,

where ds = dx i + dyj.
6. Divergence theorem in the plane . If n is an outward unit normal to aD , then

hD F · nds = iL div FdA.

Compare this with Gauss' divergence theorem in space studied later in Section 8.4.
144 CHAPTER 8

SOLU T IO N S TO SELECTED EXERCISES


2. According to Green's theorem, the area of a region is

A= ~laD(X dy - ydx) .
Since D is a d isk centered at (0,0) with radius R, the boundary aD can be parametrized by

x = R cos e and y = R sin B, 0 ~ e ~ 27r .


T hen, dx = - R sin Bde, dy = R cos e de , and so the area is
1 f2 Tr 1 f 2Tr
A = 2" lo [(Rc os B) (R cos e) - (Rsin B)( - Rsin e)] dB = 2" l o R2 dB = 7r R2 ,
which is indeed the area that is calculated by using elementary geometry.
3. (b) We use the same parametrization for aD as in exercise 2. T he left-hand side ofthe identity
in Green 's t heorem is

f (P dx + Qdy) f (x + y) dx + f y dy
JaD Ja D JaD
f2Tr f 2Tr
Jo (R cose+ Rsin e)(-RsinB)dB+ Jo (R sinB)(R cose) de
f 2Tr
Jo (- R2 cos Bsin B- R2 sin 2 e + R2 sin e cos B) dB
2Tr 2Tr
_ R2 1 sin 2 BdB = _ R2 1 1 - cos 2B dB = _ R2 7r.
0 0 2
By Green's theorem, the same integral should be equal to

J'f (OQ- OP)


oX oy
JD
dx dy.

Again, we use polar coordinates. D can be described by 0 ~ r ~ Rand 0 ~ B ~ 27r. Calculating


oQ/ Ox = 0 and fJ P/ fJy= 1 and remembering that the J acobian for polar coordinates is r, the
right-hand side of Green 's t heorem becomes

f 2Tr f R
Jo Jo (0-1) rdr dB=
10
2
" fR 2
Jo -rdr dB=-7r R .

Therefore, Green 's theorem is verified for this case,


5. Green 's theorem t ells us that the area is

A= ~ f (x dy - y dx).
2 laD
From the given parametrization of the cycloid , we compute dx = a(1 - cos B) dB and dy =
a sin B dB. Remember th at t he x axis fo rms part of the boundary. This part of the boundary
can be described by x = 27ra(1 - u) and y = 0 fo r 0 ~ u ~ 1. Notice that our cycloid is
traversed clockwise, t hat is, from left to right, starting at the origin along the cycloid, and
t hen back along the 3.; axis from right to left. Since y = dy = 0, the desired area is
1 f 2Tr
"2 J [a(B - sin (1) . asin BdB - a(1 - cos B) . a(1- cos e) dB]
1 t
+ 2" J (x· 0 - 0 . dx)
o o
~ (a2B sin B- a sin B- a + 2a cos B- a cos 2 B) dB
2 2 2 2 2

1 12Tr (a2 f) sin e- 2a 2


- + 2a 2 cos e) dB ,
2 0
THE IN TEGRAL TH EOREMS OF VECTOR ANALYSIS 145

Using integration by parts, we get

To get t he correct answer, recall that t he direction should be counterclockwise. By simply


changing the sign , we get the area spanned by one are of the cyeliod, 3a 2 '1T".

8. Let D be t he union of sim ple regions, Di , i = 1,2, 3, ... ,n, where each boundary, aD;, is
oriented counterclockwise. Suppose P and Q are continuously differentiable on D. We want to
show th at the boundary of D is the same as the boundary of all of the Di. Each intersecting
boundary lies on the bou nd ary oftwo regions. W hen orientations are considered, contributions
to the integral cancel ou t as in figure 8.1.5. Hence

{ (P dx
J& D
+ Q dy) t.=1 J& {D, + = til .(~~ - ~P)
(P dx Qdy )
.= 1 D, Y
dx dy

Jl (~~ -~:) dx dy.

11. (a) Using polar coordinates , we let x = r cos () and y = r sin (). Then D is described by 0 ~ r ~ 1
= =
and 0 ~ () ~ 2'1T". We com pute that div F 1 + 1 2. (If necessary, you should review how to
compu te a divergence in section 4.4.) Since the J acobian for polar coordinates is 1', we get

Jl div FdA = 127<11 (2)1' dr d() = 211" .


On the other hand , we know t hat the outward unit normal to t he unit circle is n::o: (cos (), sin ()),
so
27< (cos () , sin ()) . (cos () , sin ()) ds = 127< ds = 2'1T".
1&D
F . n ds =
1 a a
(b) We want to compu te
{ F · nds ,
J&D

where F = 2xyi - y2j . By the divergence theorem, this integral is equal to

Jl div F dA = Jl (2y - 2y ) dx dy = O.

15. Since cos 2 () + sin 2 () = I , let x/a = cos() and y/ b = sin (), 0 ~ () ~ 211". Then x = acos(),
dx = - asin () dO, y = bsin() and dy = bcos ()d(). By Green's t heorem , t he area of the ellipse is

A = ~ lD (x dy-Y dx)
"2112
a 7< [( a cos ())( b cos ()) - (b sin ())( -a sin ()) 1d()

! (21r ab d() = ab'lT".


2 Jo

16. In polar coordinates, x = l' cos B and y =


rsin (). Since r is a function of (), we get dx =
(1" cos () - l' sin ()) d() and dy = (1" sin () + r cos ()) d() (here 1" denotes the derivative of l' with
146 CHAPTER 8

respect t o e). Substituting into Green 's theorem , we get

A ~ [ (xdy -ydx)

2Jc

~ f b(1' cos e)(r' sin e + 1'COS e) de - (r sin B)(r' cos e - rsine) de


2 Ja
1 fb ') . ') 1 fb
2 J ,.2 (cos~ e + sm~ e) de == 2 Ja 1'2 de .
a
19. To fo rm one loop of the rose, we go from e = 0 to e == 7r /2. Using the result of exercise 16 , the
a rea IS

1
2 Jo
r/ 2
(3 sin 2e) 2 dB ~ r/ (9 sin
2
2 2B)dB = ~ r/ (1+ cOS2 4e )
2
de
2 Jo 2 Jo
~ (B+ Sin4e) 11f/2 = 97r
4 4 0 8

22 . Use the given definition of Bu/an and the divergence theorem in the pl ane for the regIOn
B = Bp:
f oou ds = jf \7u . n ds = J' f \7 . (\71/,) dA = J'{ \7 2 u dA.
J8B n 8B JB JB
26. (a) Suppose u (p ) is a maximum point on D. From exercise 25 , u (p) is the average value of u on
a disk of rad ius R centered at p. T his is possible only if u (p ) = u( q) for all q in D . Indeed, if
u (q) < u(p ) for some q in D , there must be a u(r ) > u(p ) to m aintain the average. Therefore,
tI must be constant on some disk centered at p .

8. 2: STOKES ' TH EOREM


GOALS
1. Be a ble to state and use Stokes ' theorem.
2. Be able to use Stokes ' theorem t o calculate a line integral on a closed curve or a surface integral
over a surface wi th a closed curve as its boundary.

STUDY HINTS
1. R eview. Surface integrals are used in t his section . You should review section 7.6 if you have
forgott en how to calculate a surface integral.

2, R elation to Green's theorem. Like Green 's theorem , Stokes ' t heorem converts an integral in
one dimension to an integral in two dimensions. Stokes ' theorem is a generalization of Green 's
theorem .
3. Stokes ' th eorem for graphs. If z = f(x , y) , then
Jis curl F · dS = lasF . ds.

This is a formula you should memorize. As with Green 's theorem , oS is oriented so that the
surface is on your left as you walk upright around as, which must be a closed curve .
4. Genemlized surfaces. If the surface S is not the graph of a functio n, then Stokes ' theorem is
still true if S can be descri bed using a one-to-one parametriz ation. T he boundary of D, oD,
gets m apped to as, so oD should h ave t he correct orientation. For an example when the
orient ation becomes imp ortant , see the solution of exercise 5 of section 8. 1.
THE IN TEGRAL TH EOREMS OF VECTOR ANALYSIS 147

5 , Application, According to Stokes' theorem, to evaluate ffs curl F ' dS , we can change the
surface 5 to any other surface with the same boundary 85, In most cases, we will change to
a planar surface or another simple surface, Imagine a loop of wire with an elastic sheet S on
it , The surface integral of the curl of a vector field over any surface formed by a deformation
of the elastic sheet will be equ al to t he line integral over the wire (assu ming the wire itself is
not deformed),
6, Circulation, You should know t hat curl V, n is the circulation of V per unit area of surface
perpendicular to n ,

SOLUTIONS TO SELEC TED EXERCISES


1. By Stokes' theorem , we only need to evaluate

r F . ds,
Jas
where 8S is the circle x 2 + y2 = 1, Z = 0, the boundary of OUI surface . Parame trize the circle
using polar coordinates, i.e., x = cos B, y = sinB, Z = 0, 0 ~ B ~ 27r. T hen the integral
becomes
2~ 12~

1
1
&S
F · ds =
0
[sin 8(- sin 8)- cos 8(cos B)] d8= -
0
d8 =- 27r .

4. We want to show that the derivative of he magnetic flu x with respect to time is O. We begin
wit h

~
8t Js
J'{
H . dS = J' {
8H . dS = -
Js at
8H . dS .

Js at
J'r-
Th first step is justified because S is not a function of time. By Faraday's law ,

-Jis ('v x E ) . dS .

Then by Stokes' theorem and the fact t.hat E· ds = 0, since E is perpendicular to t he boundary
of 5, we get
- { E· ds = O.
Jas
5. The boundary of th surface is a closed curve, so we m ay take advan age of Stokes' theorem:

Jis (V' x F ) . dS:;:: la s F . ds .

T he bou ndary is the circle x 2 + y2 = 1, Z = O. The right-hand side becomes

{ F · ds ={ (x, y) . (dx, dy).


Jas Jas
Now use polar coordin ates: Let x = cos 8, dx = - sin 8 d8; y = sin 8, dy = cos 8 d8; 0 ~ 8 ~ 27r .
Substitution into the last integral gives us

r (- cos 8 sin 8 + sin Bcos 8) dB =


Jo
21r
O.

9. Use Stokes' theorem . The boundary of S is the circle y2 + z2 = I , x = O. On the boundary, F


becomes - i'j . Use polar coordinates : y = sinO, Z = cosB , 0 ~ f) ~ 27r. By Stokes ' theorem ,
we get
148 (HAPTER 8

12. The flow rate is JJs curl I) ·dS and by Stokes ' theorem, this is Ja s I) . ds. We have the boundary
on the xy plane where z ~ O. Parametrize the boundary by
x = (R/4) cos O, y = (R/4) sinO .
Then the fl ux is
f2Tr R2 11"R2
1 asl)·ds = io 16(sin B+cos B) dO=-8-·
2 2

14. By Stokes' theorem,


J1s (V' x F) . dS ~ las F . ds.
J
Since F is perpendicular to the tangent to the boundary S, F·ds ~ 0. Hence, Js ('V' x F ) ·dS ~ O.
If F is an electric field, this means the rate of change of magnetic flux is zero by Faraday's
law. See example 5 in the text .
18. The solu tion relies on the basic identities of vector analysis (see table in section 4.4) . Since C
is a closed curve which is the boundary of a surface S, we have C = as in Stokes' theorem.
T hus, Stokes' theorem tells us JJs (V' x F) . dS = Je F . ds .
(a) Here, we use Stokes' theorem where F = IV' g. We need to show that \l x fV' 9 ~ V'I x V' g,
or curl(JV'g ) = \llx\lg . Bybasicidentity10,we havecurl (lV'g) =/ curl(V'g )+\llx V'g, but
by basic identity 11, curl( V' g) = 0 , and 80 we have curl (IV' g) = 1 (0) + V'I x V' 9 ~ V'I x V'g , as
required. Al tern ati vely, we could have computed the curl of IV'g = I (ag/o x )i + I(og/oy)j +
I (og/ oz) k and t hen used the equality of mixed partials since I and 9 are C 2 func tions.
(b) Here, we use Stokes' theorem with F = I V' g+ gV' I. By basic identity 6, curl(fV' g+ gV' f) =
curl(fV' g) + curl (g\l I ). Applying basic identity 10, we have

curl (IV'9 + 9 V'f) f cUrl(V'g) + V' I x \lg + 9 curl (V' f) + V'g x V'I
I curl( V'g) + 9 curl (\lf)
since V'I x V' 9 = -(V' 9 x V' f) by properties of the cross product. According to basic identity
11, curl(\ll) = cu rl (V'g) = 0, and so curl(l\lg + gV'f) = O. By Stokes' theorem ,

[(IV'9 + gV' f) . ds Jis V' x (IV' 9 + 9V'/) . dS

JIs o· dS ~ O.
2l. First , we wili calcuiate JJs (V' xF ) .dS . Weco m pute \lxF ~ (1,1, 1). Alsol)r = (cosO , sin O,O)
and 1) 0 = (- r sin 0, "cos 0, 1), so I)r x I) e = (sin e,- cos 0, r ). Therefore , we get
1
Jis (1,1,1) . {Sin 8, - cOs8, r ) d7>dO = 1 1"/2 (sino -coso+ r ) dBdr = 11 Cir) dr = ~.
On the other hand , the boundary, aS, is composed of four parts. First, when r = 1, we
have 1)(1, B) = (cos e, sin fJ, B), so F = (e, cos B, sin e) and ds = dl) (1, 8) = (- sin e, cos B, 1) dB .
T herefore,
Tr / 2
1 BS,
F· ds =
l0
(0, cos 8,8in B) . (- sin 0, cos B, 1) dO.

Using integration by parts to integrate -e sin B and the half-angle formula to integrate cos 2 8,
we get

( "20+ SI:
. 20) - cosO]1"'0/2 11"
[(Bcos B - sine) +
4
When e= 11"/2, orientation is maintained by letting r go from 1 to O. T hus, we have

f F . ds = fO(~ ,o, r). (O,1, O)dr=O.


iaS2 il
THE INTEGRAL THEOREMS OF VECTOR AN ALYSIS 149

W hen r = 0, B goes from 7r/2 to 0, so we get

[ F · ds = [0 (B , 0,0) . (0 , 0,1 ) dB = o.
las. l-rr /2

Si milarly, when B = 0, we get

[ F · ds = [l(O,,·, O).(l , O, O) dr =O .
las. lo
Adding all the parts together, we get Ias F . ds = 7r/4 + 0 + 0+0 = 7r/4, so theorem 6 is
verified.

25. For a direct computation , parametrize the sUl'face as follows: Le x = T' cos B and y = rsinB,
°
so z = t(x2 + y2 ) = r 2 /2. Also , we want ~ z ~ 2, so 0 ~ ,. 2 / 2 ~ 2, or ~ r ~ 2. In addition,
we have 0 ~ B ~ 210 . W calculate T o = (- 1' sin B,rcosB, O) and Tr = (cosB , in B, r), so the
°
outward normal is T 8 X T r = (1'2cos e, r2 sin B, -1'). Also, we calcu la te

\7 x F =
i j
a/ax a/ay a/az
3y - xz _ yz 2
k
-(-
- z 2 + x , 0 , -z - 3) -- (-1
- r4
4
+ 1'cosB ' 0 -1 )
-1'2 -3 .
, 2

Finally, IIs(\7 x F) . dS becomes


2 r2rr

1lo
o (\7 x F) . (T o x T r ) dBd1'

12 (2 7r1· 3 + 67rr ) dr = ( 7r;4 + 37r7'2) I: = 207r .


On the other hand, by Stokes' theorem, I1s (\7
x F ) . dS = Ias
F . ds. T he boundary is as,
which is the circle ofradi us 2 in the plane z = 2. It can be parametrized by (2 cos t, - 2sin t , 2)
°
for ~ t ~ 27r. We use this orientation b cause the surface lies below the boundary, so we
should traverse it in a clockwise orientation. We com pute ds = (- 2 sin t , - 2 cos t , 0) dt, so

[ F · ds
las
= 1 211'
(- 6 sint, - 4cost,8sint )· (- 2sint, -2 cost ,O) dt

r -rr (12 sin t + 8 cos t ) dt


lo
2
2 2

[12 (~ - Si: 2t ) + 8 (~ + Si:2t )] C = 207r.

If one choo es the other orientation, the answer should be - 201T.

8.3: CONSERVATIVE FIELDS


GOALS

1. Understand that the line integral of a gradient field is path independent.

2. Be able to determine whether a field is conservative.


150 CHAPTER 8

3. Given a conservative vector field , be able to fi nd a scalar function whose gradient is equal to
that vector field .

STUDY HINTS
1. Theorem 7. T his is a very important theorem . To summarize, it states that if F is a gradient,
then \l x F = 0, the line integral depends only on the endpoints, and all lin e integrals around
closed curves are O. Also, if one these con ditions hold, then F is a gradient. If the conditions
of the t heorem hold , then we say that t he line integrals are "independent of path." Note that
if a single line integral is 0, then F may not necessarily be a gradient.
2 . Example 1. In method 2, part (a), notice how after integrating in x, we add a "constant"
h 1 (y, z). It is a "constant" because it only involves the other variables. Study example 1
carefully. You should know how to use at least one of the two methods .
3. Gra di ents in 1R 2. The corollary precedi ng example 3 tells you that if the integrand in Green's
theorem, oQ/ Bx - BP/By , is 0, then F = Pi + Qj
is a gradient . Be careful!! F must be C 1 on all of
]R. 2, unlike theorem 7, which allows some exceptional
points in 1R3. Exercise 12 el aborates this point . If the
x origin is an exceptional point in IR 2, then the integral
of F over t.he path on the left (a closed path) is 0
because the path does not include the origin, while
the integral of F over the path on the right is not.
4. Is F a curl? F is a curl of some vector field if div F = O. Exercise 16 explains the procedure
for fin ding a G such t hat F = curl G.

SOLU TIONS TO SELECTED EXERCISES


2. (a) Si nce y = 2X2, we have dy = 4x dx, and so

[ [1 t 1
lc F· ds = 10 (x · 2X2 ) dx + (2X 2 ) 2. 4xdx = 10 (2x 3 + 16x 6 )dx = 2" + 3' = 6'
8 19

(b) The answer is yes, since

j k
\7 x F =I B/ox a/ ay o/Bz 1= (0 , 0, -x) f- O.
xy y2 0

Alternatively, one can pick a different path and show t hat the line integral is not equal to li.
T his would mean that t he line integral of F is path dependent.

3. If F = \l f , then t he x component of F must be of /ax, i. e.,


of/ox = 2xyz + sin x. (1)

Similarly, the y component of F must be oj/ oy and the z component of F must be oj/ oz,
%.e. ,

of/ay x2z (2)


2
Bj/oz x y. (3)

Integrating (1) with respect to x, we get f = f( 2xyz + sin x) dx = z 2yz - cos x + h(y, z),
where h is a fu nction of y and z only. When we integrate with respect to x we should restore
all the terms with x. The terms without x are treated as a constant when differentiated , and
THE INTEGRAL TH EOREMS OF VECTOR ANALYSIS 151

integration with respect to x cannot restore them. Similarly, integrate (2) with respect to y to
get
J
1= x 2z dy = x 2z y + g( X, z),

where g(x, z) is a function of x and z only. Integrating (3) with respect to z gives us I =
f x 2y dz = x 2yz + k (x, V), where k(x , y) is a fu nction of x and y only. Compare the three
results:
I(x, y, z) = x 2yz - cos x + h(y, z ) = x 2yz + g(x, z) = x 2yz + k(x, V) .
We conclude (by insp ction) t hat g(x , z) = k( x, y) = - cos x + C and h(y, z) = C, where Cis
a constant. Thus,
I(x , y, z) = x 2 yz - cos X + C.

6. (a) This fact should already be familiar (see Example 6 in Section 2.6) . Here are t he details.
We use the chain rule to compute (alox )(1 /J x2 + y2 + z2 ) = -xl (x 2 + y2 + z2)3/2. Then,
by sym metry, we get

"V (~) = '\7 ( 1 ) = - (xi + yj + zk) = ~ = -r


l' J x2 + y2 + Z2 (x 2 + y2 + z2)3/2 IIrl13 r3 .

(b) Since F is the gradient of a function j, fcF . ds = I (c(a)) - I (c (b)) for all paths c(t)
beginning at a and ending at b, unless the path passes t hrough the origin , where 1/r is not
continuous. Thus , t he work done by F in moving a particle from ro "to 00" is

1 _ lim ( ~) _ 1

J x 2 + y2 + z2 r-+ oo r -..;x 2 + y2 + z2 .

Notice th at this tells you it is more difficult to move a particle from a position near the origin.
9. T he reader should verify that '\7 x F = O. Hence, F is a gradient of some function I(x, y, z).
Integrate the i component with respect to x, the j component with respect to y, and the k
component wi h respect to z. Comparing the results, we see t hat I(x, y, z) = eX sin y + z 3 /3.
Also, we compute c(O) = (0,0 , 1) and c(l ) = (1 , 1, e). Since F is a gradient, we have

1
1
e3
c F· ds = l(c (l)) - l(c(O)) = esin 1 + 3 - 3'

13. (b) F is not the gradient of a scalar function f. If such j were to exist, then a I I ax xy and =
=
of I 8y xv· By t h equality of mixed partials, we would expect that 8 I I 8x8y 8 I I 8y8x.
2 2
=
But in this case, (818y)(8f!8x ) = x and (818x )(8118y) y. =
15. (b) To show that F is conservative, we can show t hat it is a gradient . (We can also show
that anyone of the four conditions in theorem 7 is met) . We will use the same method as in
exercise 13. The partial derivatives are:

8 ( 2X ) (2 x )( - 2y) -4xy
8y y2+1 = (y2 +1)2 = (y2 +1)2 ;

8 (-2y(x 2 + 1)) - 4x y

8x \ (y2 + 1)2 = (y2 + 1)2 '

Since these part ials are equal, F is conservative. Finding I so that F = "V I makes the evaluation
of the path integral easy. You should verify t hat if I(x, y) = (x 2 + 1) /(y2 + 1) , then F = '\7 f.
Then substitute x = t 3 - 1 and y = t 6 - t to get I (t) = [(t 3 - 1)2 + 1]/[(t 6 - t)2 + 1], and so
1
[ [(t3 _1)2+ 1]1
lc F· ds = (t 6 _ t)2 +1 o::::;;-l.
152 CH APTER 8

18 . First, we compute "V . F = (8/8x)xz + (8/oy)( -yz) + (%z)y = z - z + 0 = O. Thus, there


exists a G such that F = "V x G . To find G , use the result of exercise 16:

G1 =
lo
z
-yt dt -
lY0
t dt
_
= -yz-2 - -,
2
y2
2
G2 =- lz
0
xt dt
- xz
= --,
2
2
and G 3 = O.
Hence, G = (_~)[(yz2 + y2)i + xz 2jJ. It is a good idea to compute curl G to verify your
answer:

"Vx G 21 I a/ax J
a/oy k
8/8z = ~l (- 2xzi + 2yzj + (z2 - z2 - 2y) k)
yz2 + y2 xz 2 0
xzi - yzj + yk = F.
Note that G is not unique; for example, arbitrary constants may be added to each component
of G, or any gradient may be added to G, and "V x G would still be equal to F .

23. (a) Recall from chapter 4 that F is not irrotational when curl F # O. Thus, we com pute
curl F :
J k
"V x F = I 0/ ox 0/ 8y 8/ OZ I = 2k # O.
- y x 0
(b) Let c(t) = (x(t), y(t), z(t)) be the trajectory of the cork. By the definition of flow lines,
c/(t ) = F(c(t)), and so c/(t) = (-y(t),x(t),O) . Equi valently, we have the following system of
differential equations :

Xl (t) -y(t) (1)


yl (t) x(t) (2)
Zl (t) o. (3)

Equati on (3) can be solved easily. Its solution is z(t) = constant. T aking one more deri vative of
(1) with respect to t yields X"(t) =
_yl(t), but (2) implies that x"(t) - x (t), or x"(t) + x (t ) = =
O. T his equation represents a harmonic oscillator. T he famous solution to the differential
equation x "(t) + x(t) = 0 is

x(t) =Asint+B cos t,

where A and B are cons t ants. If t he reader is un familiar with the techniques of solving
ord in ary differential equations , the reader may verify this is indeed the solution by substitution.
Similarly,
y(t) = Csin t + Dcos t ,
where C and D are constants. Since x"(t) = -y'(t), we differentiate x and y and compare
terms to find that C = B and D = -A. T herefore, y(t) = B sin t - A cos t. Squaring x and y
and addi ng them gives us

x2 + y2 A 2 sin 2 t + 2AB sin t cos t + B 2 cos 2 t + B2 sin 2 t


-2A B sin t cos t + A2 cos 2 t = A2 + B2.

Since A2 + B2 is a constant, we recognize t he equation x 2 + y2 = A 2 + B 2 as the equation of


a circle of radius VA2 + B2 centered at (0,0) . Thus, the cork has a ci rcu lar trajectory about
the z axis in a plane parallel to the ~'y pl ane.
(c) As y increases, x decreases, since xl(t) = -yo We also kn ow t hat the cork is goin g in a
circle. T hus t he cork is revolving counterclock wise.

24. (c) T he property of being rotational is a local property, t.h at is, the field is rotational at a
poin t. In exercise 23, the cork whi rls while going around in a circle, but in exercise 24 , the
cork does not. Single trajectories have little to do with the r otationalness of the fluid.
THE INTEGRAL TH EOR EMS OF VECTOR ANALYSIS 153

8.4: GAUSS' THEOREM


GOALS
1. Be able to s ate and use Gauss' theorem.
2. Be able t o use Gauss ' t heorem to compute a double integral over a closed surface or a triple
int egral over a volume enclosed by a surface.

STUDY HINTS
1. D efi nition. A clos ed surf ace is a surface wh ich, rough ly speaking, must be p unctured in order
to get into the region it encloses . The enclosed region is denoted Wand the closed surface is
denoted oW.
2. Gauss} dw ergence theorem. If oW is a closed surface, then

fffw (diV F) dV = fhw(F.n ) dS .


T hus, a triple integral is reduced to a double integral , or vice versa. Compare th is to the
divergence theorem in the plane (section 8.1) .
3. Physlcal interpretation. div F(P) is the net outward flow at the poi nt P per unit volume. If
div F(P) > 0 material flo ws out from P, and if div F (P) < 0, material flows in towards P. If
div F(P ) = 0, t he vector field is divergence free, that is, what goes in must come out .

SOLUTIONS TO SELECTED EXERCISES


4. (a) For the faces parallel to the yz plane, the outward pointing unit normal vectors are j and
- i, respectively. For those two faces , we have

fiaw F'dS
1 1
= 1 1 \ i + j + k) .idY dZ + 1 1 \i + j +k).( - i)dy dz
111\ i + j + k) . (i - i) dy dz = o.
T he outward pointing unit normal vectors n for any two parallel faces are the exact opposite,
so the integrals over any two parallel faces of the cube cancel. Therefore, the int gral is O.
Next , we calculate that div F = 0, so by the divergence theorem, t he desired integral is

fffwdiVFdV =O.
6. (b) First , we see th at div F = 1 + 1 + 1 = 3, so by the di vergence theorem ,

flaw F . dS = 1ffw div F dV = 3 fffw dV,

which is t hree times the vol ume of W. T he problem now is to find th volume of the region of
interest. Use "cylindrical" coordinates. We will let f) range fro m -1r / 2 t o 7r / 2 since x ~ O. In
°
addition , we have :S r :S 1 and x 2 + y2 = 1'2 :S Z :S 1. Therefore,

fli W
dV = J,"/2 1111r dz dr de =
-1f' / 2 0 r 2
7r
11
0
r (l - 1'2) dr = 7r (1 1)
-
2
- ­
4
7r

Thus,

flaw F· dS = 3 (~) = 3:.

154 CHAPTER 8

10 . Use the divergence theorem . We have div F = (x 2 + y2)2, so


fhsF . dS = f fis (x2 + y2)2 dV.
Using cylindrical coordinates, the cylinder S can be described by 0 ::;: T ::;: 1, 0 ::;: B ::;: 21!', and
o::;: z ::;:
1. Since the Jacobian is r, we get

fhsF . dS = 121<1110 1
T'
4
r drdz dB = 21!' 11 5
r dr = i'
16. From section 8. 3, we know that curl F = 0 implies that F = \7 f, so t aking the divergence
of both sides gives us div F = \7 . (\7 J) =
\7 2 f. But we are given th at div F = O. Hence,
\7 f = O.
2

18. By the divergence theorem , fffw div F dV = f fow F . dS . G iven th at F is t angent to the
surface, we know t hat F is perpendicular to t he unit normal of the surface S, and so F · dS = O.
Hence, IIIw
di v F dV = O.

8. 5: S OM E DIFFERENTIAL EQ UATIO NS O F MAT H E MATICAL P HYSI CS


GO ALS
1. Be able to use vector an alysis techniques in applications such as conservation laws , heat
t ransfer and electrom agnetism .

STUDY HINTS
1. N otation. There 's a lot of new notation , so you m ay find it useful to make a list of new symbols
and t heir definitions.

2. Old concepts. No new vector calculus material is presented in this section . Only the term inol­
ogy has changed . You should be able to justify each step with the knowledge t hat you have
acquired in the previous ch apters.

S OL U T I ONS T O SELE C TED EXERCI SES


3. Using t he tr ansport equation for p, we have

:t f Jfw
p(x, y , z, t ) dx dy dz = fJfw (: + pdiv F) dx dy dz

Jf /w ( ~: + \7 P . F + pdiv F) dx dy dz

Jf fw(~: + \7 . (PF )) dx dy dz
o
for any HI in R3. Thus, the integrand must be identically 0, i.e.,

at + \7 . (pF) = 0,
ap

which is t he law of conservation of mass.

8. Given \7 2 rjJ = 0 and V = \7rjJ , we want to show that p(aV lOt + V· \7V ) = - \7p if av lot = O.
Using exercise 7, V . 'VV = ! 'V (II V I1 2) + ('V x V ) x V. Since V is a gradient , 'V x V = O. It
suffices to show that p [ ~\7( I IV I 1 2) l = - 'Vp. Choose p = -(pI2) IIV W.
THE INTEGRAL THEOREMS OF VECTOR ANALYSIS l55

9. Start with Ampere's law: \7 x H - fJE / fJt = J . Take the divergence of both sides: div J =
\7 . (\7 x H ) - \7 . (8j&t )E = 0 - (fJ / fJt )(\7 . E ) (the change of order of t he two derivatives is
j ustified since \7 takes the derivatives of spatial variables only). Now , by Gauss' law, \7 . E = p.
Combining the two results, we get div J = - op/ at , or div J + fJp/ fJt = 0, which is the equation
of continuity.

10. (a) By definition , the integral of the Poyn ting vector field is

By Gauss' theorem, we may rewrite the right-hand side as

JJ1 \7 . (E x H) dV.

Equation 8 from the table of basic identities of vector analysis in section 4.4 tells us that
div(F x G ) = G . curl F - F . curl G . Thus, the integrand above is

V · (E x H ) div(E x H ) = H . curl E - E· curl H


H . (\7 x E ) - E . (\7 x H )
H · O - E· J = -E· J

since x E = 0 and V x H = J. Therefore,

Jis p . dS = JJ1 \7. (E x H ) dV = - JJ1 E . J dV.

8.6: D IFFERENTIAL FORMS


GOALS
1. Be able to add and wedge multiply k-forms.

2. Be able to integrate and differentiate k-forms.

3. Be able to tate Green s, Stokes' and Gauss' theorems in terms of k-forms.

STUDY HI TS
1. Notation . In this textbook, w is used for I-forms, 7} is used for 2-forms and IJ is used fo r
3-forms. (This is generally the case, but not always .)

2. Definition .

(a) O-forms are scalar functions.


(b) l-forms contain dx, dy, or dz .
(c) 2-forms contain dx dy , dy dz, or dz dx in the order specified .
(d) 3-forms contain t he expression dx dy dz .

These definitions will differ slightly in higher dimensions.

156 CHAPTER 8

3. Integra tion.
(a) I-forms are integrated like line integrals.
(b) 2-forms are integrated like surface integrals. Notice that each term of the double integral
is equ a.tion (2 ) contains a Jacobian. The variables in the J acobian's "numerator" follow
the cyclic nature of the differentials.
(c) 3-forms are integrated li ke ordinary triple integral s.
4. W edge produ cts. Different forms may be multiplied. In our discussion , you can only multiply
a k-form and an I-form if 0 ~ k + I :::; 3. Otherwise, products are zero. Be aware that
w /\ J1 = (-1 )101 (I-' /\ w) . This property is called anticommutativity. Notice that dx /\ dx =
dy /\ dy = dz /\ dz = O. The other properties are similar to t hose of real number multiplication.
5. Differen tiation. If f is a O-form, df is like the ordinary derivative. The "sum" rule (linearity)
remains the same as in one-variable calculus. The "product" rule differs slightly and d(dw) = 0;
tbis last ident ity captures the two identities 'V x 'V f = 0 and 'V . ('V x F ) = 0 in one formula.

SOLUTIONS T O SELECTED EXERCISES


l. (d) Since this is a 2-form wedged with a I -form , i. e., a 3-form, the product is a (2 + I)-form.
By the distribut ive property, we get
w/\ry = (xy dy dz + x 2 dxdy )/\(dx + dz )
xy dy dz /\ dx + x 2 dx dy /\ dx + x y dy dz /\ dz + x 2 dx dy /\ dz
x y(dy dz /\ dx ) + x 2 (dx dy /\ dx ) + xy(dy dz /\ dz) + x 2(dx dy /\ dz)
= xy dx dy dz + x 2 dx dydz = (xy + x 2) dx dydz .
We used the anticommutativity fact that dydz /\ dx = (-I)2( dx /\ dydz) . Also, we used the
associativity property along with the identities dx /\ dx = dz /\ dz = O.
3. (b) When we differentiate this I-form, we get a 2-form:
dw = [d(y2 cos x) /\ dy + (_I)0(y2 cos x ) /\ d( dy )] + [d(x y) /\ dx + (-l)o(xy) /\ d(dx)] + d(dz)
= [(_ y2 sin x dx + 2y cos xdy) /\ dy + 0] + [(y dx + xdy) /\ dx + OJ + 0
= _ y2 sin x dx dy - x dx dy = (_ y2 sin x - x ) dx dy
since dx /\ dx = dy /\ dy = 0 and dy /\ dx = - dx dy.
(e) When we differentiate this 2-form, we get a 3-form :
dw = d[ ( X2 + y2 ) dyJ/\ dz +(-1)1( x2 + ~ ) dy /\d( dz )

= [d( x 2 +
~ ) /\ dy + (_1)0 (x 2 + y2) /\ d( dy)]/\ dz + 0
= [( 2x dx + 2y dy ) /\ dy]/\ dz = [2x dx /\ dy + 2y dy /\ dy]/\ dz = 2x dx dy dz
= d(dy) = dy /\ dy = O.
since d(dz)
4. Notice t hat dx /\ dx dy = dy /\ dx dy = dy /\ dy dz = dz /\ dy dz = dz /\ dz dx = dx /\ dz dx = O.
The derivative of F dx dy is
oF of dy + -0OFdz ] /\ dx dy + (-1) ° F /\ d( dx dy)
[-8 x dx + -;:;-
ay z
of of of of
ox dx /\ dx dy + oy dy /\ dx dy + OZ dz /\ dx dy == o z dz /\ dx dy
of of
az
oz dz /\ (d:c /\ dy ) = (dz/\dx )/\ dy
of of
- 7h (d:c /\ dz ) /\ dy = - 7h dx /\ (dz /\ dy )
of of

8z dx /\ (dy /\ dz ) = fu dx dydz .

THE INTEGRAL TH EO REMS OF VECTOR ANALYSIS 157

Si milarly the derivative of G dy dz is (oG j ax) dx dy dz and the derivative of H dz dx IS


(oH j oy) dx dydz . Therefore,

OG 8H OF ) .
dl1= ( ox + ay + Tz dx dydz= (d1v V)dx dy dz .

8. By a direct calculation, w note that as is the unit circle in the xy plane. The parametrization
is (x, y, z) = (cos t, sin t , 0), 0 S; t S; 27r. T herefore,

is w = 12Tr [(cos t + sin t)O + (sin t + 0)( - sin t dt ) + (cos t + O)(cos t dt)]

= inr (- sin 2 t + cos2 t) dt = io


r
2Tr 2Tr
cos 2t dt = S1;' 2tl2Tr
0 = O.

By Stokes theorem, the above calculation is equal to ffs dw . We compute dw:

d = [d(x + y) !\ dz + (x + y) !\ d(dz)] + [d(y + z) !\ dx + (y + z) !\ d(dx)]


[d(x + z ) !\ dy + (x + z ) !\ d( dy)]
= [(dx + dy ) !\ dz ) + [(dy + dz) !\ dx ) + [(dx + dz) !\ dy)

because d(dx) = d(dy) = d( dz ) = O. This simplifies to

dx A dz + dy !\ dz + dy !\ dx + dz !\ dx + dx !\ dy + dz !\ dy
= -dz dx + dy dz - dx dy + dz dx + dx dy - dy dz = O.

Therefore, we have ffs dw = 0, aLso.

l! . By Stokes theorem, we have fJaR T} = f f f R d1J. We will look at the right-hand side:

d(x dy dz + y dz dx + z dx dy)
= d[(x dy Adz) + (y dz !\ dx ) + (z dx A dy))
= [d(x dy)!\ dz + (-l) l (X dy A d{dz ))] + [d(ydz ) !\ dx + (-1/ (y dz!\ d(dx)))
+[d(zdx)!\ dy + (-1)1( z dx !\ d(dy))]
= [dx A dy + x!\ d(dy ))!\ dz + [dy !\ dz + y!\ d(dz )) !\ dx + [dz A dx + z!\ d(dx))!\ dy.

Since d(dx) = d(dy) = d(dz ) = 0, we get


(dx A dy) !\ dz + (d y !\ dz ) !\ dx + (dz !\ dx)!\ dy
= dx dy dz + dx !\ (dy !\ dz ) + (-dx !\ dz ) !\ dy
= dx dyd z + dxdydz - dx !\ (dz Ady)
= 3 dx dydz.

Thus, the right-hand side is ffJR3dxdyd z. We recognize f ffRdx dy dz as the volume of R.


Dividing by 3 gives us the desi red result.

SOLUTIONS TO SELECTED REVIEW EXERCISES FOR CHAPTER 8

2. By Gauss' t heorem, we have ffaw H· dS = fffw C'v· H) dV = fffw( div H) dV, so we need
to how that div[F x (\7 x G )] = (\7 x F ) . (\7 x G) - F · (\7 x \7 x G). By formula 8 in the
table of basic identities of vector analysis in section 4.4, we have

div(A x B ) = B . curl A - A . curl B = B· (\7 x A) - A · (\7 x B).

Now let A =F and B = \7 x G . This gives the desired result since a . b = b· a.


158 CHAPTER 8

4. According to Green's theorem, Ie P dx + Q dy = IID( 8Q/o x - {)P/fJy) dx dy, where C is the


boun dary of D and C has a counterclockwise orientation. W ith P(x, y) = x 2 y and Q (x, y) = y,
the right-band side is easily computed as

fL(0 - x
2
) dx dy = 111:( _x
2
) dy dx = 11 ( - X2yl:=x3) dx

1\-x + x ) dx= (_;4+~6)1:


3 5 1
12
For the left-hand side , first param etrize the curve C 1 : Y = x3 as x = t, y = t 3 , 0 ~ t ~ l.

11 11
T hen
6
2
1 x 2 y dx+ y dy=
C, O D
t 2 (t 3 )(dt) + (t 3)(3t 2 dt ) =
6 0
For the curve C 2 : Y = x, a counterclockwise parametrization is x = 1 - t, Y = 1 - t , 0
4t 5 dt =4t- 11
3
~ t ~ l.
Then

[ x 2y dx
le.
+y dy = ~O( [(1 - t)2(1 - t)( -dt) +(1- t)( -dt)l = lot [(1 - t)3 +(1 - t)l( -dt).
Substitu ting u = 1 - t yields
0
[ 0 3
I I (u +u) du =
(u4+ '2u
4 2
) 1
1 -4'
3

Thus, the left-h and side is ~ - ~ = - 112 ' and G reen 's theorem is verified for this case.
7. (a) To show that F is conservative, the easiest. thing to do is to show that Y' x F = 0:
J k
Y'x F a/ax %y %z
6x y(cos z) 3x 2 (cosz) - 3x 2 y(sin z)
(- 3x 2 sin z + 3x 2 sin z, 5xysin z - 6x y sin z , 5xycos z - 6xycos z)
O.

(b) If F is the gradient of some f (x, y, z), then /(x, y, z) must sat isfy
6xycosz o//ox (1)
3x 2 cosz of/oy (2)
-3x 2 ysin z of/oz. (3)
Integration of (3) with respect to z gives f( x, y, z) = 3x 2 ycos Z + g(x, V); integration of (2)
with respect to y gi ves f (x,y , z ) = 3x 2 y cos z + h(x,z); integration of (1) with respect to x
gives f( x , y,z) = 3x 2 ycosz + k (y,z). Comparing these three f's, we can see that f( x , y ,z ) =
3x 2 y cos z + C .
(c) Since F is a gradient, we only need to evaluate f at the endpoints to calcula.te the line
integral . We get
n/2

1
F · ds
c
= f (c(1r/2)) - f(c (O)) = 3(cos 3 tI)2(sin 3 tI)
I0
= (J.

9. We want to com pute IIIw V' . F dV where W is the unit cube. By Gauss' theorem,

f ffw Y' . F dV t t ~o(


= lolo (5z + x 2 + y) dx dy dz

1111 (~ + 5z + y) dy dz

1 (1-+ +-1)
o
1
3
6z
2
dz = (1 +-+
-
3
1 ) 23
2
3 =-.
6
THE IN TEGRAL THEOREMS OF VECTOR ANALYSIS 159

13. (a) We compute

'\7 1 = (al/ax)i + (al/ ay)j + (af/az)k = 3y exp(z2 )i + 3x exp (z2)j + 6xyz exp (z2 )k.

(b) '\71 i a gradient, so the integral is l (c(11")) - l (c( O)). Since I (x , y, z) = 3xyexp(z2 ),
we have I (c(t)) =
3(3cos3 t)( in 2 t ) exp (e2t ) ; thus l (c(11")) 0 and l(c(O)) O. Th refore, = =
Ie '\71 . ds = O.
(c) Let A be the region whose boundary is c(t). Then, by Stokes' theorem ,

1 '\71 · ds = 1 ['\7 x ('\7 f)] . dS.

This is 0 becau_ the uri of a gradient is always OJ the oth r part of Stokes' theorem has been
illustrated in part (b).

14. By Green' heorem, Ie x 3 dy - ~ dx =


IID(3x 2 + 3y2 ) dxdy, where D is the unit disc. Now
use polar coordinates : x = r cos 8, y =
r sin 8. The unit circle is described by 0 ~ r ~ 1 and
o ~ 8 ~ 2- . ince the Jacobian is r, the integral b omes

12"1 11
1
3r 2 . r dr d8 = 211" 3r 3 dr = 211" . -3 = -11"
3 .
o 0 0 4 2

17. By Green' eo em .

Area = Jl dz dy =~ l (x dy - y dx)

= ~ 1· (a sin 8 cos 8)(2asin 8 cos 8) d8 - ~ 1'fr (a sm 2 8)[a (cos 2 8 - sin 2 8)] de


_
Jo
= 1 a2 fo ( 2sin 2 8cos 2 8 - sm 2 ()cos 2 () + sin 4 8) de

= ~Q2 f
2 Jo
in 2 O(cos 2 () + sin 2 () ) d8 = a
2
2
r sin 2 8 d8
Jo
;~ 1 C-~os 2()) d()

= Q
2
? (~ - Si~2()) = 11":2.
I:
20. (c) If F ' a gradient, then curl F = O. We compute
,
I
.
J
curl F = a/ax a/ ay
2x~ x 2 z 3

Excep a the origin, curl F is not 0 , so F is not a gradient.

TEST FOR CHAPTER 8

1. True or false. If false, explain why.

(a) For any region to which Gre n's theorem applies and if C is traversed counterclo kwise,
the line integral Ie (4x - sin x + eX + 2y) dx + (4x + cos y + 2Y ) dy has positive value.
(b) The vector field

F (z. y z) = (y 2 e· cos z)i + (2xye Z cosz + 2ycos(y2))j + (xy2 ez cos z - xy 2 ez sin z)k
is a conservative vector field.
160 CH APT ER 8

(c) If F is a gradient, then curl F = O.


(d) The line integral Ie P dx + Q dy is independent of path if a P / o y = oQ / ax on all of lR 2.
(e) If y = l (x ) is a posi tive function on [0 , 1] , then th e area under the curve is

~ 11 [t (J' (t) dt ) - I(t) dt].

2. Let C be the perimeter of a trapezoid with vertices at (0 , 0), (4, 0) , (4, 1) and (2,1). IT C is
traversed in t he counterclockwise direction, starting and ending at the origin, calculate

fc (2x + x3 - Y + sin x ) dx + (eY - x - tan y + Vii) dy.

3. (a) Compute 'il l for I (x , y, z ) = 2X 2 Z 2 sin ycos (e X


) .

(b) Let F = 'il l as in part (a). Com pute Ic F. ds where C is t he portion of t he pl ane
2x + y + Z = 3 that lies inside the sphere x 2 + y 2 + Z2 = 16.
4. Let F (x , y, z ) = (3x + ysin z, x 2 + y + ze x , xe:ll y + z ). Let P be t he parallelepiped spanned by
the vectors (1, 2, 1), (3, 0,1) and (- 1,3, 2) and having one vertex at t he origin . Compute the
i.ntegral of F over oP.

5. (a) Compute curl F for F( x , y, z) = - 2yi + 2xj + xyz 3 k.


(b) For F as in part (a), compute IIs curl F· dS where S is the semiellipsoid
x2 2)1/ 2
z= ( l - - - !L
4 9 .

6. A curve is described parametrically by x = sin t , y = sin 2t , 0 ~ t ~ 1r. F ind the area enclosed
by t he curve. (Hint: sin(m x) cos(n x) = ~ [sin (m - n)x + sin(m + n)x].)
7. A solid W is described by -1 ~ x ~ 1, 0 ~ y ~ 4 - x , 0 ~ z ~ 10 - x 2 - y. Compute the fl ux
of F( x , y, z ) = (y + z2 )i + (x y - sin z )j + (5xe!l )k across oW .

8. Suppose t hat Iis curl F · dS = 5 for a given F and S is the cone y = 4 - x2 - Z2 , 0 ~ Y ~ 4.

(a) Compu te f IT curl F ·dS , where T is the semiellipsoid y = 5(1 - x 2 /4- z2 /4)1/2 , 0 ~ Y ~ 5.
(b) Com pu te Ilu curl F . dS , where U is t he paraboloid (y + 2)2 = x2 + z2, - 2 ~ y ~ O.
9. For a closed curve C that is traversed in a counterclockwise di rection, it has been determined
that Ie y dx - x dy is -6 . C alculate Ie(x + 2y) dx + (5x - ye!l) dy.

10. (a) Show that F(x , y , z ) = {2xyz3 + z)i + (X2Z3 + l)j + (3x 2yz2 + x )k is a conservative vector
field .
(b) Find a fUDction I such t hat 'ill = F .
(c) Suppose the same force field F as in (a) causes a ba by to m ake a random crawl fro m
(0, 0, 0) to (5, e, 0) where she knocks over a priceless obj ect. How much work is done by
the force field on the baby?
(d) T he force field F then causes t he baby to crawl up some cabinet drawers to (I, 1, 1). How
much work was done by the force field in taking the baby from (0,0 , 0) to (1 , 1, I) ?
161

9 SAMPLE EXAMS

9.1: COMP REHENSIVE TEST FOR CHAPTERS 1 - 4

1. (a) Find the relative extrema of f(x, y) = x2 - 2x + y2 - 1.


(b) For the same t, find the relative extrema on the curve x 2 + y2 = 1.
2. (a) Fido is on a strange mo untain which has a shape described by M (x,y) = e-Y xsiny. If
Fido is at (1,1':/ 2, e- rr / 2 ) , what is the d irection Fido must take to get down the mountain
as fast as possible?
(b) Fido has to follow a certain trail of tree trunks (which he has previously m arked) along
e(i) = (2t , eexp (- t 2 ), 2tsin(t 2 )) . How fast is Fido's alt it ude changing at t = I?
3. (a) Given the vector field F(x, y) = (y, -x) , find an expression for a flow line c(t) for F by
taking a good guess.
(b) Do the same fo r F (x, y) = (2x , - y).
4. (a) Find the arc length ofe (t) = (2t,t2), O:'S t:'S l.
(b) Find the arc I ngth of e(t) = (2sin t,sin 2 t ), 0 :'S t:'S 1': / 2.
(c) Explain your answers.

5. A wooden box is to be m ade with $120.00 worth of wood . The lid is to be made from wood
that costs $2 .00 per square inch, and the rest of the box is to be made from wood that costs
$2.50 per square inch . What is t he biggest box that could be made?

6. (a) Find an equation of the line of intersection ofthe planes 3x + 2y -z = 7 and x -4y+ 2z = O.
(b) Find the equation of the plane wh ich contains the points (1 ,3, -2) and (0, - 2, 1) and is
perpend icular to the plane 3x - y - 2z = 5.
7. Let t: B C ~ 2 -t ~3 be defined by (u,v) -t (uv 2 ,v 3 - u,v sinu), and g: B C IR3 -t IR 2 be
defined by (x , y, z) -t (yz e"', y3 cosx z).
(a) Calculate D(J 0 g)(O , 1, 0).
(b) Calculate D (g 0 f )(O, 1) .

8. An aquarium m anufacturer bas advertised that its tanks hold exactly 0.49 m 3 of water . Due
to production error , t he tanks have di mension 1.01 m x 0.72 m x 0.67 m rather than the
specified 1.00 m x 0.70 m x 0.70 m. Use the linear approximation to estim ate t he error in the
advertised capacity of t he tank .
9. A budding bioengineering st udent who is an amateur ornithologist wanted to fly sou th for
Christmas vacation, but couldn 't afford an airline ticket. He knew that his pet swallows also
wan ed to fly south for t he winter, so his plan was to tie lis feathered friends to a hang glider
and have them fly him home. His well-trained swallows had a veloci ty vector of 3i + 4j + k
(km/ hr) until they r ached their cruising altitude at (0, 0, ~ ). At that point, they continued
along the same path, but with no change in altitude, (Assume the Earth is flat. )
(a) What was the original starting point at (x, y, O)?
(b) How long did it take to reach cruising altitude?
162 CHAPTER 9

(c) When they reached (0, 0, ~ ), a strong wind with veloci ty 2i + j affected the flight. What
was the swallow's veloci ty vector to maintain a total velocity of 3i + 4j?
(d) After 3 hours, the bioengineer calculates that they will pass over his worst enemy's horne,
where some of his pets will give a 21-bird dropping salute. What is the position of the
enemy's house?
(e) Assuming that the swallows were unable to compensate for the wind, how far from the
target point would they be after 3 hours?

10. Tightwad Terry, the miser , has performed a computer analysis of his earning power. He has
determined that his earning fun ction is $(t, u , v, w) = t 2 e ti + v sin w .

(a) Compute $t, $u , $t1 and $w ·


(b) Assuming T ightwad Terry wants to in crease his earnings as fast as possible and he was
at (1 , 0,2, 0) on his $ graph, how should he change t, u, v and w?
(c) If he has exactly one unit of each resource (represented by t, u, v and w), what. is the
largest increase in earning th at he can expect? Note that he can use fractional amounts
t
of resources, such as of t and ~ of w.

9.2: COMPREHENSIVE TEST FOR CHAPTERS 5 - 8

1. (a) Evaluate
Jin 3(x + y)e(x-v) dx dy

over the region bounded by the lines x + y = 1, x +Y=- 1, x - y = 1 and x - y = ~ l.

(b) Evaluate
1111 siny2 dydx .

2. A block has a slanted top described by x + y + z = 2. Its edges are perpendicular to the xy
plane, and the botto m of t he block is formed by the triangle with vertices (1,0,0), (0, -1,0)
and (0 , 1,0). What is the volume of this block?

3. Find t he m ass of a wall described by a$ y $ _x 2 - 2x + 3, - 3 $ x $ 3, having cJiensity


o(x, y) = 21yl + 3.
4. Let F (x , y) = (eX cos 3y, - 3ex sin 3y).
(a) Find an f( x,y) such that Vf = F for aJi (x,y).
(b) Evaluate IeF . ds for the path c(t ) = (cos t, sin t), °$ t $ 211'.
(c) Compute V x F.

(You should be able to do parts (b) and (c) in your head.)

5. (a) Suppose information flows through an idiot's skull (a surface descri bed by x 2 + y2 + 2z2 =
1) at the velocity 2xi + 3yj + zk. After brai n surgery, his mW1bed skull, which remained
the same shape, receives infor mation at the velocity 5xi + yj + 3zk. Thinkrng of this as
a fl ux of information through a surface, did the surgery help? Explain .
(b) A particle moves in a path c(t ) = (2t, 3t , t ) in a force field F = (2x, 2y, 4z). What is the
work done in the ti me interval 0 $ t $ 5?

6. (a) A contact lens can be described as a cap of a sphere of radius R cut ou t by a cone of angle
1T / 4. Find the surface area of the lens. (Hint: Set up the lens in a correct and convenient
coordinate system. )
SAM PLE EXAMS 163

(b) Slice a sphere anywhere with 2 parallel plane which are separated by
a fixed di tance d. The bands obtained always have the sa me surface
area. Prove this. What is the area? (Hint : Use spherical coordinates.)

7. (a) Find the volume enclosed by the surface

a;2 + 9y2 + z2 - 2x + 54y - 10z + 106 = O.

(b) F ind an expression for the surface area of the surface above.

8. Which of the following vector fields are conservative?

(a) F(x, y, z ) = (x 2 + l/ x , In(y + 1) , z)


(b) F (x, y, z) = ( (x 2
- 3x
+ y2 + z2)3/2' (x2
-3 y
+ y2 + z2)3/2' (x 2 + y2
-3Z)
+ z2)3/2
(c) F( x, y, z) = (3yz , 2xz , 5xy)
9. Complete t he following st atement: A vector field F is conservative if _ _.

(i) T here is a vector field G such that F = curl G.


(ii) T here is a scalar function fsu ch that \7 f = F.
(iii) div F = O.

(a) (i) only (b) (i) and (iii) (c) (ii) only (d) (ii) and (iii).

10 . Consider the follo wing argument: given a vector field F,

by Stokes' theorem but

JJ.}wf \7. (\7 x F ) . dS = {


}s=&W
(\7 x F ) . dS

by Gauss' t heorem, so everyt hing is 0 because the divergence of the curl of any vector field is
O. W hat's wrong?

9.3: C OMPREHENSIVE T EST A FOR C HAPTERS 1 - 8

l. If true, state true. If false, expl ain why.

(a) The path integral Ie


27l' ds equals the surface area of a cylinder of radius 1 and height 27l'
ifc (t) = (cost,sin t,t ), 0 ~ t ~ 27l'.
(b) A continuous fu n tion is a differentiable fu nction .
(c) The line integral of a mass density function along a curve is the mass of the curve.
(d) curl F = \7 . F .
(e) 8 2
f /8 x 8z = 8 2f/8 zax for all continuous functions f( x,y,z ).
164 CHAPTER 9

2. M ultiple choice. Choose the correct a nswer.

(1) Which pair of vectors have the smallest angle between them?
(a) i - j , j + 2k
(b) 3i + 2k, - 2j
(c) 2i - j + k, i - j +k
(d) T here is insufficient informat ion.

(2) Green 's theorem requires


(i) continuous fi rst partial deri vat ives
(ii) continuous functions
(iii) any closed curve which is the boundary of a region
(a) (i) only (b) (ii) onl (c) (ii) and (iii) (d) (i) and (iii) .

(3) Let !(x , y, z) = z2 xe x cos(yz) . A particle t ravels along a path c from (0, - 2/ 5, 1T/4) to
(3,5, -1T /2) . If the force acting on t he particle is 'V I, then t he work done on the particle
IS

(a) negat ive


(b) zero
(c) positive
(d) unknown since c is unknown

(4) For any vectors u and v , U· (u x v) =


(i) v· (u x v )
(ii) °
(iii) (v x u ) . u
(a) (i) only (b) (ii) only (c) (i) and (ii) (d) (i) , (ii) and (iii ).

3. Let F = xi + yk. Compute IIs curl F . dS for the following surfaces S:


+ y2 + (z - 3)2 = 1, z ~ 3
(a) x 2
2
(b) 9x + 9y2 + z2 = 9, z ~ °
4. Consider the system

F1(u, v, x, y) u
2
-
2
v + 2x + x y = °
F2(U, v, x, y) 2u + 3v - 5x 2 = 0
(a) Show that one cannot solve for tl and v in terms of x and yat (u, v) = (0 , 0).
(b) Show th at au/ax exists at (u , v, x , y) = (3, 2, -1 , 0). Compute it .
5. Suppose g(u,v) = (uv 2 ,u+2v,v) . At (x, y, z ) = (0,0,0), (u ,v) = (1,2) and

D! = [1-1 0]
2 3 1 .

Let h = 9 0 f. Wh at is Dh (O, 0, o)?

6. A surface is parametrized by

x = u
2
, Y= v
2
, Z = uv , °~ u ~ 2, °~ v ~ 2.
SAMPLE EXAMS 165

(a) Find the tangent plane at (u, v) = (1, 1) as a function of x and y.


(b) When u = 1, find the arc length of the curve in space for 0 ~ v ~ 2.

7. Let ~ be the surface parametrized by


U
x:;:;;: e cosv, y =e
U
sin v, z = v, 0 ~ u ~ 1, 0 ~ v ~ 1r/2.

(a) F ind the area of the surface ~.

(b) Compute the average of z over ~.

8. Letf(x,y)=x 2 -3x+y-y2+5.

(a) At all critical points (xo, Yo), express (xo, Yo, f( Xo, Yo» in spherical coordinates.
(b) Find the extrema of f on the circle x 2 + y2 = ~.
9. A surface is described by the equation z = 2y + cos 1rX - ft. Consider the point (4, -1, 1).

(a) Find the directional derivative in the direction of i + 2j.


(b) F ind the equation of the tangent plane.

10. A region in space lies between the graphs of z = 16 + x 2 + y2 and z = J x 2 + y2. The region
also lies inside the cylinder x 2 + y2 4. =
(a) Express the volume of the region as a triple integral with cartesian coordinates.
(b) Rewrite your answer in (a) with cylindrical coordinates.
(c) Find t he volu me of the region .

9.4: COMPREHENS IVE T ES T B FOR CHAPTERS 1 - 8

1. P hysical and geometric interpretations.

(a) What do you know about u and v if u . v = O?


(b) What do you know about u and v if u x v = O?
(c) What physical interpretation is associated with a negative divergence?
(d) Give a phy ical interpretation of curl V.

2. Consider the point (1, 1,1 ,1) of the function f(x, y, z, t) = x 2 y + zt - z.

(a) Compute the directional derivative in the direction of (2,0,6,4) .


(b) Is fincreasing faster in the direction of (2, 0, 6, 4) or in the direction of (1, 1,2, I)? Explain
your answer.

3. (a) The cyli nder x 2 + y2 = 4 is cut by the plane x +y+z = 1. Show that the arc length of
the intersecting curve is
J8127r VI ­ cos 0 sin 0 dO.

(b) For the intersecting curve, find an equation for the tangent line at (1,)3, -V3).

4. Find t he m inimum and maximum of x + yz subject to the constraints x +z = 2y and


{( x,y) I (x,y) E [3, 5] x [0,2]}.
5. Let S be the boundary of a box B = [-2,2] x [- 1,1] x [-3,3], F 2xi + 3zj + 2yk and
G = x3 i + 3zj + 2yk.

(a) Com pute the integral of \l . F over B.


(b) Compute ffs G . dS .
166 CHAPTER 9

(c) Suppose t.he origin at the center of B is shifted to (8, -15, 20) and then rotated 30 0 around
the yaxis. Compute IIs F . dS .

6. A hole of radius ~ is drilled through the axis of symmetry of the hemisphere x 2 + y2 + z2 = 1,


z ~ O.

(a) Write the volume of the remaining piece in Cartesian coordinates.


(b) Write the volume of the remaining piece in cylindrical coordinates.
(c) Compute the volume.

7. A painter is scared of heights, so he charges z2 dollars per square

meter to paint objects located at height z. His latest job is to paint

the silo 5 shown here. The height of the cylindrical part is 3 meters

and the radius of the hemispherical part is 2 meters.

(a) Compute IIs dS and interpret geometrically.


3
(b) Compute how much the painter charges.

8. (a) ComputeI;Ix1cos(y2+3)dydx.

roo Joroo Joroo e- ('+'+


(b) Compute Jo x Y Z ')'/' dx dy d
z.
9. (a) Verify Stokes' theorem for F = z3 j + (x 3 - y3)j + y3k over the hemisphere x 2 + y2 + z2,
z ~ O.
(b) For the same F as in (a), evaluate IIs('V' x F) . dS for the surface x 2 + y2 + 5z 2 , z ~ O.

10. (a) Find a vector-valued function /(x, y, z) such that


CD8 y - x z sin(xy)eCDs(x y ) eeDS(XY) ]
D/(x, y, z) = [ - yz sin(xy)e (x )
y2 sin z 2xysin z xy2 cos z .

(b) For the region D shown at the right, let V be

the volume of the solid lying between /(x, y) =


y
x 3 sin y and the xy plane and lying over D. Write

Vin the form III g(x,y,z) dzdydx.

1 x

(c) Rewrite your answer to part (b) in the form III g(x,y,z)dzdxdy.
167

Appen dix
ANSWERS TO CHAPTER TESTS
AND SAMPLE EXAMS

ANSWERS TO T E ST FO R C H APTER 1

1. (a) False; dot products need to be defined for vectors in the same space, so u · v is undefined.
(b) True .
(c) True.
(d) False; le t t he vectors be i, j and k.
(e) False; v x a v = 0 for a being any real number .

2. - 2x - 2y + z =1
3. (a) a =- 5i+ j + k;h = 4i - 2k
(b) s = [0,1 and t = [0, s)
(c) v'I4
4. } 11/15

5. ±6V5
6. (a) l(t) = (1,3, - 2, 0) + t( - 1, --2, 1,1)
(b) (-1 , - 2,1, 1)· (2 , 0, 3, 1)= 2 #0

7. (a) [!3 ~]
(b) 1; area of th pa rallelogr am spanned by (1,0) and (3, -1).
(c) O' volume of th paralJelpiped spa.nned by (-2, 0, - 3), (8, 1, 5) a nd (2, 0,3) is 0; all three
vectors lie in a single plane .

8. (a) (-/3,311"/4, 1)
(b) (- .../6/2, .../6/2 , 1)

9. (a) lu ,vi = 2 ~ 2V15 = IIull II vll


(b) (2, -1 , 0, 1)/3
10 . (a) x +y - 3z =3
(b) 5IIT
(c) 10/ IIT

ANSWERS TO T E ST F O R C H APTER 2

1. (a) False ; let f (x, y , z ) = 1 if x, yor z = °


and f (x , y, z ) = 0 otherwise.
(b) False; should b e perpendicular.
168 APPEN DIX

(c) True.
(d) False; consider z = Ix!­
(e) True.

2. [ ; 16 ]

3. 8.969

4. Positive x direction .

5. z = 12x + 8y - 9

6. 8g / 8x does not exist ; 8g/8y == 0.


7. 8u/8s = 0; 8v/8t = 1
8. (a) 12
(b) lim(x ,y)-+(O,O)!(x,y) does not exist because limx-+o!(x , O) = 1 and iimy-+o!(O , y) = 0;
therefore, !( x, y) cannot be made continuous .

9. ~~ (1, 1) = 2 + e > ~~ (1, 1) = 2


10. (a) (-3.6, -36, - 0.02)
(b) J)ecreases by 3.6
(c ) No; the directional derivative is - ~;~ « -1.

ANSWER S TO TEST FOR CH APTER 3

1. (a) False; a minus sign is missing.


(b) True .
(c) False; the fourth-order derivatives must be continuous to guarantee equality.
(d) True.
(e) False; let !(x , y) = x 2 + y2 on the whole plane. It has a minimum but no maximum.
2. 4. 28; 4.2665

3. Since 8 2 z/8x 2 = 30xy, concave up if x > 0; concave down if x < 0.


4. (0,1,2, -2), (0, -1,2, -2) , (0 , 1, - 2, - 2), (0, -1, -2, -2)
5. (a) and (c) since 8 2 f/ 8 y8x = fj2 f/8 x8y.

6. Minimum = - v'6/2; maximum = v'6/2.


7. (a) Minimu m = -~ ; maximum = 0.
(b) Min imum = -~ ; no maximum.

8. The only critical point is at (- ~, ~). Also, D = (8 2 ! /8x 2 )(82 ! /8y2) - (8 2 f/8x8y) 2 = - 12 <
0, so (- ~ , ~) is a saddle point.

9. (a) 8x/8v =2
(b) Can't be done.

10. (a) (g , c) = (10,5)


(b) (10 ,5) is a m aximum point.
ANSWERS TO CHAPTER TESTS AN D SAMPLE EXAMS 169

A N SWERS T O TEST F OR CH A PTER 4


1. (a) True .
(b) False; limits of integration should be and 11'. °
(c) Fa.lse; the last term should be b x dc/dt.
(d) False; v(t) = (2,0,0) and w (t) = (0,1,0) both have 0 acceleration .
(e) True.
2. Both satisfy the desired conditions.
3. (a) All of ~ 3.
(b) Nowhere.

4. (a) ftr/ 2 J4 + 5 cos 2 t dt


(b) (-4, 0)
5. (a) 6g l/(t)i + 6[(g'(t))2 + g(t)gl/(t)Jj
(b) g(t) must be a constant.
6. z > -1
7. fa4 J~ + 4y2 + 4y2 exp(2y2 - 12) dy
8. (a) and (b) since div F 0. =
9. (a) (3:c 2 - 2:c + 3)[( 2sin(x3 - x 2 + 3x - 4) cos(x 3 - x 2 + 3x - 4))i + (4(x 3 - x 2 + 3x - 4)3_
3(x 3 - x 2 + 3x - 4))j]
(b) pl/(t)c(t) + 2p' (t)c'(t) + p(t)cl/(t)
10. (a) T hey are bot h running counterclockwise on the ellipse 4x 2 + y2 = 4.
(b) 12500i
(c) l (t) = (cos( 4rr /5),2 sin( 4rr /5)) + (t - rr /5)( -4 sin( 4rr /5) , 8 cos( 4rr /5))
2
(d) VI + cos 2 4t dt
4 f 2 11:11:/54

A N SWER S TO T EST FOR CHAPTER 5


x~
1. (a) False; shouJd be 1-2
2 4
fa - dydx.
(b) True.
(c) True .
(d) False; I( x, y) should be nonnegative at all points of D.
(e) False; the right-hand side should be

[1 2
(x
2
+ 4x + v'x) dX] [[11 (y3 - y2 + 4y) dY] .
2.

i11VYl
a a
1 Y
- dzdx
a
dy = l 1 lVY
1

a a
1
-
Z

a
dx dydz

= l 1 iVY
1 1 Y
- dxdzdy
a a a

= 1 1i~l
1 z
- dydx dz
a a x'

= i 111-X'11-Y dydz dx : 4
a a x' 15
170 APPENDIX

181
3. 12

4. (1 - sin(1))/2 by changing the order of integration.

S. (a) 1811"

3 f';
(b ) f -3 9-,,2 ( 2 2

_';9-,, 2 9 - x - y ) dydx

(c) 2711"/2

6. i
7. (a) mv(W) :S fffw f( x,y , z) dxdydz:S MV(W).
(b) The minimum value of f on W is V2/2e 2 at (11"/ 4, 1, 1) and the maximum value is 1 at
(0 , 0, 0). The vol ume of W is 11"/8. Apply these values to the statement in part (a).

8 . (e 8 - 1) /3 by changing the order of integration of x and z.


9. ~
3

10. (a) 5~2

(b) 8 - if¥­
A NS W ERS TO TEST FOR CHAPTER 6

1. (a) True.
(b) False; fooo cos xdx doesn't converge , so Fubini's theorem doesn't apply.
(c) False; the integral in polar coordinates is only being integrated over the region between
x = ±2y and x = 4 - y2 . J
(d) True.
(e) True.
2. 11"2

3. 8/311"

4. 16011"/3

5 . -~

6. (a) 2 J~1 f01 J~l(ll+v)dlldv dw


(b) 2
7. 4811"

8. 211"

9. The integral is improper near z = 0, so the integral should be calculated as f01 f~lI(l/x) dx dy+
f o1 f~ (l/x) dx dy, which diverges.
10. (0 , -10/(4 + 311") )

ANSWERS TO T EST F OR CHAPTER 1

1. (a) True.
(b) False; liT ti x T Ii II is positive and f is positive, so the scalar surface integral is positive.
(c) True.
5\ ERS TO CHAPTER TESTS AND SAMPLE EXAMS 171

(d) True.
(e) False' Ie F(x , y) == xi + xyj and choose any closed curve in the plane x == o.
2. 4A

3. -~

4. 41T

5. (a) C)(O,r/» == (2cosOsinr/>,2sinOsinr/>,2cosr/», 0 S 0 S 21T, 0 S r/> S 1T.


(b) f fs Iz l dS == fa" f02" 12 cos r/>I (4 sin r/» dO dr/>
(c) 321T
6. (a) V/== (2zsiny,2xzcosy,e Z +2xsiny)
(b) e - 5
7. (a) -15x-12y+2z==3
(b) f02 f~l (144u 2v 2 + 72uv 3 + 9v 4 + 144u 4 v 2 + 4U 2)1/2 dv du
8. 1~8
9. 7e + 361
10. 8(3V3 - 2V2)/fJ

ANSWERS TO TEST FOR CHAPTER 8

1. (a) True.
(b) True.
(c) Tru e.
(d) True.
(e) False; the formula A == ~ f aD x dy - y dx must also be applied to the lines x == 0, x == 1
and y == o.
2. -6
3. (a) (4xz 2 sin y cos(eX) - 2e x x 2z2 sin y sine eX))i + 2x2 z2 cos Y cos(eX)j + 4x 2 z sin y cos( eX)k
(b) 0
4. 70
5. (a) xz 3i - yz3j + 4k
(b) 241T

6. ~
7. _ 158

8. (a) 5
(b) - 5
9. 9

10 . (a) V x F == 0
(b) I (x y , z ) == x 2 yz 3 + y + xz
(e) e
172 APPENDIX

(d) 3

ANS W ERS TO COMPREHENSIVE T E ST F OR C HAPTERS 1 ~ 4

1. (a) (1 , 0) is a minimum
(b) (1 , 0), (-1,0)

2. (a) (0, _e- 7r / 2 , -1)


(b) (3e- 1 cos 1 - 2e- i sin 1)/(e- 2 sin 2 1 ­ 8e- 2 sin 1 cos 1 + 1 + 4e - 2 )1/2

3. (a) c(t) = (cos t ,sint)


(b) c(t) = (exp(2t) ,exp(-t))
4. (a) V2 + In(1 + V2)
(b) V2 + In(1 + V2)
(c) The two parametrizations actually trace out the same path .

5. 32V5/3 cubic inches

6. (a) (2 , 1,1)+t(0,:,2)
(b) l:3x + 7y+ 16z = 2

7. (a) OIl
00 9 -1
[ o 0 1
(b) [1- 3 0]
9

8. 0.0021 m 3

9. (a) (-~,-2)
(b) ~ hour
(c) i + 3j
(d) (125,10 , 0)
(e) 5V5/2 km

10. (a) $t = 2te", $u = t 2 eu , $v = sinw, $w = vcosw


(b) Go in the direction of (2, 1, 0, 2)
(c) (2 e1 / 3 /3,4e 1 / 3 /9,0,0)

A NSWERS TO COMPREH ENSIVE T E ST FOR C H A P TERS 5 - 8

1. (a) 0
(0) (l-cosl)/2

2. ~
3. 1848
5

4. (a) J(x , y)=excos3y+C


(b) 0
(c) 0
ANSWERS TO CHAP TER TESTS AND SAMPLE EXAMS 173

5. (a) ar e. helps; the flux is 6V before and it is 9V after , where V = 41211"/3 is the volume
0" e· -ull
(b) 3,5
6. (a) '! - R- 1 - -:-/ 2)
(b) ea 2- Rd.
7.

9. Ie)

10. In he urf ce should no be closed (so it would have a real boundary), wher­
m. the su rface has to be closed (to enclose a volume).

ANSWERS TO CO . HE . SIVE TEST A FOR CHAP T ERS 1 - 8

1. area of such a cylinder is 211", while the value of the integral is 21211" .
pie. in::-3 let z = Ixl; it 's not differentiable at x = o.
(e)
(d) a product , not a dot product .
(e) eed continuously diffe rentiable second partials.

2. (1) (c (3) (b) (4) (d)


3. (a)
(b) 0
Fl / 0 0
4.

(b)
F./8 --1
1 2 3 1-- 0
5.

6. a z ­
(b) - ..IU)l
7. (a) - - 1)
(b) ~
8.

9. a) '4
(b

10. (a) - z'! - y2 - J x 2 + y2 dy dx


r2
(b) - T
16
- 2. f 2 Jo r dr dz + 211" Jr4 rr ~
o Jo r dz dr
(e) 4 - /3
174 APPENDI'X

A NSWERS TO COM P R EHENSIVE TES T B FOR CHAPTERS 1 - 8

1. (a ) The two vectors are orthogonal, or one of them is O.


(b) The two vectors are parallel, or one of t hem is O.
(c) There's more material going in tha.n out; i.e., we have a sink.
(d) curl V . n tells you about the circulation of Von the surface with normal n.

2. (a) 8
(b) In t he direction of (1, 1, 2, 1) because the directional deri vati ve using normalized vectors
is larger in the second case.

3. (a) T he intersecting curves can be parametrized by

(x, y, z) = (2 cos B, 2 sin B, 1 - 2 cos B-2 sin B).

(b) (x , y,z ) = (1,V3,-v'3) +t(-V3,1,V3-1)


4. Maximum = 6, minimum = 3
5. (a) 96
(b) 192
(c) 96

) f 1/2
6. (a 4
l Iv'1'=? n/1 - -y' dz dydx
Vl/4-x 2J O
x 2

(b) f~1r f01/2 fo"l-r rdzdrdB


2

(c) % (1 _ ( ~ ) 3/2)
7. (a) 161!', which is the surface area of the silo.
(b) (5001!'/3) dollars

8. (a) [sin(4) - sin(3)J/2


(b) 1!'

9. (a) 31!'/4
(b) -31!'/4
10. (a) f (x ,y,z) = (ze cos (xY),xy 2 sin z)
(b) tl fO-X'+l fox'sin y dzdy dx
(c) f01 r!';~':yfOx3siny dzd:r: dy

You might also like